PASTEST NOTES 2020- AWAD PAPER 1 (APPLIED BASIC SCIENCE)

Contents Anatomy ...... 2 Abdominal Vessels ...... 2 Abdominal Viscera ...... 5 Abdominal wall and cavity ...... 11 CNS, Brain...... 12 Lower limb ...... 26 Neck ...... 30 Pelvis ...... 33 and breast ...... 36 Spine...... 39 ...... 41 Embryology ...... 50 Surface Anatomy ...... 58 Immunology ...... 87 Pharma and Micro ...... 95 Data interpretation and imaging ...... 107 Neurophysiology ...... 142

Page | 1

Anatomy Abdominal Vessels Only the superior pancreaticoduodenal artery is a branch of the gastroduodenal artery. The inferior pancreaticoduodenal artery is a branch of the superior mesenteric artery.

• The gastroduodenal artery, a branch of the common hepatic, gives off the superior pancreaticoduodenal artery which supplies the head and uncinate process of the pancreas. However, the body and tail of the pancreas are supplied mainly by the pancreatic branches of the splenic artery.

The splenic artery is a branch of the coeliac axis as shown below. It has a tortuous course, passing along the superior border of the pancreas and posterior to the body of the stomach. It can be eroded into by a posterior gastric ulcer.

• The gastroduodenal artery is a branch of the common hepatic artery. It runs behind the first part of the duodenum and is usually implicated in a bleeding posterior duodenal ulcer.

The SMA is posterior to the splenic vein – it can be seen passing beneath the neck of the pancreas and under the splenic vein

• The SMA lies to the left of the superior mesenteric vein. • The SMA arises at the level of L1. The inferior mesenteric artery (IMA) arises at L3. Both are midline branches of the aorta, however whilst the SMA runs inferiorly and to the right, the IMA runs to the left. • The SMA passes anterior to the left renal vein, Compression of the left renal vein between the SMA and the aorta is known as nutcracker syndrome. s ‘massive’ upper GI bleeding in an alcoholic patient. This makes oesophageal varices the most likely cause of the bleeding. The superficial oesophageal veins drain the lower third of the oesophagus to the left gastric vein. Portal hypertension secondary to alcoholic liver cirrhosis causes reversal of flow at portacaval anastomoses resulting in dilated superficial oesophageal veins (cf. caput medusae, anorectal varices)

Page | 2

The hepatic artery (proper) is a branch of the common hepatic artery – it can be seen branching from it in the image below. It passes in the free edge of the lesser omentum (hepatoduodenal ligament) medial to the common bile duct and anterior to the portal vein. Clamping the free edge of the lesser omentum to achieve haemostasis in the liver is known as Pringle’s manoeuvre.

The coeliac trunk is a direct branch of the aorta, arising at the level of T12. It trifurcates into the left gastric, splenic, and common hepatic arteries. The common hepatic artery bifurcates into the gastroduodenal artery and the hepatic artery proper, the latter then bifurcating into left and right branches. The right hepatic artery gives off the cystic artery, which supplies the gallbladder.

The right renal artery usually arises slightly lower than the left. It passes posterior to the inferior vena cava (IVC), the right renal vein, the head of the pancreas and the duodenum – it can be seen emerging from behind the IVC in the image below. It divides before reaching the renal hilum and, as with the left renal artery, gives branches which supply the adrenal gland and the proximal ureter. It would not be safe to divide this vessel, although occasionally there can be accessory renal arteries present branching directly off of the aorta.

Page | 3

The gonadal vessels originate from the abdominal aorta at the level of the L2 vertebra and descend into the pelvis anterior to the inferior vena cava as well as the ureters (‘water under the bridge’). It is important to identify the ureter and gonadal vessels during mobilisation of the left and right colon.

The inferior vena (IVC) passes posterior to third part of the duodenum. After the IVC receives the hepatic veins, it passes through its aperture in the diaphragm at the level of T8. Each of the three holes has three structures passing through them:

1. Caval foramen – IVC, lymphatics, right phrenic nerve (left phrenic nerve pierces diaphragm separately) 2. Oesophageal hiatus – oesophagus, vagal trunks, left gastric vessels 3. Aortic hiatus – aorta, thoracic duct, azygous vein • The IVC forms the posterior boundary of the epiploic foramen (of Winslow). The anterior boundary is the portal vein, whilst the superior and inferior boundaries are the caudate lobe of liver and the first part of the duodenum, respectively

The uterine artery arises from the anterior division of the internal iliac artery and runs medially on the levator ani towards the uterine cervix. It crosses above and in front of the ureter, to which it supplies a small branch, about 2 cm from the cervix. Reaching the side of the uterus, it ascends in a tortuous manner between the two layers of the broad ligament to the junction of the Fallopian tube and uterus. It then runs laterally towards the hilum of the ovary and ends by anastomosing with the ovarian artery. It supplies branches to the uterine cervix, the vagina, the body of the uterus, and from its terminal portion branches are distributed to the Fallopian tube and the round ligament of the uterus.

Page | 4

The internal iliac artery is a branch of the common iliac artery – it is shown with its major branches in the image. It divides into two branches, the posterior division and the anterior division.

• The posterior division gives off the superior gluteal artery (shown in turquoise) which supplies gluteus medius and minimis, and the lateral sacral (green). • The anterior division gives off the inferior gluteal artery which supplies gluteus maximus. All the muscles of the buttock are therefore supplied by the internal iliac artery, so compromise of this vessel would lead to visible buttock wasting. The blood supply of the penis is mainly derived from the pudendal artery (a branch of the internal iliac artery) and so this would also suggest the internal iliac artery was affected

There is no coeliac vein. The venous drainage of the abdominal viscera is via the hepatic portal vein

Prostate metastasis; the veins of the internal vertebral venous (Batson’s) plexus are clinically significant because they are valveless and can serve as a route for metastases.

The portal vein runs in the free edge of the lesser omentum (hepatoduodenal ligament).

The femoral vein is posterior to the artery in the adductor canal. As it exits the canal, it comes to lie medial to the artery.

• The femoral artery gives off the descending genicular artery in the adductor canal. • The femoral artery lies anterior to the adductor longus. The profunda femoris is deep to adductor longus. Abdominal Viscera

Page | 5

The pancreatic neck is anterior to the origin of the portal vein. The hepatic portal vein is formed of the confluence of the splenic and superior mesenteric veins, just posterior to the pancreatic neck, as shown below. It supplies 75% of all blood flow to the liver. The inferior vena cava lies posterior to its origin

The anterior relations of the lesser sac are:

1. visceral peritoneum along the posterior stomach 2. lesser omentum 3. gastrocolic omentum.

The lesser sac is a potential peritoneal space within the peritoneal cavity, seen in the images below as the space behind the stomach – it is shown in both axial and sagittal views. It may be entered by passing a finger into the epiploic foramen, immediately behind the portal vein in the free edge of the lesser omentum. The transverse mesocolon and visceral/parietal peritoneum that covers the diaphragm, pancreas, left kidney and duodenum form the posterior wall. The gastrosplenic ligament forms its lateral border. The falciform ligament arises from the anterior abdominal wall and demarcates the left and the right lobe of the liver. Pringle’s manoeuvre involves clamping the (proper) hepatic artery and portal vein in the free edge of the lesser omentum, interrupting the arterial and portal venous inflow to the liver and is often used when uncontrollable bleeding is encountered following trauma in an emergency surgery setting.

Page | 6

The fascia of Gerota is superficial to the perinephric fat.

The common bile duct lies to the right of the hepatic artery. Joining the pancreatic duct at the ampulla of Vater, which opens into the second part of the duodenum. It crosses a groove between the head of the pancreas and the second part of the duodenum, posteriorly to the second part of the duodenum, in front of the right renal vein.

Which feature best differentiates the upper jejunum from the lower ileum?

• There is less mesenteric fat at the border of the jejunum compared to the ileum. The jejunum has a thicker wall, less mesenteric fat, more plicae circulares, a wider lumen, fewer Peyer’s patches and fewer arterial arcades than the ileum • The jejunum has a thicker intestinal wall compared with the ileum. The jejunum makes up the proximal two-fifths of the small intestine while the ileum makes up the distal three-fifths. There is no macroscopic demarcation between the two, but macroscopic features help differentiate the two. The jejunum has a wider lumen, thicker wall and valvulae conniventes. The jejunal mesenteric vessels form one or two arcades with long and infrequent branches (vasa recta) stretching to the intestinal wall, in comparison with the ileum which has shorter vasa recta • Valvulae conniventes are mucosal folds of the small intestine which start at the second part of the duodenum. They are present in the jejunum, where they are large and thick, decrease in size in the distal ileum

Valvulae conniventes, also known as plicae circulares, are thin circular mucosal folds in the small bowel. These structures give the small bowel the distinct radiological appearances of lines appearing to pass across the full width of the lumen.

Page | 7

The transverse mesocolon is attached to the head, neck and body of the pancreas. The head of the pancreas overlies the right renal vessels (but not the right kidney itself). The transpyloric plane (L1) transects the pancreas obliquely, passing through the midpoint of the neck, with most of the head below the plane, and most of the body and tail above. The uncinate process lies posterior to the superior mesenteric vessels, and the inferior mesenteric vein passes behind the body of the pancreas, where it joins the splenic vein. Entering the lesser sac between the transverse mesocolon and body of pancreas is a common method for laparoscopic mobilisation of the splenic flexure.

Occasionally the spleen may be absent and is replaced with multiple splenunculi. The spleen lies in the concavity of the left hemidiaphragm with its long axis lying along the tenth rib, extending between ribs nine and eleven. It is fully invested in peritoneum, making it a peritoneal structure. The arterial supply is the splenic artery, which reaches the spleen via the lienorenal ligament. Venous drainage is via the splenic vein into the portal vein.

Gerota’s fascia, or the renal fascia, surrounds the kidney and the adrenal gland. It is found in the retroperitoneum, posterior to the colon and is labelled in red in the image below.

The third part of the duodenum lies in the subcostal plane (L3) and follows the inferior margin of the pancreatic head. The third part of the duodenum lies posterior to the superior mesenteric vessels

• The right psoas muscle is posterior to the third part of the duodenum. • The right renal artery is posterior to the second part of the duodenum.

The left colic flexure, also called the splenic flexure, is the point where the colon takes a sharp downward turn. This flexure is the point where the transverse colon ends and the

Page | 8 begins. It is located immediately inferior to the spleen, so an enlarged spleen must move medially to avoid this colic flexure.

The psoas major muscle is one of the posterior abdominal wall muscles that lies posterior to the kidneys. The psoas major lies immediately posterior to the kidneys bilaterally and arises from the transverse processes, lateral aspects of vertebral bodies and intervertebral discs of T12 to L5. Internal oblique and transverses abdominis form part of the anterior abdominal wall, while the erector spinae group support the trunk.

• The erector spinae muscles are a group of muscles of the back. It is made up of three muscles, from medial to lateral, the spinalis, longissimus and iliocostalis. These muscles lie posteromedial to the kidneys, not immediately posterior

These five structures form the H shape that divides the visceral surface of the liver. The gall bladder can be seen to the right of the image, the IVC superiorly and the ligamentum teres towards the lower left.

The common bile duct contains no muscle

• The hepatic artery runs to the left of the common bile duct.

A simple nephrectomy is indicated in cases of a non-functioning kidney. The adrenal gland lies over the medial border and superior to the hilum.

• The best approach for a ruptured kidney is through the anterior abdominal wall by a midline incision.

The second, third and fourth (ascending) parts of the duodenum are all retroperitoneal structures. Only the first part (superior, D1) is intraperitoneal.

• The descending part of the duodenum lies lateral to the head of the pancreas.

The portal vein is the most posterior structure in the porta hepatis. Within the porta hepatis, the common hepatic duct and proper hepatic artery typically are located anterior to the portal vein, with the common hepatic duct to the right and the proper hepatic artery to the left.

The portal vein provides 75% of blood supply to the liver, but only 50% of its oxygen supply.

Page | 9

Normally, (in non-variant anatomy cases) the portal vein divides into two main branches: left portal vein right portal vein.

1. The left portal vein supplies segments II, III and IV of the liver. 2. The right portal vein goes on to divide into the anterior and posterior portal veins supplying segments V, VIII and VI, VII respectively

The kidney is directly related to the psoas muscle at its medial posterior surface.

• The is found on the right of the aorta in front of L1 and L2. It is at a similar level to the kidneys and lies between them without direct contact.

The right suprarenal gland is pyramidal in shape while the left suprarenal gland is semi-lunar or crescentic shape.

The first part of the duodenum is the inferior border of the epiploic foremen. The epiploic foramen of Winslow lies behind the free edge of the lesser omentum. The posterior wall of the epiploic foramen is formed by the peritoneum of the posterior abdominal wall, which passes as a smooth layer from the hepatorenal pouch to the duodenum. Behind this peritoneum lies the inferior vena cava. The superior border of the epiploic foramen is formed by the inferior surface of the liver (caudate lobe). The first part of the duodenum forms part of the inferior border. The portal vein lies anteriorly in the free edge of the opening.

The testis; The nerve supply is via the lesser splanchnic nerve (Sympathetic nerve supply) and the coeliac ganglion with postganglionic fibres accompanying the testicular arteries. The testis is usually descended by birth having developed from the gonadal ridge on the posterior abdominal wall. Its thick, whitish covering is the tunica albuginea.

• Lymphatic drainage follows the arterial supply to the para-aortic nodes. • The spermatic cord contains the pampiniform plexus – a single testicular vein is formed more distally over psoas major. • The nerve supply is via the lesser splanchnic nerve and the coeliac ganglion with postganglionic fibres accompanying the testicular arteries

The small bowel mesentery contains the superior mesenteric artery supplies the mid-gut, which includes the large bowel as far as the distal transverse colon. The veins in the mesentery are all tributaries of the portal system. The root of the small bowel mesentery extends from a point to the left of the L2 vertebra, at the duodenojejunal junction, down to the right sacroiliac joint. The nerves are postganglionic sympathetic and preganglionic parasympathetic fibres (vagus nerve) from the superior mesenteric plexus. The superior mesenteric artery supplies the mid-gut, which includes the large bowel as far as the distal transverse colon. The transverse mesocolon lies anterior to the small bowel mesentery.

Annular pancreas can either partially or completely encompass the second part of the duodenum resulting in stenosis and proximal dilatation of the proximal small bowel. Clinically, this may result in abdominal pain and vomiting. Annular pancreas is an embryological anomaly that can result in duodenal obstruction. It can be complete or incomplete. It affects both children and adults causing post-prandial satiety, abdominal pain and vomiting and is diagnosed using a CT scan or MRI.

Page | 10

• An annular pancreas develops due to failure of the ventral bud to rotate with the duodenum, resulting in its encasement. • This process typically happens in the 7th week of gestation. • Down’s syndrome, pancreatitis and pancreatic cancer are associated with having an annular pancreas. Li Fraumeni’s syndrome, conversely, has no association with an annular pancreas. • A third of cases are an incidental finding and patients are asymptomatic. • In children, it commonly presents as duodenal obstruction, whereas adults typically present with signs and symptoms of pancreatitis. Abdominal wall and cavity Nine structures lie within the spermatic cord:

• three arteries (testicular, cremasteric, artery to the vas deferens); • three nerves (genital branch of the genitofemoral nerve, sympathetic fibres, ilioinguinal nerve – this is outside the cord but travels with it) and • three other structures (pampiniform plexus, vas deferens, lymphatics). • There are three covering fascial layers – external and internal spermatic and cremasteric fascia

The portal vein receives tributaries including the right and left gastric veins and para-umbilical veins and also receives cystic veins from the gall-bladder.

• The portal vein is formed at the L1 level.

The linea semilunaris is the lateral border of the rectus sheath and forms the medial border of Hesselbach’s triangle.

• The ilioinguinal nerve travels through the superficial ring but does not form a border of Hesslebach’s triangle. • The inferior epigastric vessels form the supero-lateral border of Hesselbach’s triangle. • The inguinal ligament is the infero-lateral border of Hesselbach’s triangle.

Amyand’s hernia is an inguinal hernia containing an inflamed appendix.

The hepatorenal space is a potential space and lies between the right lobe of the liver and right kidney. It forms the most dependent part of the peritoneal cavity, apart from the pelvis, in the supine abdomen and communicates freely with the lesser sac, the right paracolic gutter and right subphrenic space. Intraperitoneal fluid or pus can easily accumulate in the hepatorenal pouch.

Transversalis fascia forms the lateral aspect of the posterior wall throughout the inguinal canal and is joined by the conjoint tendon medially.

• The transverus abdominus muscle helps form the roof of the inguinal canal. • The aponeurosis of the external oblique helps form the anterior wall and is assisted laterally by the internal oblique.

Page | 11

The foregut begins at the mouth and ends just inferior to the major duodenal papilla the point at which the common bile duct and pancreatic duct empty into the duodenum, where it gives way to the midgut

The ureterovesical junction, This is the point at which the ureters enter the wall of the bladder, and is a site of ureteral constriction at which renal stones may lodge. It is the most common site of impaction as it has the smallest diameter. Generally for a stone to impact it must have a diameter of 2 mm or greater.

Scarpa’s fascia is contiguous with Colles’ fascia in the perineum.

The inferior epigastric vessels lie on the inner surface of the transversus abdominis and are covered by parietal peritoneum. Remember: the peritoneum lies over the inferior epigastric vessels to make the lateral umbilical fold.

The inguinal ligament is also known as Poupart’s ligament, named after Francois Poupart who gave it relevance with regards to inguinal hernia repairs.

• The duct of Santorini is the accessory pancreatic duct. • Alcock’s canal is a fascial tunnel on the lateral wall of the ischiorectal fossa, which conveys the pudendal nerve and vessels. It is not related to the femoral canal. • The pectineal ligament is also known as Cooper’s ligament, and should not be confused with the suspensory ligaments of the breast, also known as the ligaments of Cooper.

The visible landmark for the superficial inguinal ring are the intercrural fibres that run at right angles across the external oblique aponeurosis

Pyramidalis muscle is a small, triangular, vertical muscle that lies anterior to the rectus abdominis and attaches to the linea alba having originated at the front of the pubis and pubic symphysis. It is innervated by the ventral portion of T12 and supplied by the inferior and superior epigastric arteries. Its action is to tense the linea alba. It is naturally in up to 12% of the population CNS, Brain The nasolacrimal duct is a membranous canal, about 18 mm in length, which extends from the lower part of the lacrimal sac to the inferior meatus of the nose. It ends as the plica lacrimalis (Hasner’s fold), formed by a fold of the mucous membrane. It is contained in an osseous canal, formed by the maxilla, the lacrimal bone and the inferior nasal concha. It is narrower in the middle than at either end and is

Page | 12 directed downward, backward and a little lateralward. The mucous lining of the lacrimal sac and nasolacrimal duct is covered with columnar epithelium, which in places is ciliated.

• The anterior and middle ethmoidal air cells, and maxillary sinus drain into the middle meatus. • The frontal sinus drains via the frontonasal duct then infundibulum of the hiatus semilunaris, then into the middle meatus

The buccal branch of the facial nerve supplies the buccinator and orbicularis oris muscle. Injury to the facial nerve can occur in parotid surgery as well as middle ear and temporal bone surgery, as this represents the course of the facial nerve. Facial nerve monitoring is used to assess facial nerve proximity and to minimise the risk of intra-operative injury.

• The chorda tympani carries taste sensation from the anterior two-thirds of the tongue. It separates from the facial nerve within the temporal bone and is therefore not part of the facial nerve at the level of the parotid gland. The chorda tympani, after separating, form the facial nerve, then runs back into the middle ear to run on the posterior wall. It travels between the pars flaccida and over the handle of the malleus. It passes through the petrous temporal bone before leaving the skull via the petrotympanic fissure. The chorda tympani then enters into the infratemporal fossa to join the lingual nerve. • The muscles of mastication are supplied by the mandibular division of the trigeminal nerve.

The ophthalmic artery is a direct branch of the internal carotid artery. The artery lies inferolateral to the optic nerve. It gives off a number of branches, the first and one of the smallest being the central retinal artery.

• The anterior cerebral artery is formed from the bifurcation of the internal carotid artery into the anterior and middle cerebral arteries. It passes around the genu of the corpus callosum and mainly supplies the medial aspects of the frontal lobe.

The Eustachian tube, also known as the pharyngotympanic tube or auditory tube connects the middle ear to the nasopharynx. The posterolateral third of the tube is bony and the remainder cartilaginous. The bony part of the Eustachian tube perforates the petrous temporal bone. The Eustachian tube in a child is shorter and more horizontal. The opening of the auditory tube lies above the soft palate adjacent to the tubal tonsil.

• The cartilaginous part of the Eustachian tube is intimately related to the greater wing of the sphenoid, although the tube does not penetrate it.

The superior cerebral veins, 8 to 12 in number, drain the superior, lateral and medial surfaces of the cerebral hemispheres and are mainly lodged in the sulci between the gyri, but some run across the gyri. They open into the superior sagittal sinus, which can be seen below running in the superior aspect of the falx cerebri.

• The superior and inferior ophthalmic veins, sphenoparietal sinus, superficial middle cerebral veins drain into the cavernous sinus but not the superior cerebral veins • The inferior petrosal sinus drains the cavernous sinus. It meets the sigmoid sinus at the level of the jugular foramen to form the internal jugular vein. • The superior petrosal sinus connects the cavernous sinus to the transverse sinus.

Page | 13

The medial wall of the parotid gland capsule is separated from the carotid sheath by the styloid process and associated muscles (stylopharyngeus, stylohyoid, styloglossus).

The facial nerve runs medial to the middle ear cavity and superior to the promontory and oval window.

• The internal carotid artery runs anterior and inferior to the middle ear cavity. • The internal jugular vein is located inferior to the middle ear cavity. Thin bone separates the middle ear from the bulb of the internal jugular vein.

Above, the spinal arachnoid mater is continuous with the cranial arachnoid. Below, it widens out and invests the cauda equina and the nerves proceeding from it. The spinal part of the arachnoid is a thin, delicate, tubular membrane loosely investing the spinal cord.

The facial muscles are subcutaneous (just under the skin, in the same plane as the platysma) muscles that control facial expression. They generally originate on bone and insert on the skin of the face. The facial muscles are innervated by cranial nerve VII, also known as the facial nerve. The facial muscles are derived from the second pharyngeal arch.

• The long buccal nerve innervates the buccal mucosa and the lateral pterygoid. It is a minor branch of the mandibular division of the trigeminal nerve. • The digastric muscle’s action is to aid swallowing and depress the mandible. It is interesting to note that the anterior belly is supplied by the mandibular division of the trigeminal while the posterior belly is innervated by the facial nerve. • Both the zygomaticofacial and zygomaticotemporal nerves are both entirely sensory.

The lymphoid tissue at the base of the tongue is also known as the lingual tonsil. It forms a part of Waldeyer’s ring – a ring of lymphoid tissue within the pharynx. These lymphoid structures may become enlarged from benign causes (eg reactive secondary to infection) or malignancy (primary or secondary).

• The circumvallate papillae are large and flat topped, and found anteriorly to the terminal sulcus of the tongue in two rows forming a V-shape. • The filiform papillae represent normal structures situated on the anterior two-thirds of the tongue. • The fungiform papillae represent normal structures and are interspersed among the filiform papillae, usually situated on the tip and sides of the tongue.

Page | 14

The facial nerve supplies the muscles of facial expression including innervating buccinator. This muscle is involved in emptying the buccal sulcus during mastication.

Both the medulla of the brainstem and the spinal portion of the spinal accessory nerve pass through the foramen magnum. The anterior and posterior spinal arteries, and vertebral arteries also pass through the foramen magnum.

The superior oblique muscle depresses, internally rotates and abducts the eye.

• The inferior oblique elevates, externally rotates and abducts the eye.

The occipitofrontalis is supplied by the facial nerve. There is no C1 dermatome. There is however a C1 nerve root.

• The scalp drains to the lymph nodes of the neck. The submandibular lymph and deep cervical nodes drain the anterior scalp while the retro-auricular and occipital nodes drain the posterior scalp. • The blood supply to the scalp is from branches of both the internal and external carotid arteries. Two such tributaries from the internal carotid are the supraorbital and supratrochlear arteries.

The facial artery passes over the posterior surface of the submandibular gland in a groove.

• The facial vein is superficial to the submandibular gland. • The lingual nerve lies laterally to the submandibular duct and medial to the gland. It is at risk in submandibular gland excision along with the hypoglossal nerve.

Vestibular schwannoma also known as an acoustic neuroma. These are benign primary intracranial tumours arising from the myelin forming cells of the vestibulocochlear nerve (CN VIII). These can cause symptoms associated with hearing and vertigo. As they grow in size they can compress the facial nerve, although this tends to be late in the disease process, or often due to damage of the nerve from surgical excision. More commonly, the trigeminal nerve is affected OVALE O Otic ganglion (inferior)

V V3 cranial nerve (mandibular division of trigeminal nerve)

A Accessory meningeal artery

L Lesser petrosal nerve

E Emissary veins

The inner cheek mucosa is supplied by the buccal branch of the anterior division of the mandibular branch of the trigeminal nerve – the only sensory branch of the anterior division. This nerve is not to be confused with the buccal branch of the facial nerve, which supplies muscles of facial expression.

Page | 15

• The anterior division of the mandibular division of the trigeminal nerve contains branches to lateral pterygoid, masseter and temporalis muscles and buccal skin. Injury to this nerve produces deviation of the jaw towards the side of the lesion on protrusion due to the unopposed action of the contralateral lateral pterygoid. • The muscles of swallowing are innervated by the glossopharyngeal nerve • Sensation to the lower teeth and gums is via the inferior alveolar nerve, which comes from the posterior division of the mandibular division of the trigeminal nerve.

The olfactory foramina are located in the anterior cranial fossa. They are located within the cribriform plate, a part of the ethmoid bone. The fibres of the olfactory nerve originate from the superior surface of the superior concha, medial aspect of the superior nasal septum, and the inferior surface of the cribriform plate. These fibres pass through the olfactory foramina and synapse in the olfactory bulb on the superior surface of the cribriform plate.

The ophthalmic artery arises from the internal carotid, just as that vessel is emerging from the cavernous sinus, on the medial side of the anterior clinoid process and enters the orbital cavity through the optic foramen (canal), below and lateral to the optic nerve. It then passes over the nerve to reach the medial wall of the orbit and thence horizontally forward, beneath the lower border of the superior oblique and divides it into two terminal branches, the frontal and dorsal nasal. As the artery crosses the optic nerve it is accompanied by the nasociliary nerve and is separated from the frontal nerve by the rectus superior and levator palpebrae superioris.

The inferior orbital fissure transits the inferior ophthalmic vein, zygomatic branches of the maxillary division of trigeminal nerve, and ascending branches of the pterygopalatine ganglion

The ophthalmic division of the trigeminal nerve divides into the three main branches: the frontal nerve, the lacrimal nerve and the nasociliary nerve. The supratrochlear is a direct branch of the frontal nerve. and supplies the superior-medial orbital skin, superior to the medial canthus. It is medial to the supra- orbital nerve. The nerve passes above the pulley of the superior oblique and gives off a descending filament to join the infratrochlear branch of the nasociliary nerve.

The lingual nerve supplies the somatic sensation to the anterior two-thirds of the tongue. Therefore, loss of sensation to the tongue ipsilaterally to the side of nerve injury may result in ulceration (eg from accidentally biting the tongue). The lingual nerve is a branch of the mandibular division of the trigeminal nerve.

Page | 16

• Taste sensation from the anterior two-thirds of the tongue is derived from the chorda tympani branch of the facial nerve, which runs with the lingual nerve. • General sensation is from the lingual nerve, a branch of the mandibular division of the trigeminal nerve. • Posterior one-third – both taste and general sensation are supplied by the glossopharyngeal nerve (CN IX). • Base of the tongue – the internal laryngeal nerve, a branch of the vagus (CN X) supplies this.

The parotid duct opens on the middle third of a line between the intertragic notch of the auricle and the midpoint of the philtrum, opposite the second upper molar.

The middle meningeal artery divides from the maxillary branch of the external carotid, entering the cranium through the foramen spinosum. It is the largest artery that supplies the dura and also the calvaria. While the maxillary artery does give off branches in the pterygopalatine fossa region, the middle meningeal artery comes off more proximally, deep to the ramus of the mandible. It is particularly prone to damage caused by temporal bone fractures.

• The accessory meningeal artery enters the skull via the foramen ovale. • The middle meningeal artery is derived from the first part of the maxillary artery.

The angle of the mandible is supplied by the great auricular nerve (C2–C3).

The iris separates the anterior and posterior chambers. These regions, however, should not be confused with the anterior and posterior segments. The posterior segment consists of the vitreous humour, and structures posterior to this including the optic nerve and retina.

The tongue is retracted up and back by the styloglossus muscle, protruded by genioglossus and depressed by the hyoglossus. All these muscles are innervated by the hypoglossal nerve (CN XII).

Parasympathetic fibres from the Edinger–Westphal nucleus are carried along the oculomotor nerve and then its inferior division to synapse in the ciliary ganglion. These parasympathetic fibres then supply the ciliaris and sphincter pupillae muscles.

• The facial nerve carries multiple parasympathetic fibres. These travel on the greater petrosal nerve through the petrous temporal bone, then deep petrosal nerve. These particular fibres synapse in the pterygopalatine ganglion to subsequently supply the lacrimal, and nasal glands. Other parasympathetic fibres travel with the chorda tympani, synapse in the submandibular ganglion, and innervate the submandibular and sublingual glands. • The glossopharyngeal nerve carries parasympathetic fibres to the parotid gland via the otic ganglion

The sphenopalatine artery supplies the posterior nasal mucosa. It supplies blood to the lateral nasal wall and nasal septum. It is a branch of the third part of the internal maxillary artery, passes through the sphenopalatine foramen into the cavity of the nose, at the back part of the superior meatus.

• Here it gives off its posterior lateral nasal branches, which spread forward over the conchae and meatuses, anastomose with the ethmoidal arteries and the nasal branches of the descending palatine and assist in supplying the frontal, maxillary, ethmoidal and sphenoidal sinuses.

Page | 17

• Crossing the undersurface of the sphenoid, the sphenopalatine artery ends on the nasal septum as the posterior septal branches; these anastomose with the ethmoidal arteries and the septal branch of the superior labial. • In severe posterior nasal bleeds that are uncontrolled with non-operative management the artery can be ligated or can undergo haemostasis with diathermy. • The sphenopalatine artery is also one of the vessels that contributes to Kiesselbach’s plexus (Little’s area – shown below in the image on the anterior nasal septum), in addition to anterior ethmoidal/posterior ethmoidal arteries, superior labial arteries, and greater palatine artery

Sympathetic supply to the lacrimal gland is via the nerve of the pterygoid canal, which runs with the lacrimal nerve, one of the three major branches of the ophthalmic division of the trigeminal nerve. The lacrimal nerve also carries parasympathetic fibres from the zygomatic branch of the maxillary nerve from pterygopalatine ganglion.

The glossopharyngeal nerve innervates the tonsillar fossa. It also gives a sensory branch to the ear (Jacobsen’s nerve) from which referred pain can originate.

The afferent limb (sensation) of the corneal reflex is carried on the ophthalmic division of the trigeminal nerve, and the efferent limb (the motor reflex) is via the zygomatic branches of the facial nerve to orbicularis oculi, which results in blinking. Injury at the level of the parotid would lead to lower motor neurone paralysis of the ipsilateral orbicularis oculi and the corneal reflex would therefore be lost.

The parasympathetic innervation to the salivary glands of the oral cavity do not pass through the parotid gland. The parasympathetic supply to the parotid comes from the lesser petrosal nerve, which branches from the geniculate ganglion in the middle ear and synapses within the otic ganglion, and is carried on the auriculotemporal nerve to the parotid gland.

The parasympathetic nerve supply to the sublingual glands and submandibular gland are carried via the chorda tympani branch of the facial nerve, which joins the lingual nerve to supply the glands. Damage to the facial nerve after it exits from the skull at the stylomastoid foramen would not lead to a dry mouth as the parasympathetic supply has already been given off in the middle ear

Page | 18

The capsule of the TMJ lies anteriorly to the squamo-tympanic fissure. The temporomandibular joint is a synovial joint situated between the condyle of the mandible below and the mandibular fossa of the temporal bone above. Although it a synovial joint, it is lined by fibrous cartilage (rather than hyaline cartilage typical of synovial joints). The joint is surrounded by a capsule that is attached beyond the limits of the articular surfaces, strengthened medially and laterally by collateral ligaments.

Posteriorly, the capsule is attached to the anterior edge of the squamo-tympanic fissure.

Medially, it runs along the suture between the temporal and sphenoid bones and attaches anteriorly to the anterior end of the articular eminence.

Laterally, the capsule is attached to the articular tubercle, which forms the lateral limit of the articular eminence and the prominent ridge of the bone forming the lateral lip of the glenoid cavity. The joint is stable anteriorly but is laxed posteriorly as it is attached well below the articular surface to the neck of the condyle.

• The TMJ contains two synovial cavities – the superior and inferior synovial cavity separated by an articular disc. • The upper compartment of the TMJ is responsible for protrusion and retraction of the jaw, as well as side-to-side movement. The lower compartment is responsible for elevation and depression of the mandible.

The lingual nerve (shown in blue) is one of three major branches of the posterior division of the mandibular division of the trigeminal nerve. The other two major nerves are the auriculotemporal nerve and the inferior alveolar nerve.

Oral cavity malignancies are usually squamous-cell carcinomas and 35% of these occur on the tongue. They tend to be located on the lateral edge of the tongue. These types of cancer are associated with tobacco smoking, low fruit/vegetable diet, and alcohol consumption.

The cartilaginous part gives attachment to the tensor veli palatini muscle. This muscle is innervated by the nerve to medial pterygoid, which is a branch of the mandibular branch of the trigeminal nerve.

• The palatine tonsils are situated in the oropharynx while the Eustachian tube opens into the nasopharynx. Its opening is surrounded by the tubal tonsil. It can be obstructed by enlarged adenoids, which can increase the risk of glue ear (chronic otitis media with effusion).

Page | 19

• The Eustachian tube opens on swallowing under the action of the salpingopharyngeus and tensor palati muscles

The submandibular duct (Wharton’s duct) is about 5 cm long and its wall is much thinner than that of the parotid duct. It begins from numerous branches from the deep surface of the submandibular gland and runs forward between the mylohyoid and the hyoglossus and genioglossus, then between the sublingual gland and the genioglossus and opens by a narrow orifice on the summit of a small papilla, at the side of the frenulum linguae, near the midline in the anterior aspect of the floor of the mouth. On the hyoglossus it lies between the lingual and hypoglossal nerves, but at the anterior border of the muscle it is crossed laterally by the lingual nerve. The terminal branches of the lingual nerve ascend on its medial side.

The facial artery arises from the external carotid artery, on the anteromedial surface. It gives off an ascending palatine artery and tonsillar artery. The facial artery passes deep to the posterior belly of digastric and then runs on the posterior surface of the submandibular gland.

The submandibular gland lies within the digastric triangle (part of the anterior triangle). This triangle is bound inferiorly by the two bellies of the digastric muscle and superiorly lies bound by the lower border of the mandible. Therefore the submandibular gland lies superior to the digastric muscle

The lateral pterygoid muscle is important for active opening of the mouth – drawing the condyle and disc forwards from the mandibular fossa.

• The temporalis muscle is a muscle of mastication whose main role is to close the mouth and elevate the jaw. Its posterior fibres have a role in retracting the jaw as well. • The buccinators has a role of assisting with chewing, although it is not technically regarded as a muscle of mastication. Specifically, it has a role in holding the cheek to the teeth during chewing, and is supplied by the facial nerve. • The masseter is a muscle of mastication, although its main role is closure of the mouth • The action of the medial pterygoid is to elevate the mandible, protract it, and laterally displace the mandible. It is a muscle of mastication and supplied by the mandibular branch of the trigeminal (nerve to medial pterygoid).

The middle meatus contains the bulla ethmoidalis (or the ethmoid bulla). The ethmoid bulla is caused by bulging lamella of the middle ethmoidal cells and may be a pneumatised cell – and thereby forming a large prominent anterior air cell – or just a bony prominence – termed torus ethmoidalis.

• The sphenoidal air sinus drains into the sphenoethmoidal recess, which is posterior and superior to the superior concha • The posterior ethmoidal air sinus drains into the superior meatus. • The frontal sinus and the anterior ethmoid cells (sinus) drains into the infundibulum of the middle meatus. • The maxillary sinus and middle ethmoidal cells open into the middle meatus

The external palatine vein drains the tonsil, into the pharyngeal plexus which subsequently drains into the internal jugular vein. The venous plexus around the tonsillar bed is a common source of bleeding after tonsillectomy.

Page | 20

• The palatine fossa lies in the oropharynx, not the oral cavity. The oropharynx is bound anteriorly by the base of the tongue and palatoglossal folds. The superior boundary is the soft palate. The inferior boundary is the superior surface of the epiglottis. The posterior boundary is the posterior pharyngeal wall. • The palatine tonsillar bed contains the glossopharyngeal nerve. This nerve is responsible for the referred otalgia when the tonsils become inflamed or when patients have undergone tonsillectomy. The glossopharyngeal nerve has a tympanic nerve called Jacobson’s nerve, which is responsible for this phenomenon. • The floor the palatine tonsillar bed is bound by the superior constrictor muscle. • The tonsil is supplied by the tonsillar branch of the facial artery, which runs alongside the styloglossus, after which it penetrates the superior pharyngeal constrictor to enter the palatine fossa.

The only cranial nerve that does not arise from the brain itself.

The accessory nerve is unique in that its roots arise from motor neurons in the upper five segments of the cervical spinal cord. It leaves through the jugular foramen to innervate the trapezius and sternocleidomastoid muscles.

The falx cerebri is a crescent-shaped fold of dura mater that descends vertically. The tentorium cerebelli is an extension of the dura mater separating the occipital lobes from the cerebellum.

The middle cerebral artery (MCA) is the largest cerebral artery and is the most commonly affected vessel by cerebrovascular accident. The artery supplies the majority of the outer convex brain surface such as the lateral aspect of the parietal lobe, the basal ganglia and the posterior and anterior internal capsules. Infarctions that occur in the MCA distribution result in contralateral paralysis and sensory loss of the face and upper limb. Aphasia occurs if the infarction is in the dominant hemisphere, and hemineglect if the non-dominant side is affected.

• Posterior cerebral artery; The posteroventral nucleus of the thalamus and inferior temporal gyrus are supplied by the posterior cerebral artery. Infarctions involving the posterior cerebral artery result in contralateral hemianopsia with macular sparing. • Anterior cerebral artery; The anterior one-third of the corpus callosum, including the superior frontal gyrus, is supplied by the anterior cerebral arteries. Infarctions involving the anterior cerebral artery result in contralateral paralysis and sensory loss of the lower limb. • Posterior inferior cerebellar artery; PICA occlusion may cause infarction of the posterior inferior cerebellum, inferior cerebellar vermis and lateral medulla. Vertigo, nausea and truncal ataxia are the most common presenting features. • Acute occlusion of the basilar artery may cause brainstem or thalamic ischaemia or infarction. Patients with acute occlusion of the basilar artery will present with sudden and dramatic neurological impairment. • Both the posterior and middle cerebral arteries supply the temporal lobe. The MCA supplies the superior aspect, whereas the PCA supplies the inferior surface. Similar to many regions of the brain, there is a watershed area in which two major origins of blood supply overlap.

Page | 21

The ependymal is a layer of ciliated epithelial cells that lines the ventricles and central canal of the spinal cord. Ependymal cells are one of the glial cells of the central nervous system. The others are astrocytes, phagocytic microglia and oligodendrocytes.

• CSF total volume is around 150 ml, with around one-sixth of this found in the ventricles. The rest is found in the subarachnoid spaces of the cranium and spine. • CSF flows from lateral ventricles to the third ventricles via the foramen of Monro, and then to the fourth ventricle via the cerebral aqueduct of Sylvius. • CSF is produced by the choroid plexuses of the lateral, third and fourth ventricles. A small amount is produced by the ependymal cells. CSF is absorbed by the arachnoid granulations, which act as valves. Arachnoid granulations allow CSF to flow from the subarachnoid space to the venous system, but do not themselves produce CSF.

The most common stroke is from infarction in the territory of the middle cerebral artery (MCA), which does not usually cause a coma. The MCA supplies the lateral surface of the cortex and temporal pole. The cortical branches of the MCA supply the lateral surface of the hemisphere except for

(1) the frontal pole and a strip along the superomedial border of the frontal and parietal lobes supplied by the anterior cerebral artery, and (2) the lower temporal and occipital pole convolutions supplied by the posterior cerebral artery.

Occlusion of its inferior division is commonly associated with Wernicke’s aphasia, while occlusion of the superior division causes Broca’s aphasia.

Strokes that involve the entire MCA distribution in the left hemisphere cause combined dysfunction of the two areas. Speech output is nonfluent, and comprehension of spoken language is severely impaired, as well as naming, repetition, reading and writing. Most patients are initially mute or say a few words. Related signs include right hemiplegia, hemisensory loss and homonymous hemianopia.

Strokes involving the vertebrobasilar circulation lead to a focal brainstem syndrome.

The diaphragma sellae is a small dural structure that covers hypophysial fossa in the sella turcica of the sphenoid bone. An opening in the middle of the diaphragma sellae allows the passage of the infundibulum. This structure connects the hypothalamus to the pituitary gland. The dural partitions of the brain are projections of dura mater that subdivide the cranial cavity: the falx cerebri, the tentorium cerebelli, the falx cerebelli and the diaphragma sellae.

• The falx cerebri is a crescent-shaped downward projection, a double fold of dura mater that runs between the cerebral hemispheres. Anteriorly, it attaches to the ethmoid bone at the crista galli and the frontal crest of the frontal bone. It attaches to the tentorium cerebelli inferior- posteriorly. • The tentorium cerebelli is a double fold of dura mater that forms the roof the posterior fossa and separates the supratentorial and infratentorial compartments. • The falx cerebelli is a variably present and projection of dura mater in the posterior cranial fossa. It forms attachments to the occipital bone and to the tentorium cerebelli superiorly. It is found in the midline between the cerebellar hemispheres.

Page | 22

The third ventricle is bounded laterally by the thalamus. It communicates superolaterally with the lateral ventricles. The ventricular system of the brain comprises four communicating cavities which are responsible for the production and transport of cerebrospinal fluid (CSF). CSF travels from the lateral (first and second) ventricles to the third ventricle via the foramen of Monro, and from there to the fourth ventricle via the cerebral aqueduct (of Sylvius). The system is continuous with the central spinal canal to bathe the spinal cord. Clinically this is relevant in hydrocephalus, which occurs when there is an abnormal accumulation of CSF in the ventricles of the brain. The most common site of obstruction is at the cerebral aqueduct.

• The first and second ventricles of the brain are together known as the lateral ventricles due to their position in each of the cerebral hemispheres. They are superior to the thalamus.

The hypoglossal nerve exits the skull through its own foramen (the hypoglossal foramen).

Apical ligament of the dens; This strong fibrous band connects the second cervical vertebra to the skull in the anterior portion of the foramen magnum.

An oculomotor nerve palsy is remembered as ‘down and out’ – as the superior oblique muscle acts unopposed leaving the eyeball infraducted and abducted. Ptosis may be evident.

The inferior oblique muscle is supplied by the oculomotor nerve (the third cranial nerve). It inserts into the temporal lower quadrant of the eyeball and moves the eye superiorly and laterally.

These are the three main components of the brainstem – the midbrain (mesencephalon), the pons (metencephalon) and the medulla oblongata (myelencephalon).

• The first two cranial nerves arise from the cerebrum, III – X and XII originate in the brainstem. Cranial nerve XI has a component arising from the brainstem (the cranial accessory) and component arising from the upper segment of the spinal cord (the spinal accessory). • The pons is the largest part of the brainstem. The cerebellopontine angle is significant since intracranial growths can be found here. Early symptoms come from the compression of cranial nerve 8 – the vestibulocochlear nerve

The diencephalon is part of the forebrain. The forebrain comprises the cerebrum (telencephalon) and the diencephalon. The diencephalon consists of the thalamus and hypothalamus. The hypothalamus regulates growth via the production of growth hormonereleasing hormone, sexual maturation from gonadotrophin-releasing hormone and suppresses appetite with corticotropin-releasing hormone. The hypothalamus is affected in Prader–Willi syndrome, which is a chromosomal disorder usually due to

Page | 23 partial deletion of paternal chromosome 15. Symptoms are wide ranging and are related to the hormones secreted by the hypothalamus

From the pons the facial nerve crosses the posterior fossa of the skull before it passes laterally upwards into the petrous temporal bone to exit through the skull the internal acoustic meatus.

The cerebellum has no significant influence on the breathing rate or depth.

• cerebral cortex can control the breathing rate and depth using conscious control. Although not normally required, • The medulla, where the respiratory group of neurones lies, includes the dorsal respiratory group and the ventral respiratory group. • The pons includes the pneumotaxic and apneustic centres, the latter of which is under constant inhibition from the medulla. • The hypothalamus is one of the many influences on the respiratory centres that control breathing

The mamillary bodies lie on the inferior surface of the hypothalamus, they are small round structures consisting of two groups of nuclei. They have a role in memory and form part of the limbic system

• The posterior portion of the hypothalamus contains the nerve endings of neurosecretory cells, which run down through the infundibular stalk to the pituitary gland also known as the median eminence. • The hypothalamus makes up the floor of the third ventricle.

Acoustic neuromas arise from Schwann cells

• Acoustic neuroma is also known as a vestibular schwannoma, it is a benign, slow growing tumour that arises from the Schwann cells of the vestibulocochlear (8 ) cranial nerve. • Up to 90% of patients present with a gradual progressive unilateral deafness, often associated with tinnitus. • Recurring and severe headaches are uncommon and occur only with very large tumours. • They make up around 8% of all intracranial tumours.

The lambda is the point in the skull where the lambdoid and sagittal sutures meet. It is palpable as a depression between the occipital and two parietal bones, but in the adult is not the thinnest part of the skull. In the newborn this the site of the posterior fontanelle.

The bregma of the skull is the bony landmark where coronal and sagittal sutures meet. It is located at the top of the skull, and in the neonate is the anterior fontanelle. It closes at around 18 months. In the adult, however, it is not the thinnest par

The foramina of Magendie (midline) and Lushka (lateral) in the roof of the fourth ventricle communicate directly into the subarachnoid space.

Wernicke’s area is responsible for language comprehension. Damage to this area causes receptive dysphasia, in which speech will be fluent but lack meaning. Patients with this condition are generally unaware that their words lack meaning. Wernicke’s aphasia is mostly caused by stroke.

Page | 24

Broca’s area is responsible for language production but it is found in the frontal lobe of the dominant hemisphere. This is usually, but not always, the left hemisphere

Craniopharingioma may initially present as a lower bitemporal quadrantopia, the lesion is usually found superior to the optic chiasm..

The (part of the hindgut) has its parasympathetic innervation originating from the pelvic splanchnic nerves.

Sympathetic responses are related to the fight-or-flight response and the pupil will dilate. Sympathetic fibres originate from the thoracic cord (T1) and enter the sympathetic chain. Post-ganglionic fibres from the superior cervical ganglion supply the eye, removal of which results in unbalanced parasympathetic innervation. the metastatic disease has damaged the sympathetic innervation of the pupil on the side ipsilateral to T1. CNIII provides parasympathetic innervation to the pupil.

Unlike pre-ganglionic neurotransmission, which uses acetylcholine (ACh), and sweat glands, which also use ACh in post-ganglionic transmission, the most common post-ganglionic neurotransmitter in the sympathetic system is noradrenaline. all four are cranial parasympathetic ganglia where pre- and post-ganglionic parasympathetic fibres synapse. They also have sympathetic and sensory branches that pass through the ganglia without synapsing.

(1) The otic ganglia receive parasympathetic innervation from the inferior salivary nucleus by the glossopharyngeal nerve, along with the lesser petrosal nerve. It has sympathetic branches passing through from the superior cervical ganglion, a sensory root to the parotid gland, and a motor supply to the medial pterygoid, tensor tympani and tensor veli palatini muscles. (2) Submandibular; The parasympathetic root comes from the superior salivary nucleus through the facial nerve, chorda tympani and lingual nerve to supply the oral mucosa and submandibular and sublingual glands. Sympathetic roots come with the facial artery from the carotid plexus, and a sensory root via the lingual nerve. (3) Pterygopalatine ; This has parasympathetics from the greater petrosal branch of the facial nerve to the mucous membranes, via deep branches of the trigeminal nerve. Sympathetic branches arise from the superior cervical ganglion. There is a sensory component passing through the ganglion, from the sphenopalatine branches of the maxillary nerve. (4) Ciliary; The parasympathetic roots come from the Edinger–Westphal part of the oculomotor nucleus. Sympathetic branches that pass through come from the superior cervical ganglion, and a sensory root is from a branch of the nasociliary nerve.

Parkinson’s disease is the second most common degenerative disorder after Alzheimer’s disease. The prevalence of autonomic dysfunction varies greatly between studies due to the methodology used, but has been reported to be as high as 80%

• Lambert–Eaton myasthenic syndrome (LEMS) At around 0.3/100 000, this is a rare autoimmune disorder, which often presents as a result of a paraneoplastic syndrome in small-cell lung cancer. However, only a tiny fraction of those with small-cell cancer actually have LEMS.

Page | 25

The sympathetic chain sits paraspinally from the upper neck to the coccyx. It is the site of connections of pre- and post-ganglionic nerve fibres of the sympathetic chain

Lower limb

The deltoid ligament is a strong triangular ligament that holds the talus to the medial malleolus of the distal tibia. There are three arches in the foot: the medial longitudinal arch, the lateral longitudinal arch and the transverse arch. The image shows the foot from below. Ligaments that support the medial longitudinal arch include the long plantar ligament, short plantar ligament (plantar calcaneocuboid ligament), spring ligament (plantar calcaneonavicular ligament), interosseous ligament (talocalcaneal ligament), deltoid ligament (medial collateral ankle ligament) and the plantar aponeurosis (deep plantar fascia).

The calcaneonavicular (spring) ligament – labelled in the image of the foot seen from below - connects the plantar surface of the navicular bone to the sustentaculum of the calcaneus. It also gives support to the head of the talus, supports a significant amount of body weight, and maintains the medial longitudinal arch of the foot. Injury to this can lead to a flat foot deformity

The hip joint is a ball and socket synovial joint

• The iliofemoral ligament is the strongest ligament in the human body and does indeed prevent overextension of the hip joint. It originates from the anterior inferior iliac spine, and inserts into the intertrochanteric line of the femur. • The capsule of the hip joint is very thick and dense, much more so than the capsule in the shoulder joint. It is thickest and strongest anteriorly, where there is the most resistance. • The lateral rotators of the hip are the quadratus femoris, obturator internus, obturator externus, piriformis, gemellus superior and gemellus inferior muscles. • The rim of the acetabular labrum keeps the head of the femur inside the acetabulum, enclosing it beyond its equator, so increasing hip stability

The vastus medialis is one of the quadriceps muscles. It is attached to the medial border of the patella. The patella will dislocate laterally because of the pull of quadriceps

The natural load to failure of the ACL is about 2000 Newtons. Grafts for ACL reconstruction can be categorised as autograft (patient’s own tissue) or allograft (from cadavers). Autografts use either the hamstring (semitendinosus, along with gracilis) or patella tendon

Page | 26

The Popliteal fossa contents (from deep to superficial) include the popliteal artery, popliteal vein, tibial nerve, common peroneal nerve, small saphenous vein and lymph nodes.

Piriformis, gluteus medius, gluteus minimus, obturator internus, obturator externus, vastus lateralis and the gemelli are all attached to the greater trochanter of the femur.

Which is responsible for the hind foot equinus deformity?

The ankles are rotated internally in club foot deformity. Much of surgery concerns either tenotomy or lengthening of the Achilles tendon.

The triad of injury involving tear of the anterior cruciate ligament, medial collateral ligament and meniscal tear is known as the O’Donoghue’s or the unhappy triad.

• Recovery after surgery from an unhappy triad injury is 4–8 months and can in some cases last up to 1 year • Priority in treatment involves reconstruction of the anterior cruciate ligament with a graft either from the patellar tendon or from the semitendinosus. During the procedure the meniscal tear can also be corrected. Depending on the grade of injury, the medial collateral ligament may heal with immobilisation. • Meniscal tear associated with this kind of injury occur in the vast majority (80%) to the lateral meniscus and only in 20% in the medial meniscus. While O’Donoghue’s original description was of medial meniscal injury, this was revised in the early 1990s to describe lateral meniscal tears.

The tibial nerve is the large terminal branch of the sciatic nerve. It arises at the apex of the popliteal fossa, giving off a small cutaneous branch – the medial sural cutaneous nerve. This is a purely sensory nerve. The sural nerve is formed by this branch from the tibial and another from the common fibular nerve

The common fibular (peroneal) nerve usually gives off two purely sensory and two mixed branches.

• Common fibular (peroneal) nerve entrapment results in a high stepping gait, where the patient is noted to lift the affected foot excessively from the ground during the swing phase, to overcome their inability to dorsiflex the foot and therefore to avoid the foot hitting the floor. This results in excessive hip and knee flexion. It is sometimes called slapping gait as the affected foot ‘slaps’ the floor. A shuffling gait is associated with Parkinson’s disease • Maisonneuve fracture refers to a combination of a fracture of the proximal fibula together with an unstable ankle injury (widening of the ankle mortise on x-ray), often

Page | 27

comprising ligamentous injury (distal tibiofibular syndesmosis, deltoid ligament) and/or fracture of the medial malleolus. The gluteus medius and minimus (abductors) insert into the greater trochanter on the lateral side

The ischial spines are palpable per vaginam, and allow the guidance of a needle to the pudendal canal for transvaginal pudendal nerve block.

The adductor canal (Hunter’s or subsartorial canal) extends from the apex of the femoral triangle to the opening in adductor magnus. It contains the femoral artery and vein, saphenous nerve and the nerve to vastus medialis.

The tibio-peroneal (TP) trunk is the direct continuation of the popliteal artery. The TP trunk then divides into the peroneal and posterior tibial arteries.

The femoral canal is the most medial feature in the sheath (which contains the femoral vein, artery and the femoral canal – the nerve is outside of the sheath). It is sometimes referred to as the ‘empty space’ – this is because it lies medial to the femoral vein, and this space is where the vein expands into during times of increased venous return.

The medial (adductor) compartment of the thigh contains adductor longus, adductor brevis, adductor portion of adductor magnus, gracilis, pectineus and obturator externus. They are all supplied by the obturator nerve except the pectineus, which is supplied by the femoral nerve.

The knee is a modified synovial hinge joint. The presence of a cavity and synovial fluid makes the joint freely movable. The ‘hinge’ describes the flexion–extension movements, however the knee also permits rotation and slight varus/valgus movements. It is therefore typically described as a modified hinge joint. There are many different ways of classifying joints in the body. The easiest way is to consider the structure of joints (fibrous, cartilaginous and synovial), and the function of the joints (immovable, slightly movable, and freely movable)

The hip joint is a ball and socket synovial joint. The inferior tibiofibular joint is a fibrous joint ie fixed, and held together by fibrous connective tissue, known as the syndesmosis There is no synovial cavity. Other examples of fibrous joints are sutures between cranial bones, and articulations between the teeth and the mouth. The intermetatarsal joints between the proximal ends of the metatarsals allow gliding, not flexion/extension

The gluteus maximus is the most superficial muscle and is supplied by the inferior gluteal nerve. It extends and laterally rotates the hip. The superior and inferior gluteal arteries supply the gluteus maximus and are branches of the internal iliac artery

The anterior cruciate ligament (shown in blue below) provides stability to the knee joint. High impact sports that involve rapid changes of direction or direct lower limb trauma are often associated with injuries to the ligament. The ligament originates from the medial aspect of the lateral femoral condyle to insert into the intercondyloid tibial eminence. Lachman’s test requires the knee to be slightly flexed, and is positive when the tibia can be pulled abnormally anteriorly from a fixed femur

Page | 28

• Posterior cruciate ligament injury can be diagnosed with the posterior drawer test. The classical description of how this injury is sustained is the knee hitting the dashboard of a car – essentially, a direct blow to a flexed knee. • The posterior cruciate ligament (PCL) is stronger, but shorter and less oblique in its direction, than the anterior. It is shown in yellow in the image below. It is attached to the posterior intercondyloid fossa of the tibia and to the posterior extremity of the lateral meniscus and passes upward, forwards and medialward, to be fixed into the lateral and front part of the medial condyle of the femur. This configuration allows the PCL to resist forces pushing the tibia posteriorly relative to the femur.

The knee joint can flex to about 135 degrees, maximised by the posterior rollback of the femur on the tibia during flexion

The saphenous nerve is a branch of the femoral nerve and supplies skin on the medial side of the lower leg

• The sural nerve is a branch of the tibial nerve but it receives a communicating branch from the common peroneal nerve. It is a purely sensory nerve, providing sensation to the posterolateral aspect of the distal third of the leg, and the lateral aspect of the foot, heel and ankle.

The prepatellar bursa (labelled in green) lies between the patella and the overlying skin. Patient can develop prepatellar bursitis – sometimes known as ‘housemaid’s knee’. This is caused by recurrent, minor injury to the kneecap, eg where the patient spends a long time kneeling forwards and putting pressure on their patella. There are four anterior bursae, four posterior bursae, two medially and two laterally – key ones are labelled in the image. They are fluidfilled sacs which reduce friction over bony prominences, and can be either communicating or non-communicating with the knee joint.

• The popliteal bursa is found behind the knee, not in front, between the popliteus tendon and the posterior aspect of the tibia and fibula. • Inflammation of the infrapatellar bursa, found between the skin and tibial tuberosity, is often known as ‘Clergyman’s knee’. • Suprapatellar bursa, This is found between the femur and quadriceps muscle, communicating with the synovial joint

Page | 29

The medial plantar nerve is a branch of the posterior tibial nerve (L4–S3; a branch of the sciatic nerve) and supplies sensation to the medial three and a half toes on the plantar surfaces and the dorsal surfaces to the nail beds – it is highlighted in pink below in the image of the sole of the foot. Compression of the tibial nerve as it travels through the tarsal tunnel can give rise to, among others, the symptoms described.

Neck

The inferior thyroid artery should be ligated in continuity before the inferior thyroid veins. Mass ligation should not be performed because of the risk of injury to the recurrent laryngeal nerve. The superior thyroid artery arises from the external carotid artery and enters the upper pole of the thyroid gland close to the external laryngeal nerve.

• The inferior thyroid artery, absent in 5%, arises from the thyrocervical trunk of the subclavian artery • The isthmus is normally inferior to the thyroid cartilage and in front of the second and third tracheal rings, although variations are common.

The hypoglossal nerve passes through the superior part of the carotid sheath. It supplies all of the muscles of the tongue except the palatoglossus muscle, which is supplied by the pharyngeal plexus and accessory nerve

The cricothyroid is the only tensor muscle of the larynx. It brings the thyroid forward to tense and elongate the vocal cords.

Page | 30

• Posterior cricoarytenoid opens the vocal cords by abducting and laterally rotating the arytenoid cartilage. They are the only muscles to open the vocal cords. • Lateral cricoarytenoid acts in the opposite manner to the posterior cricoarytenoid, ie adducts and medially rotates the arytenoid cartilage. • Thyroarytenoid is antagonistic to the cricothyroid muscle – it decreases the tension of the vocal cords • The transverse arytenoid brings the arytenoids towards each other when they contract, so adducting the vocal folds.

The trachea commences just below the cricoid cartilage (at the level of C6). Within the thorax and on the right, the trachea is in contact with the pleura, vagus and subclavian artery. On its left, the trachea is in contact with the left recurrent laryngeal nerve, aortic arch and the left common carotid and subclavian arteries. The trachea ends at the upper border of T5, where it bifurcates.

• The bifurcation of the right common carotid artery is associated with the upper border of the lamina of the thyroid cartilage, at approximately the level of the fourth cervical vertebra.

Local anaesthetic can be injected along the posterior border of the sternocleidomastoid (between the superior and inferior thirds) – the nerve point of the neck for a cervical plexus block. The transverse cervical nerve emerges as a single trunk behind the posterior border of the sternocleidomastoid and is superficial to the muscle.

The borders of the posterior triangle are anteriorly – the posterior border of sternocleidomastoid; the ‘apex’ of the triangle – the junction of sternocleidomastoid and trapezius; and posteriorly – the anterior border of trapezius; the ‘base’ of the triangle – the middle third of the clavicle. About an inch superior to the ‘base’/middle third of the clavicle lies the inferior belly of omohyoid. This divides the posterior triangle into two further subtriangles. The more superior of these triangles is known as the occipital triangle; the more inferior one, the subclavian triangle .

The internal jugular vein receives the facial, pharyngeal, lingual and superior and middle thyroid veins. It is formed from the sigmoid and inferior petrosal sinus and continues to the brachiocephalic vein. The external jugular arises from the junction of the posterior auricular vein and the posterior division of the retromandibular vein and drains into the subclavian. Note that the anterior portion of the retromandibular vein proceeds forward to the anterior facial vein, and together they join to form the common facial vein. The transverse cervical vein is a tributary of the external jugular vein.

The conus elasticus is mostly yellow, elastic tissue. It is the lateral part of the cricothyroid membrane. Superiorly, its free edge forms part of the vocal ligaments.

• The thyroepiglottic ligament connects the thyroid cartilage to the lower edge of the epiglottis

Page | 31

There are several anatomical differences between adults and children that make their intubation more difficult:

1. Head size: children have a relatively larger head, which tends to flex the head on the neck, making airway obstruction more likely. 2. Tongue size: the relatively larger tongue in children tends to flop back and obstruct the airway in the obtunded child, which means that there is less room in the mouth when intubation is being carried out. 3. Larynx: The larynx is positioned more cephalic in children. (glottis at C3 in infants compared with C6 in adults) 4. Trachea: the trachea is shorter in children. 5. Finally, the narrowest point of a child’s airway is the cricoid ring; in adults, it is the glottis

Turning head to the left against direct opposing pressure from your hand:

• The movement takes place at the atlantoaxial joint about a vertical axis through the odontoid process of the axis. • The head is turned using the contralateral sternocleidomastoid muscle. • The atlanto-occipital joint enables flexion and extension of the neck.

The external jugular vein drains most of the scalp and side of the face. It begins near the angle of the mandible and is formed from the union of retromandibular and postauricular veins, receiving branches from the posterior external and transverse cervical veins. The external jugular vein has two pairs of valves, which do not prevent regurgitation of the blood upward. The lower pair are placed at its entrance to the subclavian vein, the upper (in most cases) about 4 cm above the clavicle. The external jugular vein lies anterior to scalenus anterior and pierces the deep fascia of the neck, usually posterior to the clavicular head of the sternocleidomastoid muscle before draining into the subclavian vein.

• The internal jugular vein lies in the carotid sheath. It lies anterior to the anterior scalene muscle. • The external carotid artery is a branch of the common carotid artery that supplies the head and neck. It begins at the level of C4, which corresponds roughly to the upper border of the thyroid cartilage

Page | 32

The inferior thyroid artery is a branch of the thyrocervical trunk, which arises from the subclavian artery. The inferior thyroid artery supplies the inferior pole of the thyroid gland and gives off oesophageal branches. In addition, it supplies both the superior and inferior parathyroid glands. The course of the inferior thyroid artery is closely related to the recurrent laryngeal nerve: in a thyroidectomy, therefore, it is preferable to ligate the artery lateral to the gland to avoid neural damage.

Anterior jugular veins and inferior thyroid veins may be encountered during tracheostomy.

The carotid canal is found on the inferior surface of the petrous part of the temporal bone. It ascends vertically at first and then bends and runs horizontally forward and medially. It transmits the internal carotid artery and the carotid plexus of nerves into the cranium.

The facial nerve innervates the muscles of facial expression, including the platysma. The platysma is a superficial sheet of muscle that takes origin from the fascia and skin of the upper chest; it inserts into the mandible and skin of the lower face. It is a wide, paired, flat muscle, with the sheets meeting in the midline.

Berry’s ligament connects the thyroid to the cricoid cartilage and upper trachea

Pelvis

The superficial primarily drain the external genitalia of the male.

The prostatic venous plexus drains the prostate gland. This plexus drains into the internal iliac veins and also has connections to the internal vertebral venous plexus and the Batson’s venous plexus.

The cervix is an anterior relation of the . On DRE, the structures that can be palpated in either sex are: the coccyx and lower sacrum posteriorly; the ischial spines and ischiorectal fossas are lateral; and the anorectal ring at the anorectal junction. In men, the prostate can be felt (but normal seminal vesicles are not usually palpable). In women, the cervix can be felt through the vaginal wall, with the uterosacral ligaments laterally, and sometimes the ovaries.

• The ischial tuberosities are impalpable during a DRE.

The sciatic nerve is indeed the largest nerve in the body and with a width of about 2 cm, it has a ‘band- like’ appearance. Its course is as follows: it travels from the pelvis into the gluteal region through the greater sciatic foramen travels inferiorly to the piriformis muscle (although this is variable and the nerve can pass through piriformis or occasionally above piriformis) enters posterior thigh, passing deep to the long head of the biceps femoris

• bifurcates at the apex of the popliteal fossa into: tibial nerve and common fibular (peroneal) nerves • The sciatic nerve crosses the posterior surface of the obturator internus, superior and inferior gemellus and quadratus femoris muscles

The upper third of the rectum is covered by peritoneum anteriorly and laterally; the middle third only anteriorly and the lower third is not covered by any peritoneum

Page | 33

• There is no rectal mesentery – the mesorectum is a clinical term for the visceral pelvic fascia that contains pararectal lymph nodes.

The ‘fossa navicularis’ is a normal small dilatation found in the distal penile urethra.

• The membranous urethra does pass through the urogenital diaphragm, surrounded by the external urethral sphincter. The external urethral sphincter is distal to the verumontanum, so during a trans-urethral resection of the prostate the resection is limited to the verumontanum distally to avoid incontinence. • The anterior urethra is lined with pseudostratified columnar epithelium, whereas distally it is lined by squamous epithelium

Anal canal; Venous drainage above the dentate line occurs via the superior rectal vein into the inferior mesenteric vein. Below the dentate line, venous drainage occurs via the inferior rectal vein into the internal pudendal vein.

• Arterial supply: above the dentate line: superior rectal artery (from ima) below the dentate line: inferior rectal artery (from the internal pudendal artery). • The anal canal is made up of a circular muscular layer that forms the internal and external anal sphincters. This is unlike the rest of the gastrointestinal (GI) tract, which has both circular and longitudinal fibres • Innervation above the dentate line and to the internal anal sphincter is attributed to the pelvic plexus (sympathetic) and parasympathetic with afferent sensory contributions (pelvic splanchnic nerves) • The lymph of the perineum and the anal canal is drained to the superficial inguinal lymph nodes, whereas the lymph of the pelvic territory flows to iliac and lumbar nodes.

The prostatic nerve plexus contain cavernous nerves which provide parasympathetic innervation. It is responsible for the vascular changes which result in erection:

‘P’ – Parasympathetic = Point (erection). ‘S’ – Sympathetic = Shoot (ejaculation)

The pudendal nerve is formed by the ventral rami of the sacral spinal nerves from S2, S3 and S4. It maintains voluntary urinary continence by facilitating contraction of the external urethral sphincter. It also helps to maintain voluntary faecal continence via contraction of the external anal sphincter. Damage to this nerve could result in urinary or faecal incontinence. Cyclists may develop pudendal nerve entrapment syndrome (also known as Alcock canal syndrome), which can present with various symptoms including pain, urinary and faecal incontinence and sensory disturbances of the perineum and genitalia due to irritation, entrapment or damage of the pudendal nerve.

The genitofemoral nerve is formed by the branches of the anterior rami of L1 and L2 spinal nerves of the lumbar plexus. It divides into the genital and femoral branches just above the inguinal ligament. In men, it travels with the spermatic cord and supplies motor fibres to the cremaster and dartos muscles. It also supplies sensory fibres to the spermatic fasciae and tunica vaginalis of the testis. In addition, it supplies sensory innervation to the upper part of the scrotum. Damage to this nerve could result it numbness of the upper scrotum but it would not result in any urinary incontinence.

Page | 34

The levator ani is a large sheet of muscles made up of three paired muscles, which are the pubococcygeus, puborectalis and iliococcygeus muscles.

• The three components of the pelvic floor are: levator ani muscles, coccygeus muscle and fascia coverings of the muscles

The prostate is a fibromuscular and glandular organ that consists of four lobes: anterior, posterior, median and lateral. The lateral lobes lie on either side of the urethra.

• The bulbourethral or Cowper’s gland are a pair of small exocrine glands located in the urogenital diaphragm that produce an alkaline mucous secretion before ejaculation that drains into the spongy urethra to neutralise traces of acidic urine in the urethra.

The lateral cutaneous nerve of the thigh is a branch of the lumbar plexus arising from the dorsal branches of the second and third lumbar nerves, not the sacral plexus.

• The inferior gluteal nerve, posterior cutaneous nerve of the thigh and pudendal nerves pass through the greater sciatic foramen below piriformis. It is responsible for innervation of gluteus maximus and has no sensory function. The following nerves pass through the greater sciatic foramen: 1. ABOVE piriformis – superior gluteal nerve 2. BELOW piriformis – inferior gluteal nerve, posterior cutaneous nerve of the thigh and pudendal nerve

The lymphatic drainage of the uterus is as follows:

1. fundus: para-aortic nodes 2. body and cervix: internal and external iliac nodes; superficial inguinal nodes.

The pelvic floor musculature provides the primary support to the uterus. The ligaments which provide secondary support are the broad ligament, round ligament, ovarian ligament, cardinal ligament and uterosacral ligament. The uterus is covered by peritoneum anteriorly and superiorly except for the vaginal part of the cervix.

The majority of ectopic pregnancies (80%) occur in the ampullary portion of the Fallopian (uterine) tubes. The uterine (or Fallopian) tube begins at the infundibulum and progresses medially to include the ampulla, the isthmus and the intramural portion.

Only 5% of ectopic pregnancies occur in the fimbria and therefore is not the most likely site of implantation.

In total, 0.2% of all ectopic pregnancies occur in the ovary. Along with the cervix, it is the most unlikely site of implantation.

The isthmus is the second most common site of ectopics at 12%.

Only 1.4% of all ectopic pregnancies occur within the abdominal cavity

Page | 35

What is the correct order of layers one would pass through when making an incision through the scrotum to the testes?

• Skin, dartos fascia (a continuation of Scarpa's fascia), external spermatic fascia, cremaster muscle and fascia, internal spermatic fascia, parietal layer of tunica vaginalis, visceral layer of tunica vaginalis, tunica albuginea of testis • the layers of the abdominal wall extend to the scrotum as the testes descend during development.

The transverse perineal muscles do indeed pass between urethra and anus into a central tendinous portion

The testes share T10 innervation with the ureter and the umbilicus, which is why testicular pain can often be felt in the abdomen and ureteric colic is sometimes accompanied by perception of pain in the testes Upper limb and breast

Teres minor originates from the lateral border of the scapula and inserts into the inferior facet of the greater tubercle. along with the other rotator cuff muscles. It is innervated by the axillary nerve (C5 C6) which is a continuation of the posterior cord from the brachial plexus. It innervates teres minor and deltoid muscles. One way to remember this is that a child (minor) needs help (an auxiliary). A patient with an axillary nerve transection would also have anaesthesia in the ‘regimental badge area’ – the skin over the lower deltoid. This is because the axillary nerve gives rise to the upper lateral cutaneous nerve of , which gives sensory supply to this area.

The rotator cuff muscles are all supplied by more proximal nerve roots within the brachial plexus.

• Supra- and infraspinatus are supplied by the suprascapular nerve (C4, C5, C6); • subscapularis by the upper and lower subscapular nerves (C5, C6, C7); and • teres minor by the axillary nerve (C5, C6) the nerve roots of the musculocutaneous nerve are C5, C6 and C7

The axillary artery begins at the lateral border of the first rib as the continuation of the subclavian artery, and ends at the inferior border of the teres major. For descriptive purposes the axillary artery is divided into three parts by the pectoralis minor.

1. The first part has one branch (superior thoracic), 2. the second part gives off two branches (thoracoacromial and lateral thoracic), and 3. the third part gives off three branches (subscapular, anterior circumflex humeral, and posterior circumflex humeral).

The lateral thoracic artery is larger in women, and it supplies the lateral part of the mammary gland

Remembering the brachial plexus: 3:1:0:3:5:5

• 3 branches from roots (LSD) 1. Long thoracic nerve C5,6,7,

Page | 36

2. Nerve to Subclavius, 3. Dorsal scapular nerve • 1 from trunk 1. Suprascapular nerve C5,6 • 0 from divisions • 3 branches from lateral cord (LML) 1. Lateral pectoral nerve 2. Musculocutaneous nerve 3. Lateral root of the median nerve. • Five branches from medial cord (Miss Mary Makes Me Unhappy) 1. Medial pectoral nerve 2. Medial cutaneous nerve of forearm 3. Medial cutaneous nerve of arm 4. Medial root of median nerve Ulnar nerve. • Five branches from posterior cord (2 STAR): 1. 2 Subscapular nerves (upper/lower) 2. Thoracodorsal nerve 3. Axillary nerve 4. Radial nerve.

Various muscles attach the scapula (shoulder blade) to the chest wall and help maintain normal scapular control. These muscles include trapezius, levator scapulas, rhomboids major, rhomboids minor, pectoralis minor and serratus anterior. The latissimus dorsi has a small attachment at the base of the scapula but does not significantly contribute to scapular stability. Of these muscles, the serratus anterior and the trapezius are the most important. A winging scapula is nearly always associated with partial or complete paralysis of either of these muscles. Weakness or paralysis of the serratus anterior, secondary to palsy of the long thoracic nerve, is the commonest cause of winging. The long thoracic nerve (from the C5-C7 roots) is thin, fragile and runs an anatomical course in the neck and upper thorax that makes it susceptible to damage by compression or trauma

Accessory nerve (XI) damage can also produce scapular winging via weakness of trapezius, but this would be milder, is rarer, and would be expected to be associated with weakness of shoulder elevation. Nerve conduction studies and electromyography would help confirm the diagnosis.

The cephalic vein begins at the radial aspect of the dorsum of the hand in the anatomical snuffbox and ascends up the lateral aspect of the arm. It continues up the lateral aspect and arches over to run in the deltopectoral groove where it pierces the clavipectoral fascia to join with the axillary vein

The suprascapular nerve runs beneath the suprascapular ligament (otherwise known as the superior transverse scapular ligament), which crosses the suprascapular notch. The suprascapular artery conversely runs above the ligament.

Screw The Lawyer Save A Patient Parts and branches of Axillary Artery S Superior thoracic (first part)

Page | 37

T Thoracoacromial (second part)

L Lateral thoracic (second part)

S Subscapular circumflex humeral artery (third part)

A Anterior circumflex humeral artery (third part)

P Posterior circumflex humeral artery (third part)

The median nerve is lateral to the brachial artery proximally, and the median nerve crosses anteriorly over the brachial artery in the middle of the arm to become medial to the artery in the cubital fossa. The brachial artery does not cross over the nerve.

• The most superior branch of the brachial artery is the profunda brachii. The nutrient humeral artery (where present) arises inferior to it from the lateral aspect of the brachial artery. The brachial artery begins at the inferior border of the teres major as the continuation of the axillary artery, and ends opposite the neck of the radius. During its initial course it runs with the median nerve, which crosses anterior to the artery in the middle of the arm. • Within the cubital fossa, the brachial artery is between the biceps tendon (laterally) and the median nerve (medially).

The elbow joint consists of three joints: between the ulna and the humerus (trochlea and trochlear notch); between the radius and humerus (radial head and capitulum/capitellum); and finally between the radius and ulna (proximal radioulnar joint). It allows movement only in one plane. Remember that supination and pronation of the forearm occur at the proximal and distal radioulnar joints

Blood supply to the scaphoid is chiefly via the dorsal carpal branch of the radial artery, with some contribution from the superficial palmar branch of the radial artery on the palmar surface. Both of these arteries supply the distal pole and then supply the proximal pole in a retrograde fashion through interosseous perforators. When the scaphoid fractures through the waist, the fracture line severs the blood supply to the proximal fragment, and avascular necrosis of the proximal pole is therefore a common complication. Tenderness in the anatomical snuffbox may be the only clinical sign of the fracture before radiological evidence develops (often around 2 weeks later).

The median nerve supplies sensation over the radial half of the palm via the palmar cutaneous branch (shown in blue below – the image shows median nerve sensory distribution in the hand), which is given off before the median nerve enters the carpal tunnel, and palmar sensation is therefore spared in carpal tunnel syndrome. However, if the nerve is anaesthetised at the elbow then sensation over the thenar eminence will be affected.

Page | 38

The brachial artery divides in to its two terminal branches (the smaller radial artery and the larger ulnar artery) in the distal aspect of the antecubital fossa, opposite the neck of the radius.

The cubital is described as having the following borders, as shown in the image:

• Proximally: a line connecting the medial and lateral epicondyles of the humerus • Medially: lateral border of the pronator teres • Laterally: medial border of brachioradialis • Roof: Bicipital aponeurosis, antebrachial and brachial fascia, subcutaneous fat and skin • Floor: Brachialis and supinator muscles.

The blood supply to the breast is chiefly from the lateral thoracic artery (from the axillary) and the internal thoracic also known as the internal mammary artery (from the subclavian). It also receives a contribution from the pectoral branch of the thoracoacromial artery, and the second to sixth intercostal arteries.

The axillary vein is formed by the union of the basilic vein (which runs up the medial/ulnar aspect of the forearm and arm) and the venae comitantes of the brachial artery at the inferior border of teres major. The cephalic vein by contrast is formed at the radial aspect of the dorsal venous network of the hand, running over the anatomical snuffbox and then up the lateral/radial aspect to joins the axillary vein directly.

The basilic vein pierces the deep fascia of the arm (brachial fascia) approximately half way up the arm. It runs with the brachial artery for a short distance before joining with the venae comitantes of the brachial artery at the inferior border of teres major to form the axillary vein Spine The lumbosacral trunk (L4/L5) passes anterior to the ala of the sacrum, under cover of the common iliac vessels, and joins the S1 ventral ramus anterior (deep) to the piriformis muscle.

• The adductor muscles are supplied by the L2–L4 roots of the lumbar plexus. They are supplied by anterior divisions.

The extradural, or epidural, fat is the location for introducing epidural anaesthesia and where the epidural catheter should be sited. Within this area lies a complex varied plexus of veins without valves

The ligamentum flavum can thicken and hypertrophy, narrowing the spinal canal causing spinal stenosis.

• The posterior longitudinal ligament lies anterior to spinal canal.

Page | 39

• The spinal canal is anterior to the ligamentum flavum but posterior to the intervertebral vertebral discs and posterior longitudinal ligament. The spinal cord ends at L1.

NoaH Told MariaH To Try Cervical Counting Cervical vertebral level landmarks

NH C1: Nose (base), Hard palate

T C2: Teeth

MH C3: Mandible, Hyoid bone

T C4: Thyroid cartilage (upper)

T C5: Thyroid cartilage (lower)

C C6: Cricoid cartilage

C C7: Cricoid cartilage (just below)

C3 is related to the mandible and the hyoid bone, so is the correct answer here. The strap muscles, or infrahyoid muscles either originate from or insert onto the hyoid bone.

The vertical positioning, along with the coronal orientation of the thoracic vertebral articular facets, allows a large degree of rotation in this region of the spine.

• The sacrum is almost completely fused in adults, so it has little articulation. • The cervical articular facets are relatively horizontal

Page | 40

• Although lumbar vertebral articular facets are vertical, they are positioned in a more parasagittal plane, which allows lateral and forward flexion/extension

The knee flexors (hamstrings) are innervated by L5 and S1

The ligamentum nuchae is a triangular, sheet-like structure in the median sagittal plane. The base of the triangle is attached from the external occipital protuberance to the foramen magnum, and the apex to the tip of the spinous process of C7

The spinothalamic tract does contain the second order fibres that carry pain and temperature sensation.

• The spinothalamic tract conveys pain, temperature, crude touch and pressure sensations from one side of the body to the contralateral side of the brain. • Vibration and joint position sense (proprioception) are conveyed via the posterior column. 1. The first neuron of the spinothalamic tract synapses in the dorsal horn; 2. the next neuron crosses to the contralateral side of the spinal cord and synapse in the thalamus; 3. the third neuron arises in the thalamus to pass to the cortex.

Vertebra C2 is also known as the axis, and is characterised by a prominent odontoid process called the dens, which acts as a pivot allowing the atlas and a person’s head to rotate on the axis. It is best assessed on X-ray using an anteroposterior (AP) open mouth view.

• Vertebra C1 is also known as the atlas and articulates with the head. It is distinguished by the fact that it lacks a vertebral body – the vertebral body of C1 fuses onto the body of C2 during development to become the dens of C2.

The dens is anterior in the spinal canal and anterior to the spinal cord. To protect the spinal cord, an exceptionally strong transverse ligament traverses the space between the dens and the cord.

The laminae lie on both sides of the vertebral arch, between the transverse processes and the spinal process. The pedicles complete the arch and are between the transverse processes and the body.

Thorax The main blood supply to the latissimus dorsi is via the thoracodorsal artery, a branch of the subscapular artery (which branches from the third part of the axillary artery).

Aortopulmonary (subaortic) nodes are difficult to access via mediastinoscopy as the aorta is ‘in the way’ of the advancing mediastinoscope. They are accessed via thoracoscopy [such as video-assisted thoracoscopic surgery (VATS)] rather than mediastinoscopy.

• The pretracheal space is entered as part of the mediastinoscopy procedure, and these nodes are therefore available for sampling. • Paratracheal nodes are accessible through mediastinoscopy and are commonly sampled. • Anterior subcarinal nodes are accessible for sampling through mediastinoscopy, but can be technically challenging

Page | 41

The arch is symmetrically covered by the pleura from both sides, which meet in the midline behind the manubriosternal joint.

• it arches over the right pulmonary artery. • The arch is entirely superior to the manubriosternal joint. • The ascending aorta arises from the left ventricle at the level of the manubriosternal joint.

In around 15% of patients, the posterior descending (posterior interventricular) artery does not arise from the right coronary artery, instead arising directly from the left coronary artery (in around 10%) or via a co-dominant system where it arises from both (in around 20%)

• 30% of patients do not have a right-dominant system. • 70% of patients have a right-dominant system where the posterior descending artery arises from the right coronary artery.

The azygos lobe is the commonest accessory lobe, seen in around 1% of people and around 0.4% of chest radiographs. It is an embryological variant, where the azygos vein causes a deep impression in the right upper lobe during development. The lobe is seen superior to the right hilum, separated from the rest of the lung by a groove containing the azygos vein. It is not a true lobe however, as it does not have its own separate bronchus. The azygos lobe, if present, is found in the right upper zone

• It is seen as a ‘reverse comma sign’ behind the medial end of the right clavicle

The T4/5 disc is the level of the transthoracic plane, which corresponds to the level of the manubriosternal angle (angle of Louis). At this level is: the aortic arch; the junction of the azygos vein and the SVC; the second costal cartilage; the tracheal bifurcation, and the decussation of the thoracic duct from right to left.

The oesophagus is formed at the lower border of the cricoid cartilage. It is crossed anteriorly by the left main bronchus, lies behind the left atrium and passes through the muscular part of the diaphragm to the left of the central tendon, through the oesophageal hiatus, which is formed by the muscular sling of the right crus. It is innervated, in part, by the recurrent laryngeal nerve, not the phrenic nerve.

• It passes to the left of the central tendon, not through it.

The lung hila sit either side of the heart in the mediastinum. The lung hila are at approximately the 3rd and 4th costal cartilages, corresponding to the T5/T6 vertebrae posteriorly

The moderator band (Septomarginal trabecula) runs from the interventricular septum to the anterior papillary muscle of the right ventricle. It carries the right bundle branch to the anterior papillary muscle. The conducting system of the heart is shown schematically below.

Page | 42

The pectinate muscles (musculi pectinati) are the name given to the muscular ridges in the anterior right atrium. They do not play a specific part in the conducting system of the heart, other than transmitting sinoatrial (SA) nodal impulses towards the arteriovenous (AV) node.

The papillary muscles are attached to the cusps of the mitral and tricuspid valves by the chordae tendineae. They do not play a part in the conducting system of the heart. Do not get confused with the anterior papillary muscle of the RIGHT ventricle, which receives the moderator band from the interventricular septum – part of the conducting system of the heart that carries the right bundle branch.

The crista terminalis demarcates the border between the primordial atrium and the sinus venarum. It is seen as a vertical line running between the superior and inferior vena cava in the internal aspect of the right atrium. On the external aspect of the atrium is the sulcus terminalis, which corresponds to the crista terminalis. The sinoatrial (SA) node is located in the upper aspect of the crista terminalis.

The serratus anterior has a large origin from the upper 8 ribs – it is shown in blue in the image below. It inserts into the entire medial border of the scapula. Contraction of the upper or lower aspect of the muscles therefore rotates the scapula.

The anterior scalene runs from the transverse processes of C3–C6 to insert on to the scalene tubercle of the first rib.

The middle scalene originates from the transverse processes of C5–C7 to insert on to the first rib more posteriorly. They serve as the anterior and posterior borders of the nerve roots of the brachial plexus as they emerge from the intervertebral foraminae of the C5–T1 vertebrae.

The transverse cervical artery is a branch of the thyrocervical trunk (from the first part of the subclavian artery). It gives off the superficial cervical artery (supplying trapezius) and usually the deeper dorsal scapular artery (which supplies levator scapulas and the rhomboids). The dorsal scapular artery can also branch directly from the third part of the axillary artery

The phrenic nerve runs over the anterior surface of scalenus anterior and enters the chest by descending from the medial lower border of the scalenus anterior muscle, between the subclavian vein and artery. The autonomic fibres in the phrenic nerve are sympathetic and pass from the superior (C1–

Page | 43

C4) and middle (C5/C6) sympathetic cervical ganglia as grey rami into the C3–C5 roots of the phrenic nerve, and innervate blood vessels in the diaphragm. The nerve lies on the fibrous pericardium and is sensory to the mediastinal and diaphragmatic pleura, and also to the diaphragmatic peritoneum.

During development, the heart tube begins looping around day 23 – as shown in the stages below. The sinus venosus gives rise to the smooth part of the right atrium (the sinus venarum – as opposed to the auricle, which develops from the primitive atrium) and the coronary sinus

The heart is the first organ in the embryo to begin functioning, and it begins to beat in the fourth week of development. The primordium of the heart forms in the cardiogenic plate located at the cranial end of the embryo. Angiogenic cell clusters, which lie in a horseshoeshaped configuration in the plate, coalesce to form two endocardial tubes. These tubes are then forced into the thoracic region due to cephalic and lateral foldings, where they fuse together forming a single endocardial tube during the third week. The heart begins to beat in the fourth week at about the same time that the septum primum appears and the bulboventricular loop is formed (shown in the images between days 24 and 35). From the fourth week onwards, septa begin to grow in the atria, ventricle and bulbus cordis to form right and left atria, right and left ventricles and the two great vessels – the pulmonary artery and the aorta. By the end of the eighth week, partitioning is completed and the fetal heart has formed.

The left coronary artery branches from the aorta at the left coronary cusp, and runs a short distance between the pulmonary trunk and the left atrial appendage (left auricle). In the short segment, it is also referred to as the left main stem. It then gives off the left anterior descending and branches in to the left circumflex and left marginal arteries. The heart is shown from the front below.

In around 80% of people the inferior cervical ganglion fuses with the first thoracic ganglion to form the stellate ganglion (otherwise known as the cervicothoracic ganglion). The stellate ganglion lies at the level of C7, in front of the neck of the first rib.

Page | 44

A widened mediastinum should arouse suspicion of aortic injury, and is a sensitive indicator. However, it is non-specific, and aortic injury can occur without evidence of mediastinal widening. Chest radiographs are the initial screening investigation performed for the trauma patient with blunt thoracic injuries, however the sensitivity and specificity for detection of aortic injuries is variable and suspicious clinical or radiological findings warrant a chest CT. Various signs on chest X-ray can arouse suspicion of a blunt aortic injury:

1. widened mediastinum 2. obliteration of the aortic knuckle 3. depression of the left main bronchus 4. large left-sided haemothorax 5. pleural cap (blood above the left lung apex) depression of right bronchus

The transverse pericardial sinus lies posterior to the aorta and pulmonary trunk, and anterior to the superior vena cava (SVC). It can be used in cardiac surgery, as seen below, to interrupt flow from the heart and enable coronary bypass. The oblique pericardial sinus, by contrast, is a blind-ending sinus posterior to the heart (below the transverse sinus) that ends between the left and right pulmonary veins.

The first rib does not move during respiration, and therefore does not contribute to respiratory effort.

• The serratus anterior is one of the accessory muscles of respiration and is recruited to increase the respiratory effort.

Page | 45

Lower lobe consolidation typically causes the diaphragmatic borders to become indistinct, in contrast to middle lobe consolidation which causes the right heart border to become indistinct, and lingular consolidation which involves the left heart border.

• Kerley B lines are one of the signs of pulmonary oedema, caused by oedema of the septae between the lung lobules. Kerley B lines A Alveolar bat-wing shaped shadowing

B Kerley B lines

C Cardiomegaly

D Dilated upper lobe vessels

E pleural Effusions

The right coronary artery supplies the AV node in around 85–90% of people, via the AV nodal branch.

Endothoracic fascia lies between the inner aspect of the chest wall and the parietal pleura, and it is this layer that provides a cleavage plane for the parietal pleura in pleurectomy.

• The transversus thoracis fascia is associated with the transversus thoracis muscle only, and does not play a role with the pleura.

The SVC enters the right atrium behind the right third costal cartilage.

The right main bronchus is more vertical than the left, and is the usual site where aspirated foreign bodies end up. As this patient has aspirated the tablet while lying supine, it is likely to fall in to the superior segmental bronchus of the right inferior lobe, which branches posteriorly.

The subclavian artery arises from the brachiocephalic trunk on the right, and directly from the aorta on the left. It supplies:

1. the thyroid gland via the inferior thyroid artery, a branch of the thyrocervical trunk 2. the breast, rectus abdominus and diaphragm via the internal thoracic artery 3. the brainstem, cerebellum and posterior cerebral circulation via the vertebral arteries 4. the upper limb and muscles of the back.

Aortic stenosis is associated with ‘drop attacks’, as the stenosed valve causes a fixed limitation on maximal cardiac output which is then unable to maintain cerebral perfusion with peripheral vasodilation. This is also why anaesthesia in patients with severe aortic stenosis is more risky. The classic triad for aortic stenosis is heart failure, angina, and syncope. Bicuspid aortic valves are prone to early calcification, causing aortic stenosis that presents in middle-age rather than in the elderly. They are also associated with sudden death. Bear in mind that the commonest congenital causes of sudden cardiac death are arrhythmogenic conditions such as a long QT interval, Brugada or catecholaminergic polymorphic VT (CPVT).

Page | 46

• Small ventricular septal defects are usually of no significance and close early in childhood. If large, they lead to shunting of blood from the left to the right side of the heart due to the left– right pressure gradient. If untreated this will eventually progress to Eisenmenger’s syndrome (particularly with pulmonary outflow obstruction), in which right-sided heart pressures become higher than left-sided pressures and deoxygenated blood is shunted from right to left. They are not commonly associated with sudden cardiac death. • Although dextrocardia and situs inversus can be associated with other abnormalities such as Kartagener syndrome (bronchiectasis, sinusitis, and situs inversus), it is not commonly associated with sudden cardiac death. • Patent foramen ovale does not cause cardiovascular effects, as atrial pressures are similar between the left and right. It can however lead to paradoxical embolic phenomena such as stroke/TIA. These paradoxical emboli can very rarely cause myocardial infarction (MI) (and therefore potentially sudden death), but would not be associated with syncopal. • Single coronary arteries are extraordinarily rare, and would not be associated with drop attacks as described. While isolated dextrocardia can be associated with severe cardiac anomalies, dextrocardia with situs inversus has a low incidence of accompanying defects and the heart functions normally.

The thoracic sympathetic chain lies on the heads of the ribs, anterior to the posterior intercostal vessels, immediately under cover of the pleura, with the splanchnic nerves passing from the chain medially and anteriorly over the vertebral bodies. The thoracic sympathetic chain receives white rami from all the intercostal nerves, and passes into the abdomen under the medial arcuate ligament of the diaphragm.

• The spinal cord, not the sympathetic chain, runs in the vertebral canal and is covered by the meningeal layers and bathed in cerebrospinal fluid.

The aortic valve lies in the centre of the heart when viewed from above, with the pulmonary valve lying anteriorly, the mitral valve to the left, and the tricuspid valve to the right.

• The aortic valve, along with the tricuspid and pulmonary valves, consists of three leaflets. However, a proportion of patients (1–2%) are born with a congenitally bicuspid valve, which predisposes them to calcification, aortic stenosis and regurgitation in later life. Unicuspid and quadricuspid valves are also possible. • The right coronary artery arises from the right aortic sinus, and the left coronary artery from the left. The posterior sinus has no artery arising from it, and is therefore referred to as the non-coronary sinus. • The atrioventricular node (not the SA node) lies approximately between the right and noncoronary cusps of the aortic valve in the triangle of Koch. .

Page | 47

The trachea branches into the left and right main bronchi at the carina, with the left dividing into upper and lower, and the right dividing into upper, middle and lower bronchi. These in turn branch into segmental bronchi – the right lung has ten bronchopulmonary segments. Each of these segments are supplied by a bronchus and by a pulmonary artery, and are therefore distinct and resectable.

• The left and right pulmonary arteries supply deoxygenated blood at low pressure from the right ventricle to the lungs for gas exchange. The lungs themselves are supplied by bronchial arteries that branch from the descending thoracic aorta

β -Adrenoceptors are present in the heart (cardiac ventricles) and are both chronotropic and ionotropic. β - Adrenoceptors are present in blood vessels, causing vasodilation; in gut smooth muscle causing relaxation; in bronchial smooth muscle causing relaxation; and in the bladder wall causing relaxation.

• The aortic arch (along with the carotid sinus) contains baroreceptors, which mediate arterial tone and heart rate and contractility to maintain arterial pressure.

Although there are various landmarks used for approaches to the subclavian vein, a commonly used landmark for infra-clavicular subclavian vein cannulation is 1–2 cm inferior and lateral to the clavicular transition point (which is approximately the junction of the medial one-third and lateral two-thirds of the clavicle), with the needle directed towards the suprasternal notch. It is also worth bearing in mind that this procedure is normally performed under ultrasound guidance. Risks include pneumothorax, chylothorax (on the left due to damage to the thoracic duct), nerve injury, and arterial puncture. If subclavian vein cannulation is attempted and fails, it is not advised to attempt to place a subclavian line on the opposite side due to the risk of inducing bilateral pneumothoraces. If there is unilateral lung pathology it is usually best to place the subclavian line in the ipsilateral side to avoid the risk of a pneumothorax in the good lung

• Although a supra-clavicular approach to subclavian vein cannulation is possible, the landmarks would be 1 cm superior and lateral (not medial) to the junction of the lateral aspect of the sternocleidomastoid muscle and the clavicle. • In obese patients, the standard position for right subclavian central venous cannulation is 2 cm under the mid-point of the clavicle and 1 cm laterally. In thin patients the standard position for insertion is 1 cm under the mid-point of the clavicle and 0.5 cm laterally. • Jugular vein cannulation is now the preferred choice for central venous catheterisation, as insertion under ultrasound guidance is associated with a much lower rate of complications than subclavian insertion.

The lingula is a branch of the left upper lobe. The left lung is divided into upper and lower lobes by a long deep oblique fissure. The upper lobe has a wide cardiac notch on its anterior border. The anteroinferior part of the upper lobe also has a small tongue-like projection called the lingula.

The cervical pleura is also known as the dome or cupola. It is important to know that the pleura extends approximately 2.5 cm above the medial one-third of the clavicle, higher on the left. Pneumothorax is a potential complicating factor in neck stabbings.

The azygos vein arches over the right main bronchus from behind and drains directly in to the SVC at the T4/5 level (the transthoracic plane), which approximates to the manubriosternal angle. This level also

Page | 48 marks the carina, the start and end of the aortic arch, the point at which the azygos vein joins the SVC, the level at which the thoracic duct moves from right to left, and the level at which the left recurrent laryngeal nerve loops around the aortic arch. The azygos vein itself is on the right side, and receives blood from the posterior intercostal veins and segmental veins of the abdomen, It passes through the diaphragm via the aortic opening at T12 and ascends on the right side of the vertebral bodies posterior to the oesophagus. On the left a variable pattern exists, which usually involves an accessory hemiazygos vein (superiorly) and hemiazygos vein (inferiorly).

Dorsal and ventral primary rami are the first branches from spinal nerves.

• Dorsal rami provide sensory innervation to the skin over the back and give motor innervation to the true back muscles; • ventral rami supply sensory innervation to the skin over the limbs and the skin over the ventral side of the trunk, as well as motor innervation to the skeletal muscles of the neck, trunk and extremities.

The dorsal and ventral roots of spinal nerves are not directly responsible for any sensory innervation to the skin. Dorsal and ventral rootlets emerge from the spinal cord to form the dorsal and ventral roots. The ventral roots contain efferent motor fibres to skeletal muscles, while the dorsal roots contain afferent sensory fibres. These roots combine to form the spinal nerve, which then gives off the primary rami.

• The dorsal roots themselves do not supply sensation to the skin, it is the primary rami that branch from the roots that supply sensation

Aspiration pneumonitis is more common in the right lower lobe than the left. The right main bronchus is more vertically oriented and shorter than the left – as a result, it is the most likely destination for inhaled foreign material.

The diaphragm is active in both normal and increased respiration. In normal quiet breathing the diaphragm itself is the major active muscle, whereas the accessory muscles of respiration are recruited during increased work of breathing.

• The scalene muscles are accessory muscles of respiration that act to raise the first rib, and are therefore recruited during increased work of breathing. • The external intercostals are active during forced inspiration, and the internal intercostals during forced expiration. • The internal oblique muscles act during forced expiration, but do not act alone – there are other muscles that assist such as the latissimus dorsi and the anterior abdominal wall muscles. • The external intercostals raise the ribs during inspiration, as they are aligned so that their fibres run inferomedially. Their contraction therefore raises the ribs and expands the ribcage. This action is in contrast to the internal intercostals, which are active during expiration.

The dorsal scapular nerve is a motor nerve from the C5 nerve root directly that innervates the rhomboids and levator scapulas. These muscles help to retract and elevate the scapula, so these movements would be weakened following that damage.

Page | 49

The vagus nerve arises from the medulla and exits the skull through the jugular foramen. It travels in the carotid sheath between the internal jugular vein and the common carotid artery in the neck. In the chest it lies posterior to the hilum of the lung, while the phrenic nerve travels anterior to the hilum. The left vagus travels on the anterior aspect of the oesophagus, while the right one travels on the posterior side. Both left and right nerves enter the abdominal cavity at the level of T10 (oesophageal hiatus).

The fossa ovalis (highlighted in green in the image) is found on the interatrial septum, which forms the posteromedial wall of the right atrium. It is formed by the closure of the foramen ovale at birth, when the septum primum and septum secundum fuse.

Musculi pectinati are located in the anterior part and the auricle of the right atrium (the embryological remnant of the true atrium). The posterior wall of the right atrium is smooth, and is derived from the sinus venarum. The right atrium itself forms the right border of the heart between the superior vena cava and IVC. The right atrium can be seen opened from the front below – the auricle and its musculi pectinati within can be seen highlighted in red.

Endoderm is one of the three embryonic germ-cell layers (the others being ectoderm and mesoderm). It forms the epithelium of the gastrointestinal tract, respiratory system, auditory system (eg. Tympanic membrane), and urinary system.

• The adrenal cortex is derived from mesoderm, and the medulla from ectoderm. • The dermis of the skin forms from mesoderm. • The gonads develop broadly from intermediate mesoderm. • The lens of the eye is ectodermal in origin, from the lens placode

The phrenic nerves run over the anterior of the scalenus anterior muscle

Embryology Features of the VACTERL association V Vertebral defects

A Anal atresia

Page | 50

C Cardiac defects

T Tracheo-esophageal fistula

R Renal abnormalities

L Limb abnormalities During the 4th gestational week the respiratory diverticulum appears as a ventral outgrowth of the foregut endoderm. As it grows, longitudinal folds form to become tracheaoesophageal ridges that progress to allow separation of the trachea and oesophagus. Failure of this process can lead to trachea- oesophageal fistulas, which are associated with the VACTERL group.

• The mesoderm develops into the skeletal and muscular systems, the dermis of the skin and the reproductive system (aside from the germ cells). • The ectoderm forms the skin cells of the epidermis, the nervous system, adrenal medulla and tooth enamel among other parts of the adult body.

Bockdalek Hernia; Examination findings include: barrel chest scaphoid-shaped abdomen respiratory distress (cyanosis, grunting, retractions) poor air entry on auscultation of the ipsilateral side, medial displacement of heart sounds. Diagnosis occurs with a chest X-ray. Initial management requires endotracheal intubation and ventilation to support the respiratory system and passing a nasogastric tube to decompress the abdominal contents and thus reduce compression of the lung. Definitive repair is via surgery, usually within the first week of life, with reduction of the abdominal viscera and repair of the diaphragmatic defect. The other type of congenital diaphragmatic hernia is the Morgagni hernia that occurs anteriorly and is less severe. Use the ‘5 Bs’ mnemonic to remember the differences between the two: Bochdalek hernias are Bad, Big, occur in Babies and are found at the Back of the chest (posterior).

The endocardial cushions are masses of mesenchymal tissue that form in the atrioventricular and conotruncal regions of the bulbus cordis, one dorsal and one ventral. These grow towards each other and fuse to form the atrial/ventricular septa, the atrioventricular canal and valves. The septae form between the 27th to 37th day of development. Failure of the endocardial cushions to fuse gives rise to atrioventricular septal defects

Left atrial pressure rises at birth because of the increasing amount of blood returning to the heart from the pulmonary circulation.

• During fetal development a patent foramen ovale is required so that blood can flow from the right to the left atrium. This provides oxygenated blood to the systemic circulation. • Blood flow is facilitated from the right atrium to the left atrium. The foramen ovale is replaced by the interatrial septum at birth. • The right atrial pressure actually drops, which facilitates the closure of the foramen ovale. This occurs because oxygen filling the alveoli causes pulmonary arterioles to open, reducing right atrial pressure.

Page | 51

• Oxygenated blood enters the heart from the pulmonary veins after birth. During development in the womb oxygenated blood from the inferior vena cava (IVC) crosses the patent foramen ovale to supply the systemic circulation.

Oesophageal atresia is a congenital disorder occurring in 1 in 2500 live births. It describes a failure of the oesophagus to form as a continuous tube but rather forms as a blind ended pouch. It is part of the VACTERL syndrome, and can be-associated with a tracheoesophageal fistula (TEF). It arises in the 4th week of gestation when the oesophagus and trachea normally separate. A fetus with oesophageal atresia cannot swallow amniotic fluid effectively, especially in the absence of TEF. Swallowing reduces the amount of amniotic fluid present and with impaired swallowing polyhydramnios develops in almost all mothers carrying foetuses with oesophageal atresia without TEF. Symptoms of oesophageal atresia in the neonate include: cyanosis when feeding, drooling, poor feeding, and gagging on attempted feeding. Polyhydramnios is a condition defined as an excessive amount of amniotic fluid around the fetus. This can be detected during antenatal ultrasound screening and is alsoassociated with the following conditions: duodenal atresia, cardiovascular defects, neural tube defects, renal disorders, trisomies, maternal diabetes, fetal anaemia and congenital infections such as parvovirus/toxoplasmosis/rubella/CMV.

Anencephaly is described as a gross malformation of the brain and cranial vault with reduced or absent cerebrum and cerebellum, affecting 1 in 10 000 pregnancies. It is part of the neural tube defects, and is secondary to failure of neural tube closure in the 3rd to 4th week of development. It can be detected antenatally via an ultrasound (US) scan or an elevated maternal serum a-fetoprotein. It is non- compatible with life, given the severe abnormalities, even if born alive death in unavoidable. Folic acid is used pre-conception and antenatally to reduce the incidence of neural tube defects. Anencephaly is associated with other congenital malformations most commonly spina bifida, cleft lip/palate, gastroschisis and genitourinary anomalies. It is not associated with oesophageal atresia.

Oligohydramnios is defined as diminished amniotic fluid. This is detected during routine antenatal ultrasound scans and can be due to the following:

1. placental causes: placental abruption, twin to twin transfusion syndrome 2. fetal factors: renal agenesis, chromosomal/congenital factors, IUGR, premature rupture of membranes, intrauterine fetal death 3. maternal factors: uteroplacental insufficiency, HTN, pre-eclampsia, chronic hypoxia 4. drug induced: NSAIDs, angiotensin-converting enzyme (ACE) inhibitors 5. idiopathic

In utero, the ductus arteriosus allows oxygenated blood to pass from the pulmonary artery to the aorta, and then to the fetal circulation. Fetal blood is oxygenated from the placenta via the umbilical arteries.

• In normal development the ductus arteriosus narrows within 12–24 h, but does not completely close until 2–3 weeks of life. PDA can cause presentations similar to the scenario in the question within the first year of life. If left untreated it can reverse from a left–right shift to a right–left shunt, also known as Eisenmenger’s syndrome. This can progress to pulmonary hypertension.

Page | 52

• It is the foramen ovale in the developing fetal heart that facilitates oxygenated blood flow from the right to the left atrium. • The ductus arteriosus becomes the ligamentum arteriosum within three weeks after birth.

The cloaca is an embryonic structure that gives rise to the urogenital sinus anteriorly and the anorectal canal posteriorly. The former will form the larger part of the bladder, the urethra and genital tubercle, whereas the latter will form the rectum and anal canal. A persistent cloaca describes a congenital condition where the urethra, vagina and rectum empty into one common channel.

• The urachus is a fibrous remnant of the allantois, connecting the fetal bladder to the yolk sac and thus removing waste products. The urachus will become the median umbilical ligament. • The Müllerian or paramesonephric ducts are they embryological precursors of the female reproductive system, giving rise to the uterus, fallopian tubes, cervix and upper two-thirds of the vagina. In men, the Müllerian ducts regress due to the influence of the antiMüllerian hormone. • The Wolffian or mesonephric ducts are the embryological precursors of some of the man reproductive structures. Under the influence of testosterone they form the epididymis, vas deferens and seminal vesicles. In the female the mesonephric ducts regress, as there is no testosterone

The midgut and hindgut are in continuity. The midgut rotates around the superior mesenteric artery (SMA). The hindgut moves in a more simplistic way during development; as the midgut returns to the abdominal cavity, it in turn moves to the left on the hinge of its mesocolon. As a result of this movement, the vessels that supply it are anterior to other posterior abdominal wall constituents.

Omphalocele (exomphalos major, >4 cm defect) a congenital defect of the abdominal wall affecting 1 in 2500 live births. During normal development of the abdominal viscera, by the 6th week of embryonic development, the rapid growth of the liver and the intestines causes the midgut to herniate through the umbilical ring. By the 10th week, the abdominal cavity has grown enough to accommodate the organs and the midgut returns into the abdomen, with rotation and fixation of the duodenum and colon. In exomphalos (omphalocoele) there is failure of the return of the midgut in the abdominal cavity and the intestine remains herniating into the umbilical cord. The herniating sac can contain bowel and liver and is surrounded by a peritoneal membrane.

• Exomphalos major is associated with a >4 cm abdominal wall defect, whereas exomphalos minor is associated with a less than 4 cm defect. • Exomphalos major is associated with congenital heart defects (such as tetralogy of Fallot) in 50% of cases, limb abnormalities, ectopia cordis, bladder exstrophy. • Exomphalos minor is less commonly associated with associated anomalies. • Diagnosis is via routine antenatal ultrasound and can be suspected with a raised maternal serum afetoprotein.

During the 5–7th week of embryonic development, the cloaca will divide into the urogenital sinus (anterior) and the anal canal (posterior). The former will form the larger part of the bladder, the urethra and genital tubercle, whereas the latter will form the rectum and anal canal. Hypospadias, is a birth defect presenting in about 1 in 300 male live births. It occurs due to incomplete fusion of the urethral folds leading to a displaced urethral meatus (opening), not at the tip of the penis, but somewhere along the inferior of the shaft of the penis.

Page | 53

There are two types of hypospadias:

1. Anterior or Distal = The meatus is found on or near the glans. This is the most common type, also referred to as glanular hypospadias. 2. Posterior or Proximal = The meatus is found near or within the scrotum.

Diagnosis is clinical with a displaced urethral opening and incomplete foreskin. Symptoms include: difficult urination, abnormal stream, or curvature of the penis. It is corrected surgically. It is commonly associated with undescended testes. Diagnosis is by its characteristic appearance of the penis with a displaced urethral opening, incomplete foreskin. In mild cases it can be asymptomatic. When symptomatic it can cause difficulty in urination, abnormal stream or curvature of the penis. It is not associated with bilateral hydronephrosis. Minor abnormalities do not require treatment. In symptomatic patients the treatment is surgical, by distal urethral repositioning and glanuloplasty.

Posterior urethral valves are a form of congenital obstruction secondary to malformation of the posterior urethra. They occur in 1 in 5000 live births. In the posterior urethra, valves occur by one of four proposed mechanisms: hypertrophy of the urethral mucosal folds, abnormal development of the Wolffian duct, fusion of the verumontanum or persistence of the urogenital membrane (a structure that normally vanishes). All the above cause obstruction. They are usually diagnosed antenatally by evidence of bilateral hydronephrosis, thickened bladder wall and bladder diverticula. It is the most common cause of bladder outlet obstruction in male newborns. After birth there is bladder distention, dribbling urine or complete retention. Management of the newborn is with a urethral catheter to relieve the obstruction followed by vesicotomy or valve ablation. Patients may require a cutaneous ureterostomy to ensure urine drainage.

Anterior urethral valves are rare congenital anomalies in the form of diverticula that typically occur when there is a defect in the corpus spongiosum. This leaves a thin walled urethra that balloons out during voiding. Symptoms mimic those of a posterior urethral valve, but anterior urethral valves are rarer. They can present with recurrent urinary tract infections, dribbling, or a weak stream. Diagnosis is with voiding studies showing a dilatation in the urethra. Ultrasound scan shows evidence of hydronephrosis, incomplete voiding, distended bladder.

Imaging studies can show evidence of hydronephrosis. They are found in children and can lead to lower urinary tract obstruction. They are treated with endoscopic or open ablation of the urethral diverticulum.

Bladder exstrophy does not present with hydronephrosis. This is a rare, anterior midline defect of the infraumbilical abdominal wall affecting the pelvis, abdominal muscles, urinary tract and external genitalia. It presents as incomplete closure of the lower abdominal wall at the midline and therefore protrusion of the bladder through the defect. Typical examination findings are low set umbilicus, a red patch of mucosa freely draining urine, wide pubic symphysis, open urethra (men), absent mons pubis and bifid clitoris (women). Treatment is with staged operations: after birth (best within 72 h): operation to close the bladder and pelvis 2 years: genitoplasty (repair of epispadias and penis) 4–5 years: bladder neck repair to achieve continence.

Vesicoureteric reflux is retrograde flow of urine from the bladder into the kidneys that can lead to infection, scarring and kidney damage. Primary vesicoureteric reflux is present at birth and is caused by

Page | 54 a defect in the valve that normally prevents backflow of urine. It is associated with recurrent urine infections and pyelonephritis leading to kidney damage. Treatment is aimed at preventing kidney damage. If severe, it can lead to hydronephrosis. Diagnosis is via ultrasound of the kidneys and bladder as well as a voiding cystourethrography. Even though both anterior and posterior urethral valves can cause bilateral hydronephrosis, posterior urethral valves are the most common form of male newborn obstruction.

The hydatid of Morgagni, also called the appendix testis, is the remnant of the Müllerian duct found in 90% of men. It is on the upper pole of the testis, adjacent to the tunica vaginalis. It has no function but carries the risk of torsion presenting with unilateral testicular pain. Diagnostically, patients present with a tender ‘blue dot’ discoloration of the testis. Torsion of the hydatid of Morgagni does not affect testicular function, but may require surgical excision for symptomatic relief.

• The Gartner’s duct is derived from the remnants of the mesonephric ducts in the women. It persists between the layer of the broad ligament and the wall of the vagina. It can give rise to a cyst.

In men the mesonephric ducts give rise to the tubular system of the genito-urinary system, ie epididymis, vas deferens, ureter, renal pelvis, calyces, collecting tubules, seminal vesicle and ejaculatory duct. In women they only contribute to the tubular system of the urinary system leading to the formation of the ureter, renal pelvis, calyces and collecting tubules

Bicornuate uterus is often discovered incidentally. It occurs when there is incomplete or partial fusion of the paramesonephric (Müllerian) ducts, and results in a uterus with two separate horns. It is associated with recurrent pregnancy loss and cervical incompetence.

• Unicornuate uterus is a congenital abnormality in which the uterus has a single horn. It occurs when only one paramesonephric duct fails to develop, but not with partial fusion. It is associated with miscarriage.

The bulbourethral glands form during the 10th to 12th week of gestation.

• The bulbourethral glands are found only in men. Their equivalent in women are the Bartholin’s glands, found on either side of the vaginal opening. • The bulbourethral glands are exocrine glands, because their main function is to secrete lubrication to the spongy urethra. They also produce prostate-specific antigen (PSA). They are found inferior to the prostate and posterolaterally to the membranous urethra. • These glands do arise from the urogenital sinus, which is the ventral part of the embryonic cloaca. It is divided into upper, pelvic and phallic sections. Also arising from the urogenital sinus in men are the urethra, bladder (except trigone) and the prostate gland. • The bulbourethral glands are also known as Cowper’s glands, after the English surgeon William Cowper who first described them in the 17th century. Skene’s glands are found at the anterior wall of the vagina.

By week 8 of development the interstitial cells of Leydig develop. These produce testosterone, which is responsible for the induction of gender differentiation in the male fetus.

Page | 55

• The Sertoli cells produce Müllerian Inhibiting Factor (MIF). This factor suppresses the development of the paramesonephric duct. • It is the absence of TDF in the female that causes the gonads to develop into ovaries. • The embryonic urogenital folds form the labia minora in women, and the ventral aspect of the penis in men. The labia majora are formed from the labioscrotal swellings and are correspondent in men with the scrotum. These develop from the ectoderm layer as paired swellings on either side of the embryonic cloacal membrane. In the man they fuse to form the scrotum with the fusion line known as the scrotal raphe. In the woman they do not fuse, except for the portion that becomes the frenulum of the labia majora.

The round ligament of the uterus in the woman corresponds to the gubernaculum testis in the man, and both develop from the gubernaculum, which is derived from the mesoderm of the embryo.

• The gonads are derived from the mesoderm, not the endoderm. • From the seventh week of gestation male and female systems become more distinguishable under differing hormonal influences. • It is only from around week 9 that the genitalia are distinguishable. They are similar until week 7 and fully developed by week 9.

An undescended testicle is the most common disorder of development. It is best managed conservatively in the first instance because many undescended testicles will descend within the first 6 months. After this point few will descend.

• Evidence suggests that earlier surgery is best (3–6 months ideal, 6–12 months acceptable) to reduce the associated risks of an undescended testicle.

Similar to most developmental disorders, hypospadias is linked with many other problems. Mild, distal hypospadias carries with it a lower rate of concurrent disorder whereas proximal hypospadias has a higher rate. It is important to investigate children appropriately for other abnormalities

Although the spinal cord in an adult finishes in the upper lumbar vertebrae, the cerebrospinal fluid (CSF) within the dura passes all the way down to the sacral vertebrae. The exact termination is variable within the sacrum (s1-3).

The cranial neuropore closes before the caudal aspect at around 25 days. The caudal neuropore closes around two days after this.

• The neural plate is composed of thickened ectoderm. The neural plate appears around the beginning of the third week. The lateral edges of the plate become elevated, forming neural folds that come together to fuse the neural tube. • Neural tube fusion begins in the cervical region and extends in the caudal direction. • The central nervous system (CNS) first appears as a plate composed of thickened ectoderm, not mesoderm

Anencephaly results from the failure of the cranial neuropore to close around day 25. Spina bifida results from the failure of the caudal neuropore to close, which occurs several days later at day 27

Page | 56

Folic acid supplementation is recommended for women planning a pregnancy to decrease neural tube defect occurrence. However, it should be commenced before conception, and continued for 12 weeks. Those felt to have a higher risk of neural tube defects (family history) should take a higher dose spinal bifida in which a neural tube defect causes an opening in the vertebral column, exposing the spinal cord and meninges which protrude through the opening. It is generally seen as a fleshy protrusion and can cause paralysis, absence of sensation and bladder/bowel symptoms. As with other neural tube defects, it results from failed neural tube pore closure by 28 days during embryological development. Treatment is surgical and the brain should be imaged to assess for a Chiari 2 malformation (also known as an Arnold–Chiari malformation) and hydrocephalus.

Approximately 10 – 15% children are born with lack of fusion of neural arch. The bony defect rarely carries with it any concurrent back problems and can be identified by a commonly found tuft of hair at this location. More severe forms of failure of neural arch fusion, such as spina bifida cystica, are far more rare and come with significant associated paralysis, and bowel and bladder problems.

Syndromes associated with neural tube defects include Edward’s syndrome (Trisomy 18), Patau’s syndrome (Trisomy 13), and Meckel-Gruber and Roberts.

• maternal obesity that is associated with a two-fold increase in neural tube defects.

The 3rd pharyngeal pouch forms the inferior parathyroid glands and the thymus. By week 6 the dorsal part of the pouch forms the inferior parathyroid glands, and the ventral part forms the primordia of the thymus.

• The ultimobranchial body is formed by the 4th and 5th pharyngeal pouches, and cells from this structure migrate into the thyroid gland and differentiate into the parafollicular (C) cells of the thyroid. • The 1st pharyngeal pouch develops into the external auditory meatus and middle ear cavity. • The 2nd pharyngeal pouch forms numerous infoldings that become the crypts of the palatine tonsil.

The first pharyngeal arch, also known as the mandibular arch differentiates to the following structures:

1. artery: maxillary artery, external carotid artery 2. cranial nerve: trigeminal nerves (V2 and V3) 3. muscle component: muscles of mastication, mylohyoid, anterior belly of digastric, tensor tympani, tensor veli palatini 4. skeletal/Connective tissue derivatives: maxilla, zygomatic bone, squamous temporal bone, palatine bone, malleus, incus, sphenomandibular ligament, Meckel’s cartilage.

The muscles of facial expression are derivatives of the second pharyngeal arch, also known as the hyoid arch. The second pharyngeal arch differentiates into the following:

1. artery: stapaedial artery 2. cranial nerve: facial nerve (VII) 3. muscle component: muscles of facial expression, posterior belly of digastric, stylohyoid muscle, stapedius, platysma

Page | 57

4. skeletal/connective tissue derivatives: lesser horn of hyoid, upper half of body of hyoid, stapes, styloid process, stylohyoid ligament, part of the hyoid cartilage.

The glossopharyngeal nerve innervated the third pharyngeal arch, which differentiates into the following structures:

1. artery: common carotid artery, proximal internal carotid artery 2. cranial nerve: glossopharyngeal nerve (IX) 3. muscle component: stylopharyngeus 4. skeletal/connective tissue component: greater horn of hyoid, lower half of the body of the hyoid, thymus a branchial cleft cyst, which is typically as a result of failure of obliteration of the second branchial cleft. They account for almost 20% of neck masses in children and present as a solitary painless mass previously unnoticed until infected (e.g. respiratory infection).

On day 37 (beginning of the sixth week), a small sulcus forms above and below each eye, known as the eyelid groove. As these grooves deepen eyelid folds develop into the eyelids and cover the eye.

Closure of the eyelids is complete by the middle of the eighth week of development.

• The external parts of the ear (auricle), are distinguishable from week 6 of development. • By the eighth week of development, the upper lip is completed, not the lower lip. • At the eighth week of development the head flexure is somewhat reduced, allowing for lengthening of the neck, however the latter is not yet fully developed. • The palate forms by the palatine shelfs, that first appear in the sixth week of development and are directed obliquely towards the tongue. • In the seventh week they elevate and become horizontal. They fuse with each other at the midline and with the primary palate at the incisive foramen, forming the complete palate by 10 weeks of development.

Medulloblastoma is the most common malignant brain tumour in children. It is a highly invasive tumour of neuroepithelial origin that arises in the cerebellum, predominantly from the cerebellar vermis. Approximately 70% of affected patients are diagnosed in the first 15 years of life. Patients typically present with the triad of morning headaches, vomiting and lethargy relating to raised intracranial pressure. As the tumour grows, worsening ataxia occurs mainly of the lower limbs. It can metastasise outside the central nervous system (CNS) and especially to bone.

• Intra-cranial ependymomas are most commonly found in the fourth ventricle • It is the pontine gliomas that arise in the pons, and comprise about 10–15% of paediatric brain tumours. They are extremely rare in the adult population. Surface Anatomy The anatomical landmark used to identify L4 is the top of the iliac crest. The line connecting the top of the two iliac crests, the supracristal line, passes through the spinous process of the L4 vertebra. Therefore, by finding the tops of the iliac crests, you should be able to identify L4.

Page | 58

The mid-point of the inguinal ligament lies at a point halfway between the anterior superior iliac spine and the pubic tubercle, and is the surface landmark of the deep inguinal ring. It is distinct from and lies laterally to, the mid-inguinal point, which lies halfway between the anterior superior iliac spine and the symphysis pubis and is the surface marking of the femoral pulse.

The transpyloric plane is a convenient way to relate anatomical structures. It is an imaginary transverse plane with a surface marking mid-way between the jugular notch (or suprasternal notch) and the pubic symphysis. The important anatomical relations of this site include – the origin of the superior mesenteric artery, the pancreatic neck, the first part of the duodenum, the pylorus, the renal hila, the duodenojejunal flexure, the gall-bladder fundus, the origin of the hepatic portal vein, the transverse mesocolon and the splenic hilum.

In males, the seminal vesicles lie in the rectovesical pouch, posterior to the posterior wall of the bladder. They are at risk of local invasion from posterior wall bladder tumours.

• The prostate gland is inferior to the bladder • The perineal body is a midline perineal structure that is inferior to the bladder.

The general structure of the male pelvis is heavier and thicker than the female pelvis and has more prominent bone markings. The female pelvis is wider, shallower, and has both a larger superior and inferior pelvic aperture. The ischial tuberosities are farther apart in the female pelvis because of the wider pubic arch, and the sacrum is less curved in the female pelvis. In addition to this the obturator foramina are round in the male and oval in the female. The adaptations to the female pelvis compared with the male largely serve to ensure a wide and safe birth canal.

The female pelvis is wider and shallower than the male pelvis.

The ureter descends towards the pelvis anterior to the psoas major muscle but posterior to the gonadal artery. It then crosses anterior to the bifurcation of the common iliac artery at the level of the pelvic brim before descending posteroinferiorly along the lateral pelvic side wall.

The uterine artery crosses anterior and superior to the ureter near the lateral portion of the fornix. This relation is of surgical importance as the ureter is in danger of being damaged at the time of a hysterectomy.

• The point of crossing of the uterine artery and ureter is approximately 2 cm superior to the ischial spine

The perineum and the external genitalia, including the scrotum and labia majora, drain to the superficial inguinal lymph nodes.

In men, the deep perineal pouch contains the membranous urethra, sphincter urethra, bulbourethral glands, deep transverse perineal muscles, internal pudendal vessels and dorsal nerves of the penis. The identified mass must therefore be within the membranous urethra, and of the available options the bulbourethral glands are the most likely structures to be invaded as they lie on each side of the membranous urethra.

• The pudendal nerve provides sensation to the perineum and external genitalia – it runs in the pudendal canal, not the deep perineal pouch.

Page | 59

Pain and temperature are carried in the spinothalamic tract; these tracts carry contralateral sensation – a right-sided injury to the spinothalamic tract would therefore cause this sensory loss. The spinothalamic tract conveys pain, temperature, crude touch and firm pressure sensations from one side of the body to the opposite side of the brain. The first neurone of the spinothalamic tract synapses in the posterior horn; the next neurone crosses to the contralateral side of the spinal cord and synapse in the thalamus, after ascending through the cord and brainstem; the third neurone arises in the thalamus to pass to the cortex. Axons from the cervical region synapse medially while axons from the lumbar region synapse laterally. As the fibres decussate in the spinal cord, a lesion of the spinothalamic tract at the level of the spinal cord would lead to loss of pain sensations on the contralateral side, beginning one level below the level of the lesion.

In current practice a CT scan is the gold standard in diagnosing odontoid peg fracture. Many hospitals include C-spine CT as standard with any head CT request, particularly in elderly patients. The odontoid peg (dens) is an upward projection from the axis (C2). The peg has an articular facet at its front and forms part of a joint with the anterior arch of the atlas. The atlanto-axial joint (C1–C2) allows rotation. It is a non-weight-bearing joint. It is supported by a number of ligaments – the alar ligaments (from the side of the peg to the occipital condyle), the apical ligament (from the tip of C2 to the anterior margin of the foramen magnum), and the transverse ligament (which arches across the atlas to maintain the position of the peg).

• Open mouth XR is the standard imaging technique used in plain imaging to look for possible peg fractures. However, with significant clinical/radiological concern, a CT scan is the investigation of choice. In a number of units, especially in the elderly, due to the availability of CT scanning, this has largely replaced plain radiographs as the initial diagnostic tool.

A left-sided occipital haemorrhage will result in a right-sided homonymous hemianopia. There may or may not be macular sparing, depending if the occipital pole is spared or not.

• Occipital lobe stroke typically results in a homonymous hemianopia contralateral to the side of the lesion. Macular or central field sparing can occur if the occipital pole remains intact through dual blood supply from a branch of the middle cerebral artery

The true vocal folds have a stratified squamous epithelium, innervated by the recurrent laryngeal branch (CN X), and are formed by the vocal ligament (the free edge of the quadrangular membrane forms the false vocal cord). The cords are adducted by the lateral cricoarytenoid muscle, abducted by the posterior cricoarytenoid and tensed by tilting the thyroid cartilage downwards and forwards by contracting the cricothyroid muscle. All the laryngeal muscles are supplied by the recurrent laryngeal nerve except for cricothyroid, which is supplied by the external laryngeal nerve.

Page | 60

Pathology The predominant bacterial flora of the oropharynx are Streptococcus viridans, coryneforms and Neisseria spp.

• Many anaerobic bacteria colonise the colon, especially Bacteroides fragilis. • The lower respiratory tract is usually sterile. • The predominant commensal bacteria of the skin is Staphylococcus epidermidis. Others include Staphylococcus aureus, micrococci, coryneforms and anaerobic cocci. Aerobic bacteria such as Escherichia coli and Enterococcus spp. are also present in large numbers. • Lactobacilli are the predominant species in the vagina. Others include Gardnerella vaginalis and anaerobes.

Pre-eclampsia is said to be present when hypertension arises in pregnancy (pregnancyinduced hypertension) in association with significant protein in the urine, usually after the 20th week of pregnancy. Its cause remains unclear, although the principal cause appears to be a substance or substances from the placenta causing endothelial dysfunction in the maternal blood vessels. While blood pressure elevation is the most visible sign of the disease, it involves generalised damage to the maternal endothelium, kidneys and liver; with the release of vasopressive factors only secondary to the original damage. Pre-eclampsia may develop at varying times within pregnancy and its progress differs among patients; most cases present pre-term. It has no known cure apart from ending the pregnancy (induction of labour or abortion). It may also present up to 6 weeks post-partum. It is the most common, dangerous complication of pregnancy and it may affect both the mother and the fetus.

Preeclampsia is diagnosed when a pregnant woman develops high blood pressure (two separate readings taken at least 6 h apart of 140/90 or more) and 300 mg of protein in a 24-h urine sample (proteinuria). Swelling or oedema, (especially in the hands and face) was originally considered an important sign for a diagnosis of pre-eclampsia, but in current medical practice only hypertension and proteinuria are necessary for a diagnosis. However, unusual swelling, particularly of the hands, feet or face, notable by leaving an indentation when pressed on, can be significant and should be investigated.

HIV is present in high titre in the blood of asymptomatic carriers, and in many asymptomatic HIV- positive patients

• Definitive diagnosis is made by western blotting following a positive enzyme linked immunosorbent assay (ELISA). More recently, polymerase chain reaction (PCR) techniques have been used.

An abscess is collection of pus within soft tissues and occurs when the host’s response to infection is inadequate. Predisposing factors include foreign bodies, haematoma formation and poor blood supply. An abscess contains bacteria, acute inflammatory cells, protein exudate and necrotic tissue and is surrounded by granulation tissue (the ‘pyogenic membrane’). The organisms usually involved cause:

• superficial abscesses – Staphylococcus aureus, Streptococcus pyogenes • deep abscesses – Gram-negative species (eg Escherichia coli) and anaerobes (eg Bacteroides).

Page | 61

Pseudomonas aeruginosa is a Gram-negative infection and an important cause in the immunocompromised. It is common in patients who have had a prolonged hospital stay

Keloids commonly affect children and young adults, and they undergo rapid growth during puberty and increase in size during pregnancy.

• Keloids extend beyond the original scar margins, while hypertrophic scars are confined to the borders of the original wound. • Keloids are dermoproliferative disorders unique to humans. The aetiology of keloids is unclear, although various theories including familial tendency such as an autosomal-dominant or - recessive inheritance, hormonal influence, altered immunological response, enhanced role of transforming growth factor-β, abnormality of keratinocyte control over fibroblasts, and downregulation of apoptosis-related genes have been purported. Factors such as haematoma, infection and wound dehiscence predispose to hypertrophic scar formation (not keloids). • Collagen synthesis is three times higher in keloids than in hypertrophic scars, and 20 times higher in keloids than in normal skin. The absolute amount of collagen is also increased in keloids, indicative of increased collagen synthesis or decreased collagen degradation. • Hypertrophic scars generally develop within weeks of injury, whereas keloids can develop up to 1 year later. • Keloid scars are 15 times more common in patients with darkly pigmented skin

Amyloidosis is a disorder in which extracellular proteinaceous substances (protein fibrils) are deposited in body tissues, locally or systemically. Amyloidosis may be of primary or secondary aetiology, and is linked with chronic disease conditions that are usually inflammatory (rheumatoid arthritis, Crohn’s disease) or caused by infections (tuberculosis); it is also associated with multiple myeloma, Hodgkin’s disease and Waldenström’s macroglobulinaemia. Other forms of amyloidosis are familial and isolated (as in Alzheimer’s disease).

Actinomycosis bacterial infections cause pus containing sulphur granules. It is a rare bacterial infection.

• Actinomycosis is a Gram-positive, non-acid-fast, anaerobic to microaerophilic bacterial infection.

An undesirable side-effect of chemotherapy used to treat malignancies is the destruction of T cells, which play a key role in the development of immunity against viral infections. This is especially true for viral diseases in which the causative agent remains dormant in the body. Varicella-zoster is an excellent example of such a virus and flare-ups of varicella-zoster infections are well known occurrences in cancer patients who receive chemotherapy. The varicella-zoster virus is a single entity. It is a medium-sized (100–200 nm), double-stranded DNA virus of the herpesvirus group, with only one serological type.

There are around 50 types of human papillomavirus (HPV). The virus is associated with anal and cervical cancer. The human papillomavirus (HPV) vaccine is now issued to all girls between the ages of 12 and 18.

Smoking is a risk factor for deep vein thrombosis (DVT).

• Nicotine increases myocardial oxygen demand via an increase in heart rate and blood pressure.

Page | 62

• Smoking cessation for 12 h pre-operatively results in a significant decrease in carboxyhaemoglobin levels

Liquefactive necrosis is characteristic of focal bacterial, or occasionally fungal, infections, because microbes stimulate the accumulation of inflammatory cells. Hypoxic death of cells within the central nervous system evoke liquefactive necrosis. Although the reasons for this are obscure, it is thought to happen due to the reduced structural framework of neural tissues. Whatever the pathogenesis, liquefaction completely digests the dead cells. The end result is transformation of the tissue into a viscous liquid mass. If the process was initiated by acute inflammation the material is frequently creamy yellow in colour because of the presence of dead white cells and this is called ‘pus’.

• Coagulative necrosis is a type of cell death caused by ischaemia or infarction. It is important to note that while ischaemia in most tissues of the body will cause coagulative necrosis, in the central nervous system it causes liquefactive necrosis, as there is very little structural framework in neural tissue. • Lacunar infarcts are small infarcts in the deep cerebral white matter, basal ganglia or pons. They are thought to occur due to occlusion of a single small perforating artery.

Blastomycosis is a pulmonary and occasionally a haematogenous fungal infection caused by the organism Blastomyces dermatitidis. Endemic in parts of North America, blastomycosis causes clinical symptoms similar to histoplasmosis. Infection occurs by inhalation of the spores of the fungus from its natural soil habitat. Once inhaled into the lungs, the fungus multiplies and can disseminate through the blood and lymphatics to other organs, including the skin, bone, genitourinary tract and brain. The incubation period is 30–100 days, although infection can be asymptomatic.

Staphylococcus aureus produces haemolysins, fibrinolysin, hyaluronidase (may assist spread of infection), leucocidin (destroys polymorphonuclear leukocytes), coagulase, enterotoxin (heat-stable proteins causing vomiting), toxic-shock syndrome toxin (related to enterotoxins) and epidermolytic toxins (cause splitting of epidermis and blister formation).

MRSA is Gram-positive coccus. Over the last 20 years the prevalence of MRSA in hospitals has fluctuated and is now found in around 50% of hospitals in the UK. The mecA gene encodes an additional penicillin binding protein (PBP), which confers methicillin resistance. It classically grows on culture medium in clusters as opposed to streptococci, which grow in chains. The routine and consistent use of handwashing and/or alcohol gels is the most effective preventative measure, as most transmission is skin to skin.

In uterine growth during pregnancy, both cell proliferation involving the endometrial glands and muscle enlargement of the uterine wall occur. These processes offer models of both hyperplasia and hypertrophy. When both are present, DNA synthesis is markedly accelerated. Hyperplasia is an increase in the number of cells, whereas hypertrophy is an increase in cell size

• Breast tissue enlargement resulting from hormonal influences is due solely to an increase in cell numbers (hyperplasia). • In athletes, enlargement of skeletal muscle occurs through hypertrophy. • In cardiac muscle, an increase in cell size (hypertrophy) is seen in response to volume overload or systemic hypertension.

Page | 63

The spleen plays an important role in the removal of dead and dying erythrocytes and in the defence against microbes. Removal of the spleen (splenectomy) leaves the host susceptible to a wide array of pathogens, but especially to encapsulated organisms. Certain bacteria have evolved ways of evading the human immune system. One way is through the production of a ‘slimy’ capsule on the outside of the bacterial cell wall. Such a capsule resists phagocytosis and ingestion by macrophages and neutrophils. This allows them not only to escape direct destruction by phagocytes but also to avoid stimulating T-cell responses through the presentation of bacterial peptides by macrophages. The only way that such organisms can be defeated is by making them more palatable by coating their capsular polysaccharide surfaces in opsonising antibody. The production of antibody against capsular polysaccharide primarily occurs through T-cell-independent mechanisms. The spleen plays a central role in both the initiation of the antibody response and the phagocytosis of opsonised encapsulated bacteria from the bloodstream. This helps to explain why following a splenectomy the host is most susceptible to infection by encapsulated organisms, notably Streptococcus pneumoniae (pneumococcus), Neisseria meningitidis (meningococcus) and Haemophilus influenzae.

The internal organs of the body are free of commensal bacteria, apart from the alimentary tract, upper respiratory tract and oropharynx and genital tract.

The skin, Oropharynx, external auditory meatus and conjunctiva are colonised with commensals.

Staphylococci are aerobic Gram-positive cocci. The presence of the coagulase enzyme is a test to subdivide the Staphylococcus micro-organisms. Staphylococcus epidermidis is coagulase negative.

• Staphylococci are aerobic Gram-positive cocci. Staphylococcus aureus is coagulase positive • Streptococci are aerobic Gram-positive cocci. Haemolytic groups are used to subdivide streptococci. Streptococcus viridans is an a-haemolytic Streptococcus. • Enterococcus faecalis is an anaerobic microorganism.

Streptococcus pyogenes belongs to Lancefield group A. It is associated with rheumatic fever, carditis and glomerulonephritis as immunological sequelae. The Lancefield classification is based on the composition of bacterial antigens found the organisms’ cell walls.

• Streptococcus spp. produces streptolysins O and S, hyaluronidase, streptokinase, leukocidin and an erythrogenic toxin that causes scarlet fever. • Staphylococci are found in clusters; streptococci are found in pairs or chains

The mode of action of ultraviolet light on micro-organisms is related to its absorption by the DNA, formation of pyrimidine dimer. This absorption leads to the formation of covalent bonds between adjacent pyrimidine bases. These pyrimidine dimers alter the form of the DNA and so interfere with normal base pairing during the synthesis of DNA.

• Removal of free sulfhydryl groups is caused by heavy metals such as soluble salts of mercury. • The addition of alkyl groups to cellular components is caused by ethylene oxide or formaldehyde. • Disruption of the bacterial cell membrane is caused by detergents. • Protein denaturation is commonly caused by heat or alcoholic compounds.

Verotoxin is produced by the enterohaemorrhagic Escherichia coli and can cause haemolytic uraemic syndrome and is similar to the shiga toxin produced by Shigella dysenteriae.

Page | 64

• Exotoxin A is associated with Pseudomonas aeruginosa. • Staphylococcus aureus, Streptococcus pyogenes and Clostridium perfringens produce hyaluronidase. S. aureus utilises hyaluronidase to break down the connective tissue matrix. • Toxic-shock syndrome toxin is a superantigen produced by Staphylococcus aureus causing toxic- shock syndrome. • Neurrotoxins are produced by bacteria such as Clostridium botulinum.

Infectious mononucleosis (or glandular fever) is caused by the Epstein–Barr virus (human herpes virus 4, HHV-4). A penicillin will cause a rash in infected patients. It can cause splenomegaly, haemolytic anaemia and hepatitis (subsequently jaundice).

Tumours derived from all three cell layers are called ‘teratomas’. In a newborn infant, the most common location is the sacrococcygeal area. Overall, however, a haemangioma is the most common tumour in infancy and most of these regresses over time.

Urinalysis demonstrates the specific gravity of urine as well as the presence of nitrites, haemoglobin, leucocyte esterase, protein, glucose and ketones.

• Urine myoglobin can be detected in conditions such as rhabdomyolysis but would not be identified on urine dipstick. It may be detected as blood on the dipstick.

This CT scan shows peri-pancreatic stranding, indicating pancreatitis. Pancreatitis is associated with low serum calcium, not raised calcium. Hypocalcaemia is due to saponification of peri-pancreatic fat. This is induced by the release of amylase from the injured pancreas

The predominant cells types in chronic inflammation are macrophages and lymphocytes.

• Neutrophils are the predominant cell type in acute inflammation and are recruited to the site within minutes following trauma.

Page | 65

Autophagic vacuoles appear to form when breakdown of intracellular components and organelles occurs. Separation of damaged cell substances in the form of autophagic vacuoles appears to be an adaptive response in atrophy.

Numerous organisms can cause infection and abdominal pain in HIV-infected individuals. The well documented organisms include: cryptosporidia, cytomegaloviruses, Mycobacterium tuberculosis; Salmonella, Shigella and Campylobacter spp., Neisseria gonorrhoeae, Treponema pallidum, Mycobacterium avium-intracellulare, Listeria monocytogenes, Entamoeba histolytica, Giardia lamblia, Isospora belli, Candida albicans, Histoplasma spp. and herpes simplex viruses

• Infectious causes of abdominal pain in HIV-infected individuals include CMV • Cryptococcus infection is one of the most common opportunistic infections in patients with HIV with most presenting with central nervous system (CNS) manifestations. Signs and symptoms include headache, fever, malaise, visual disturbance and cranial neuropathies.

Endotoxins are lipopolysaccharides forming integral parts of the cell walls of Gram-negative bacteria. Interleukin-1 and tumour necrosis factor which are released by macrophages, mediate most of the biological effects of endotoxins. Bradykinin release causes vasodilatation, increased vascular permeability and hypotensive shock. The activation of the alternative pathway of complement cascade mediates inflammation. The activation of Hageman factor triggers the coagulation cascade and release of vasoactive kinins. They differ from exotoxins, which are excreted by living bacteria. The baroreflex provides a rapid negative feedback loop in which an elevated blood pressure reflexively causes the heart rate to decrease and also causes blood pressure to decrease. Decreased blood pressure decreases baroreflex activation and causes heart rate to increase and to restore blood pressure levels

• There is a decrease in plasma volume in acute dehydration leading to an inhibition of the baroreceptors and a lower firing rate.

Actinomycosis is usually caused by Actinomyces israelii and is frequently misdiagnosed as a neoplasm. In health, A. israelii colonises the vagina, colon and mouth. Infection is established first by a breach of the mucosal barrier through various procedures (eg dental, gastrointestinal), aspiration, or pathological conditions such as diverticulitis. The chronic phase of this disease is also known as the ‘classic phase’ because the acute early phase is often missed. This is characterised by slow contiguous growth that ignores tissue planes and forms a sinus tract that can spontaneously heal and recur, leading to a densely fibrotic lesion. This lesion is often described as ‘wooden’. Sulphur granules form in a central purulence surrounded by neutrophils. This conglomeration of organisms is virtually diagnostic of A. israelii.

o Oral–cervicofacial disease is the most common form of actinomycosis. It is characterised by a painless ‘lumpy jaw’. Lymphadenopathy is uncommon in this form of the disease. o Another form of actinomycosis is thoracic disease, which is often misdiagnosed as a neoplasm, as it forms a mass that extends to the chest wall. It arises through aspiration of organisms from the oropharynx. Symptoms include chest pain, fever and weight loss. o Abdominal disease is another manifestation of actinomycosis. This can lead to a sinus tract that drains to the abdominal wall or the perianal area. o Pelvic actinomycosis is often caused by intrauterine devices. Symptoms include fever, abdominal pain and weight loss.

Page | 66

Patients who have undergone splenectomy are 60 times more likely to develop overwhelming infection than the normal healthy population. Sepsis is more common in patients after splenectomy for conditions that are partially improved by the procedure eg thalassemia and rare in patients after splenectomy for trauma or hereditary spherocytosis. The sepsis is more common in younger patients and is usually of insidious onset, pursues a fulminant course and may progress to death in 24–36 h, with a 50–80% mortality. Streptococcus pneumoniae is the causative organism in 55% of cases. Sepsis may be prevented by vaccination, preferably administered before surgery. Serious infections are most likely to occur within the first 3 years post splenectomy. It is now recommended that regular amoxicillin should be given as prophylaxis until the age of 15 (not lifelong) after which it should be commenced at the first sign of febrile illness.

Wound healing is the process by which damaged tissue is restored, as closely as possible, to its normal state. The steps in wound healing include:

o haemostasis (first few minutes), o inflammation (0–3 days), o proliferative (3 -5 days), o maturation and remodelling (5 days to 6 months).

The various cell types which take part in wound healing and time of appearance in wound as follows:

o Platelets: Immediate o Neutrophils: 0–1 day o Macrophages: 1–2 days o Fibroblasts: 2–4 days o Myofibroblasts: 2–4 days o Endothelial cells: 3–5 days.

The C3b is a protein fragment released from complement component C3. It serves as an effective opsonin. Soluble C3 convertase, also known as ‘iC3Bb’, catalyses the proteolytic cleavage of C3 complement component into C3a and C3b as part of the alternative complement system. C3b can then bind to microbial cell surfaces within an organism’s body. This can lead to the production of surface- bound C3 convertase and so more C3b components. Also known as ‘C3bBb’, this convertase is similar to soluble C3 convertase, except that it is membrane bound. Alternatively, bound C3b can help in the opsonisation of the microbe by macrophages. Complement receptor-1 (CR1) on macrophages allows the engaging of C3b-covered microbes.

In healing by secondary intention, the wound closes by both wound contraction, granulation and epithelialisation.

o Primary healing (healing by first intention) occurs when a wound is closed within 12–24 h of its creation as in a clean surgical incision or a clean laceration.

Pyogenic abscesses account for 75% of liver abscesses in developed countries. Most pyogenic liver abscesses are secondary to coliforms such as E. coli and Klebsiella. The most common cause is cholangitis associated with biliary stones followed by diverticulitis. Most are polymicrobial with Gram- negative aerobic and anaerobic organisms predominating.

Page | 67

o Worldwide, amoebic abscesses are the commonest cause of liver abscess with 10% of the world’s population infected with Entamoeba histolytica

Cryptosporidium parvum is one of several species that cause cryptosporidiosis. C. parvum infection is a protozoal infection that causes an acute diarrhoea in immunocompromised patients. In HIV infection, it can cause a watery diarrhoea, which can be associated with anorexia, nausea, vomiting and abdominal pain. There is no effective treatment (other than supportive treatment) for this infection apart from paromomycin antibiotics, this treatment can have a limited effect on the diarrhoea.

o Mycobacterium avium complex typically affects individuals who are immunocompromised but is primarily a pulmonary pathogen. It generally presents with a productive cough, weight loss, fever and lethargy o Toxoplasma gondii causes toxoplasmosis that in immunosuppressed patients such as those with HIV can be rapidly fatal. In 50% of cases it involves the central nervous system causing seizures and focal neurological deficits. It does not typically result in diarrhoea.

Cardiac effects of hypokalaemia are usually minimal until plasma potassium levels are less than 3 mmol/l. Hypokalaemia leads to sagging of the ST segment, depression of the T-wave and elevation of the U-wave. With marked hypokalaemia, the T-wave become progressively smaller and the U-wave becomes increasingly larger. Sometimes, a flat or positive T-wave merges with a positive U-wave, which can be confused with qT prolongation. Hypokalaemia can also cause premature ventricular and atrial contractions, ventricular and atrial tachyarrhythmias and second- or third-degree atrioventricular block. Such arrhythmias become more severe with increasingly severe hypokalaemia; eventually, ventricular fibrillation can occur. Patients with significant pre-existing heart disease and/or those receiving digoxin are at risk of cardiac conduction abnormalities, even with mild hypokalaemia

ECG changes in hyperkalaemia include:

o QRS widening o fusion of QRS-T o loss of the ST segment o P waves widened and of low amplitude o tall tented T-waves.

At potassium concentrations of >7.5 mmol/L atrial and ventricular fibrillation can occur.

Hyperthecosis is the presence of nests of luteinised theca cells in the ovarian stroma due to differentiation of the ovarian interstitial cells into active luteinised stromal cells. The result is greater production of androgens and women typically present with hirsutism and virilisation.

An increase in coronary artery p(CO2 ) results in coronary vasodilatation and so increased coronary blood flow.

o ADH/vasopressin causes coronary vasoconstriction resulting in decreased coronary blood flow o Serotonin has been shown to result in coronary vasodilatation in normal endothelium but when the endothelium is damaged to have a vasoconstricting effect.

SVC obstruction results in a build-up of venous return despite collateral pathways, resulting in a raised JVP.

Page | 68

o Tricuspid stenosis results in a slow y-descent of the JVP complex but not an elevated JVP.

Abdominal aortic aneurysm; Mycotic aneurysms although rare can be caused by infections such as Staphylococcus aureus, Salmonella and Streptococcus

1% of hydatid disease can disseminate to the brain and present with seizures due to spaceoccupying effect of these cysts.

o There is no known association of hydatid disease to hepatocellular carcinoma. o Rupture of hydatid cyst can present with peritonitis. o The adult, mature worm resides in the small intestine of a definitive host, which includes dogs and other carnivores o Eggs are released that are passed in the faeces of the host and ingested by an intermediate host, such as humans and other mammals. Human infection with E. granulosus frequently occurs in early childhood by direct contact with infected dogs or by eating uncooked or improperly washed vegetables contaminated with canine faeces.

Lipoprotein lipase is an enzyme that hydrolyses lipids in lipoproteins, like those found in chylomicrons and very-low-density lipoproteins (VLDL), into three fatty acids and one glycerol molecule. It requires apoC-II as a co-factor. LPL deficiency leads to hypertriglyceridaemia.

o Apolipoprotein B100 plays a role in cholesterol transport around the body. Genetic mutations in apoB-100 are associated with familial hypercholesterolaemia o HMG–CoA reductase is an enzyme that catalyses the reaction of acetyl-CoA to mevalonate, as part of the pathway for cholesterol production. Statins are HMG–CoA reductase inhibitors and serve to lower cholesterol production by inhibiting this pathway. o Disorders of LDL receptors result in familial hypercholesterolaemia

Complications of C. difficile infection include electrolyte disturbances, paralytic ileus and, if pancolitis develops, toxic megacolon, endotoxic shock and bowel perforation. Clostridium difficile infection is usually detected by identification of the cytotoxin. Gram staining of faeces is unhelpful in detecting the organism as it cannot distinguish C. difficile from many of the other gut organisms.

o The two principal therapies are vancomycin and metronidazole. For severe cases, usually a combination of oral vancomycin and intravenous metronidazole are used. o A single dose of oral metronidazole is unlikely to adequately treat C. difficile infection.

Sjögren's syndrome is a systemic chronic inflammatory disorder characterised by lymphocytic infiltrates in exocrine organs. Most common presentation is with sicca symptoms, ie xerophthalmia (dry eyes), xerostomia (dry mouth) and parotid gland enlargement. Salivary flow increases with nausea, both in patients with and without Sjögren’s.

o Although secretion of saliva is reduced during periods of sleep, it does not stop completely. o The parotid gland produces predominantly serous saliva, the sublingual glands produce mainly mucous saliva and the submandibular glands produce a mixture of the two. o Salivary glands normally produce 0.75–1.5 litres of saliva per day, in Sjögren’s syndrome this figure is much reduced and would be less than 0.75 litres per day.

Page | 69

o Due to reduced volume of saliva produced and active transport against a concentration gradient, saliva contains a higher concentration of potassium compared with plasma.

A hamartoma is composed of tissues that are normal to an area but growing in a disorganised manner.

o A teratoma is a tumour with tissue from more than one germ layer. Therefore these cells may be different to surrounding cell types.

Patients with terminal ileal Crohn’s disease are at increased likelihood of developing Cholesterol gallstones. Normally, bile acids that are excreted into the small bowel from the liver and gallbladder are reabsorbed in the terminal ileum so that they may be secreted again. This is known as the entero- hepatic circulation. So in terminal ileal Crohn’s disease this recycling of the bile salts is impaired and there is a net loss of the available bile salt pool and the body is unable to secrete significant concentrations of bile salts. Gallstones are at increasing likelihood of forming in such situations as there is an insufficient concentration of bile salts to emulsify the cholesterol and the cholesterol leaches out of solution, forming gallstones.

Myasthenia is characterised by fatigability of striated muscle. It is an autoimmune disease with antibodies against acetylcholine receptors.

The ventromedial nucleus of the hypothalamus is responsible for satiety. If destroyed, this will lead to overeating as there is nothing telling the body that it is full.

o The suprachiasmatic nucleus is responsible for circadian rhythm. o Antidiuretic hormone is synthesised in the supraoptic nuclei of the hypothalamus and is released by the posterior pituitary gland o The lateral geniculate nucleus of the thalamus is the location of synapse between the optic tract and radiation.

Stroke can cause thalamic syndrome (Dejerine–Roussy syndrome), which results in contralateral hemianaesthesia, burning or aching sensation (hyperaesthesia), often accompanied by mood swings.

o Thirst is regulated by the hypothalamus, not the thalamus.

Peyronie’s disease is a chronic inflammation of the tunica albuginea, which surrounds the corpora cavernosa of the penis. With time, fibrous tissue develops which causes the abnormal curvature and erectile dysfunction.

Benzidine is an industrial carcinogen linked with bladder cancer. Smoking is the leading cause of bladder cancer. Cyclophosphamide has been linked with an increased risk of developing bladder cancer, and more specifically an increased rate of developing muscle-invasive bladder cancer.

In Bladder carcinoma, Deletion of chromosome 9 is the most common chromosomal abnormality, along with alterations in the tumour suppressor genes p53 and Rb

People working in the rubber and dye industries, where benzene is used, are known to have a higher incidence of bladder cancer.

Seminoma is the commonest germ-cell tumour of the testes accounting for 40% of cases

Page | 70

Embryologically horseshoe kidney results from the kidneys fusing at the inferior (medial) pole having their ascent arrested by the presence of the isthmus on the inferior mesenteric artery. Horseshoe kidney is more common in men. The anatomical location and blood supply is very variable. They are more prone to infection and calculi formation due to relative upper-tract stasis. They are also more prone to trauma.

A fibrothecoma is a benign ovarian tumour. The thecoma component of the neoplasm gives the tumour a yellowish appearance because of the lipid content and can also produce oestrogen, which is responsible for the endometrial hyperplasia.

Acute renal tubular dysfunction

• It can be initially treated with osmotic diuretics, but the majority of cases will resolve with supportive treatment and careful fluid balance monitoring. • It is due to ischaemia of the medulla not the cortex.

Renal cell Carcinoma; Approximately 30% of patients have metastasis at presentation. There is a 2:1 ratio towards men.

• A left-sided renal tumour may present with a left varicocele as the left testicular vein drains into the left renal vein, which may be occluded by the tumour. • Renal cell carcinoma is a feature of von Hippel–Lindau syndrome rather than von Recklinghausen’s disease. • Approximately 90% of renal tumours are renal cell carcinomas.

Potassium levels will be increased in acute renal failure. The biochemical features of acute renal failure include a hyperkalaemia, hyponatraemia and elevated urea and creatinine. Phosphate is elevated and calcium lowered.

• Acute renal failure causes metabolic acidosis, therefore the pH will be decreased. • Plasma bicarbonate will be decreased as it will be used as a buffer for the acidosis.

The mutation or deletion of the WT1 tumour suppressor gene is responsible for nephroblastomas in more than 20% of cases Wilms’ tumour is rare (1 in 10 000) but accounts for 80% of all genitourinary (GU) tumours in children under the age of 15.

• The survival rate is generally good, ranging from 55 to 97% depending upon the tumour stage and histology. • Men and women are equally affected, 20% are familial. • Only 5% of cases are bilateral.

Diabetes can cause impotence by causing impairments in blood vessels, nerves and muscle function. Vascular impairment is a key cause of ED. Often, the first sign of a metabolic or cardiovascular condition is impairment of penile artery flow in an otherwise healthy patient. Investigating cardiovascular compromise is imperative in younger men in which diagnosing treatable conditions such as diabetes can be crucial in avoiding adverse outcomes later on in life. The main causes of erectile dysfunction are: diabetes, vascular disease, drugs, pelvic surgery, pelvic or spinal injury, neurological disease and hormonal problems. There are also psychological causes.

Page | 71

• Proton pump inhibitors (PPIs) like omeprazole can cause impotence.

Microglandular hyperplasia (endocervical polyp) occurs secondary to oral contraceptives (progestogens) and must be distinguished from adenocarcinoma histologically.

• Clear cell adenocarcinoma is a subtype of vaginal carcinoma. This is a rare tumour, often linked to pre-natal exposure to diethylstillbestrol (DES), a drug which was prescribed in highrisk pregnancies. Clear cell adenocarcinoma accounts for approximately 10% of primary vaginal malignancies, and cases are mainly diagnosed between the ages of 15 and 31. • Endocervical polyps are usually reddish-pink, measure less than 1 cm in all dimensions and are friable. They are rarely malignant. It does not seem to be associated with nulliparity or age at menopause, and its relationship to long-term use of oral contraception is still being explored. • Sarcoma botryoides, a malignant sarcoma of the vagina or other hollow mucosal-lined structures. It most likely presents before 8 years of age.

Cystine and urate stones form in acidic urine and urine alkalisation forms part of the treatment of these types of stones. Cystine stones are poorly radiopaque on X-ray but can usually be seen on CT.

• Staghorn calculi (also known as ‘struvite’) are composed of magnesium, ammonium and phosphate. They form when urine has a pH > 7. They are associated with Proteus spp. and other Gram-negative organisms that contain the enzyme urease

Punctate basophilia is seen in patients with lead poisoning, thalassaemia, and haemolytic anaemia due to pyrimidine-5-nucleotidase deficiency. It occurs when erythropoiesis is dysfunctional.

Rouleaux formation is the tendency for red cells to stack up like a pile of coins. The amount of rouleaux formation is determined by the concentration of protein in the plasma and is therefore seen in myeloma, in which blood viscosity is high

The lymphatic drainage of the oesophagus is important surgically and is divided roughly into thirds. Lymph drains to the cervical neck lymph nodes, nodes around the aorta and abdominal lymph nodes in the coeliac plexus. Lymphatic drainage follows the blood supply.

Chagas’ disease is caused by the parasite Trypanosoma cruzi, which is found in South America. The parasite Trypanosoma brucei (along with Trypanosoma gambiense) causes sleeping sickness and is found in Africa. Many patients with Chagas’ disease will present with cardiac manifestations, and these include dilated cardiomyopathy, which in turn can lead to arrhythmias and sudden death.

• Trypanosoma cruzi causes destruction of ganglionic cells at the time of infection, but also contributes to their loss over time. This progression is why the disease has two phases, an acute phase, and a chronic phase. The symptoms of the acute phase are non-specific, and Chagas’ disease may not be picked up at the time of infection. With ageing, it becomes clinically apparent as the number of ganglionic cells falls, which can lead to megacolon. When Chagas’ disease affects the oesophagus, it can mimic achalasia, and is a cause of ‘pseudo-achalasia.’ • Pseudo-achalasia is the term used to describe dilatation of the oesophagus secondary to narrowing of the distal oesophagus.

Page | 72

• True achalasia is caused by denervation of the distal oesophagus, which leads to impaired relaxation of the lower oesophageal sphincter and aperistalsis of the lower oesophagus. This situation in turn causes a dilated oesophagus. Clinically and radiologically pseudo-achalasia and achalasia are very similar. • Treatment is by cardiomyotomy, preferably laparoscopic, although in unfit patients or in more mild cases the alternative treatment is endoscopic pneumatic dilatation or botulinum toxin injections.

Up to 45% of cases of Hodgkin’s lymphoma are linked to Epstein–Barr virus (EBV). Infectious mononucleosis is a common manifestation of EBV and seems to precede diagnosis of Hodgkin’s lymphoma by about 3–4 years. Immune deficiency is a clear risk factor for Hodgkin’s lymphoma, which was first described by Thomas Hodgkin in 1832.

• Patients who have received a solid organ transplant are about two to four times more likely to develop Hodgkin’s lymphoma • Hodgkin’s lymphoma is one of the few diseases that has a higher prevalence in more affluent people. It is believed that this could be due to reduced exposure to common infections in childhood. Being an only child also represents a risk factor for seemingly similar reasons.

Rheumatoid nodules develop in about 30% of patients with rheumatoid arthritis. They consist of a central necrotic area surrounded by palisaded histiocytic macrophages, all enveloped by lymphocytes, plasma cells and fibroblasts. Nodules and vasculitis can also develop in many of the visceral organs

• So, we should rheumatoid factor • Testing for antinuclear antibodies is a non-specific way of looking for disease. Elevated levels may be present in individuals without disease. Its interpretation has to be undertaken in the context of the clinical picture and other tests.

This classic radiological finding is most commonly seen in Ewing’s sarcoma (a pattern of bone destruction and multiple concentric rings of new bony formation), a malignant round-cell tumour of the long bones. It carries with it a relatively poor prognosis. If metastatic disease is present, 80% of patients will not make it to 5 years before death.

• Osteosarcoma is the most common primary malignancy of bones

Ewing’s sarcoma, Small round blue cell tumours are a group of tumours characterised by the presence of undifferentiated cells that stain blue at histology. In Ewing’s sarcoma, plain X-rays do not usually reveal the full extent of bone involvement. Computed tomography scan and magnetic resonance imaging are better for defining the extent of disease and can help guide treatment. Many other benign and malignant tumours can have identical appearances, so diagnosis is made by biopsy. Ewing’s sarcoma is commonly seen at young age. It is malignant. Most tumours develop in the extremities, but any bone can be involved. Ewing’s tumour tends to be extensive, sometimes involving the entire bone shaft. Pain and swelling are the most common symptoms. Lytic destruction is the most common X-ray finding, but multiple layers of subperiosteal reactive new bone formation may produce an ‘onion skin’ appearance.

• Osteoblastoma is a rare benign tumour of the bones. It usually affects young adults in their 20s or 30s

Page | 73

• Chondroblastoma is a benign tumour of the bone that arises in the epiphyses or apophyses. It also affects young people. Chondroblasts are seen in biopsy samples.

Hyaline cartilage forms the articular surface and is avascular, relying on diffusion from synovial fluid for nutrients

• Chondrocytes secrete proteoglycans as well as collagen. They are embedded in the cartilage and migrate to the joint surface along with the matrix that they produce.

Ortolani’s test checks for dislocated hips that are relocatable. The femoral head is felt returning into the acetabulum. The hips and knees are flexed, and the hips slowly abducted, relocation occurs as full abduction is approached, if the hip is irreducible abduction is restricted

• Barlow’s test is checking for a hip that may be dislocated. The pelvis is fixed and backwards pressure applied to attempt to sublux/dislocate the hip. If the hip dislocates/subluxes then relocation is attempted with forward pressure or abduction of the hip.

In the presence of bone metastases, calcium ions move from the serum into the metastatic bone cells, as they are blastic cells. This fall in serum calcium stimulates the parathyroid glands to release more parathyroid hormone (PTH), which plays an important role in bone and calcium homeostasis. Its general purpose is to release calcium ions from the bone into the serum, so that it can be used in other processes in the body.

The action potential of the skeletal muscle spreads out from the motor end plate, through the T tube system, this causes the mobilization of Ca from the sarcoplasmic reticulum to the cytoplasm. The action potential of cardiac muscle is longer than that of the skeletal muscle.

• There is no plateau phase in the action potential of skeletal muscle. A plateau phase is seen in the action potential of cardiac muscle. • The mobilization of Ca is from the sarcoplasmic reticulum into the cytoplasm

In what order do the stages of bone healing occur?

Fracture haematoma, vascular granulation tissue, sub-periosteal osteoblast stimulation, bone matrix deposition, endochondral formation, callous formation, remodeling

Fibroadenomas are the most common benign lesions seen in young women (less than 30) They account for 90% of breast masses found in women under the age of 19 years. They arise from the breast lobule and contain both stromal and epithelial elements.

• They present clinically as a firm, smooth, mobile, well defined, painless mass of 1–5 cm in maximal diameter. Fibroadenomas can enlarge during pregnancy and can be multiple at presentation in 10–15% of cases. They tend to occur most commonly in the upper outer quadrant of the breast. • On radiography they appear well circumscribed, discrete, homogeneous lesions. Diagnosis is usually with a biopsy under ultrasound guidance and if no atypical features are found it can be followed up radiographically. • If atypical features are found or lesions are large and symptomatic then surgical excision is preferred.

Page | 74

• The risk of malignant transformation of a fibroadenoma is <1%

Locular Carcinoma can be mammographically occult.

Lymphocytic lobulitis, Patients with type I diabetes are particularly at risk of this disorder. an irregular firm lump

Triple-negative breast cancer means that the tumour growth is not supported by hormones and, subsequently, the disease does not respond to commonly used therapies such as tamoxifen and trastuzumab. NICE guidelines suggest that genetic screening should be offered to all patients with a 10% risk of carrying BRCA1, BRCA2 or TP3. This correlates to women aged under 40 who present like this. BRCA1 and BRCA2 mutations increase the risk of breast cancer in both men and women. Women with these mutations are more likely to present at an earlier age and with triple-negative breast cancer.

Congenital adrenal hyperplasia (CAH); Partial deficiency of 21-hydroxylase (ie in the simple-virilising form of CAH) allows patients to produce near normal (or normal) amounts of cortisol due to increased ACTH output. Late-onset disease can be due to partial enzyme deficiency.

• 17α-Hydroxylase deficiency is a rare form of congenital adrenal hyperplasia that features excess mineralocorticoid production but deficiency in the production of sex steroids and cortisol. • In the salt-losing form of congenital adrenal hyperplasia (ie with complete deficiency of 21- hydroxylase, there is complete inability to produce cortisol and aldosterone), leading to an earlier presentation than in partial 21-hydroxylase deficiency

Thyroid adenomas may be so small that they only show up on a I-radioistope scan.

• The autoantibody in Graves’ disease is an IgG antibody known as a long-acting thyroid stimulator (LATS). It stimulates the function and growth of the thyroid follicular epithelium.

Carcinoid heart disease is characterised by pathognomonic plaque-like deposits of fibrous tissue secondary to elevated levels of serum serotonin. These deposits occur most commonly on the endocardium of valvular cusps and leaflets, the cardiac chambers, and occasionally on the intima of the pulmonary arteries or aorta. Carcinoid syndrome is due to the excess secretion of serotonin by argentaffin cells. Carcinoid syndrome typically causes diarrhoea. Diagnosis is by raised urinary excretion of 5-hydroxy indole-acetic acid (5-HIAA), a metabolite of serotonin. VMA levels are elevated in phaeochromocytoma.

A deficiency of insulin causes protein catabolism. Glucose is synthesised (mostly from amino acids, released by protein catabolism).

• There will be a high concentration of fatty acids in the blood. A deficiency in insulin results in increased fatty acid production from stored lipids, increased fatty acid release into the blood and decreased fatty acid uptake.

Most patients with primary aldosteronism (Conn syndrome) have an adrenal adenoma. The increased plasma aldosterone concentration leads to increased renal Na reabsorption, which results in plasma volume expansion. The increase in plasma volume suppresses renin release from the juxtaglomerular apparatus and these patients usually have low plasma renin levels

Page | 75

• Upright posture causes a decrease in renal perfusion pressure and therefore increased renin release from the juxtaglomerular apparatus.

The diffuse form of scleroderma can be associated with hyperplastic arteriolosclerosis and malignant hypertension.

• Post-streptococcal glomerulonephritis produces glomerular hypercellularity. • The child is most likely to have minimal-change disease and the kidney will be grossly normal, will be normal on light microscopy and will show only fusion of foot processes on electron microscopy. • End-stage kidneys all look alike: thickened arteries, globally sclerotic glomeruli and interstitial scarring with chronic inflammation.

Antithrombin III deficiency is a rare hereditary disorder that predisposes patients to clotting and often presents with recurrent deep vein thrombosis (DVT) and pulmonary embolism (PE).

• a-2-Antiplasmin is responsible for inactivating plasmin. As plasmin is responsible for degrading blood clots, reduced inhibition of plasmin increases the risk of bleeding. • a-2-Macroglobulin also inhibits plasmin. If levels of a-2-macroglobulin are reduced then there is reduced inhibition of plasmin therefore increasing the risk of bleeding. • Factor V deficiency is also known as Owren’s disease or parahemophilia. It’s a rare bleeding disorder that results in poor clotting after an injury or surgery. Factor V deficiency shouldn’t be confused with factor V Leiden mutation, a much more common condition that causes excessive blood clotting. • Fibrinogen deficiency (or factor I deficiency) is a rare bleeding disorder that would increase the risk of spontaneous bleeding.

Protein C deficiency increases the likelihood of a patient forming clots as it is hypercoagulable state. Therefore haemarthrosis would not be a presenting symptom.

The myeloproliferative diseases are a group of diseases of the bone marrow in which excess cells are produced. They are related to, and can evolve into, myelodysplastic syndrome and acute myeloid leukaemia, although the myeloproliferative diseases on the whole have a much better prognosis than these conditions. All cell lines can be increased with myeloproliferative disorders. Thrombocytosis is one manifestation.

Blast cells are a feature of acute myeloid leukaemia and this would be associated with a leucocytosis.

• Nucleated red blood cells are found in leucoerythroblastic pictures such as metastatic carcinoma. • Schistocytes are fragmented red blood cells and are often seen in patients with haemolytic anaemia

Factor V Leiden tests are indicated for venous thrombo-embolism presenting at a relatively young age, or in an unusual part of the body. They are also indicated when a patient has a strong personal or family history of recurrent deep vein thromboses (DVTs), or when they are experiencing unexplained miscarriages. In Britain, 5% of the population carry one or more genes for factor V Leiden. These

Page | 76 patients are at an increased risk of venous thrombosis. This risk is five times greater than that of the general population. Factor V Leiden is pro-thrombotic.

Factor XI is activated by factor XIIa. Factor XI (or plasma thromboplastin antecedent) is one of the enzymes of the coagulation cascade. Like many other coagulation factors, it is a serine protease, produced by the liver and circulates in its inactive form. The zymogen factor is activated into factor XIa by factor XIIa (FXIIa), thrombin and it is also autocatalytic. FXI is a member of the ‘contact’ pathway. Factor XIa activates factor IX, which in turn activates factor X

Thrombomodulin is a membrane protein expressed on the surface of endothelial cells. It functions as a cofactor in the thrombin-induced activation of protein C in the anticoagulant pathway, ultimately leading to reduced blood coagulation. In a patient with thrombomodulin deficiency therefore, clotting could in theory be enhanced inappropriately.

Conditions such as haemolytic anaemia or recovery from acute haemorrhage lead to an increased erythropoietic drive and hence the appearance of reticulocytes in the circulation. This may manifest as a raised mean cell volume because reticulocytes have a higher cell volume than mature red cells. A raised mean cell volume occurs with macrocytosis associated with the dietary deficiency or malabsorption of vitamin B and folate, liver disease (especially associated with alcohol abuse), hypothyroidism, myelodysplasia and after exposure to some drugs (cytotoxics). Vitamin B is found exclusively in animal products and deficiency is a hazard of veganism. Conditions such as haemolytic anaemia or recovery from acute haemorrhage lead to an increased erythropoietic drive and hence the appearance of reticulocytes in the circulation. This may manifest as a raised mean cell volume because reticulocytes have a higher cell volume than mature red cells. Iron deficiency is associated with microcytosis.

Tissue plasminogen activator (tPA) is produced by endothelial cells. It binds to fibrin, converting fibrin- bound plasminogen to plasmin, which in turn degrades to fibrin. tPA is the commonest activator of fibrinolysis.

• Tranexamic acid, Fibrin dissolution can be reduced by tranexamic acid, given in polytrauma protocols, which acts by inhibiting plasminogen activation and fibrinolysis. It is useful in cases in which it is difficult to stop haemorrhaging directly, as in prostatectomy. • Epsilon-aminocaproic acid (EACA) inhibits the fibrinolytic system

Haemophilia B is a sex-linked disorder arising from deficiency of factor IX.

• Factor XI deficiency is otherwise known as haemophilia C and is also passed on in an autosomal- recessive pattern. • Alexander’s disease is deficiency of factor VII and is passed on in an autosomal-recessive pattern.

Heparin greatly enhances antithrombin III activity. Antithrombin III is a small molecule that inactivates several enzymes of the coagulation system. It is a glycoprotein produced by the liver. Antithrombin III is a serpin (serine protease inhibitor) that inactivates a number of enzymes from the coagulation system, namely the activated forms of factor X, factor IX, factor II (thrombin), factor VII, factor XI and factor XII. The rate of its reaction with these molecules (ie, its effectiveness) is greatly enhanced by heparin. Antithrombin III deficiency is a rare hereditary disorder that generally comes to light when a patient suffers recurrent venous thrombosis and pulmonary embolism.

Page | 77

• Coumarin is another name for warfarin and has no effect on antithrombin III activity • Warfarin has no effect on antithrombin III. Instead it acts upon the vitamin K-dependant factors II, VII, IX, and X.

Calcium is an essential co-factor in the coagulation cascade. The clotting cascade has two different pathways, the extrinsic (activated by tissue thromboplastin) and the intrinsic (activated by collagen and glass among others). These both culminate in the activation of the common pathway. This involves activation of factor X that in turn activates prothrombin, generating thrombin. The thrombin activates fibrinogen to produce fibrin. This fibrin polymerises under the effects of factor XIII to form the fibrin plug that contributes the framework of the clot within which platelets and erythrocytes become enmeshed. Calcium is a co-factor and is vital for function of the clotting cascade.

• The result of the coagulation cascade is a platelet plug strengthened by fibrin. • Citrate or EDTA are used as anticoagulants in blood sample bottles as they chelate calcium to stop clotting from occurring. • The clotting cascade is deficient in haemophilia A (factor VIII), haemophilia B (factor IX) and von Willebrand’s disease (a clotting protein which binds factor VIII). • Von Hippel–Lindau’s disease is a genetic abnormality associated with tumour formation.

Methotrexate can reduce folate levels within the body, therefore patients receive folic acid supplementation during treatment.

Howell–Jolly bodies are nuclear fragments of condensed DNA that appear as spherical blueblack inclusions within red blood cells on Wright-stained smears. They are found in splenectomy patients as the spleen would normally filter out these type of red blood cells.

Erythropoietin is a glycoprotein hormone produced primarily by the peritubular fibroblasts of the renal cortex, in response to hypoxia. In premature, as well as full-term infants, the liver is the primary site of erythropoietin production. The kidney becomes the primary site of erythropoietin synthesis shortly after birth. Small amounts of erythropoietin are also synthesized in the hepatocytes of healthy adults. Erythropoietin production is stimulated by reduced oxygen content in the renal arterial circulation. Some of the clinical conditions that give rise to tissue hypoxia include anaemia, lung disease and cyanotic heart disease, all of which stimulate erythropoietin production

• Erythropoietin production is stimulated by reduced oxygen in the renal arterial circulation. • The liver, rather than the kidney, is the site of development of erythropoietin in the fetus.

Factor VIII is synthesised by the sinusoidal cells of the liver as well as by the vascular endothelium.

Iron deficiency anaemia is microcytic and hypochromic on a blood film. Anisocytosis and poikilocytosis are features of iron deficiency anaemia.( Anisocytosis means that there are red blood cells of varying sizes on your blood smear. Poikilocytosis means that there are red blood cells of varying shapes on your blood smear)

• Basophilic stippling is seen in lead poisoning and thalassemia. Sideroblasts are a feature of sideroblastic anaemia. • Spherocytes are seen in haemolysis such as haemolytic anaemia. • Target cells are seen in liver disease, thalassemia and sickle cell disease.

Page | 78

In Grey Platelet Syndrome (GPS) there is a reduction or absence of alpha granules in the platelets. GPS is a very rare inherited disorder that is characterised by thrombocytopenia, abnormally large, agranular platelets in peripheral blood smears (which appear grey), splenomegaly, and myelofibrosis. Alpha platelet granules normally serve to promote platelet adhesion and wound healing. Patients with GPS may present with prolonged bleeding following injury, nosebleeds and easy bruising.

LMWH acts predominantly upon factor Xa, via antithrombin III. LMWHs are prepared by the degradation of unfractionated heparin, so reducing their mean molecular weight to less than 8 kDa. LMWHs have a longer half-life than unfractionated heparin and better bioavailability after subcutaneous injection. Only a once-daily administration is required. Anti-factor Xa levels can be measured to monitor the effects of LMWH. Excretion is almost exclusively renal and so dose reduction is required in patients with renal failure.

Factor IX, When activated into factor IXa, it acts by hydrolysing one arginine–isoleucine bond in factor X to form factor Xa.

The low mean corpuscular volume (MCV), low ferritin and high transferrin suggest iron deficiency anaemia.

• Anaemia of chronic disease would be normocytic rather than microcytic. The transferrin falls and ferritin is raised as negative and positive inflammatory markers respectively. • Thalassaemia suffers would have a high ferritin level.

The presence of nucleated red blood cells along with immature granular cells shows a leucoerythroblastic picture. This is due to space-occupying lesions in the bone marrow, including myeloma, myelofibrosis or in malignancy, metastatic prostatic or lung carcinoma.

Nitric oxide is derived from l-arginine by nitric oxide synthase.

The pancreatic juices are rich in potassium and the presence of a high-output fistula may lead to hypokalaemia.

• Metabolic acidosis results in hyperkalaemia. • Extensive muscle trauma or rhabdomyolysis would cause hyperkalaemia due to the release of potassium from the intracellular space.

28 mmol of lactate is found in 1 litre of Hartmann’s solution. This undergoes gluconeogenesis to be turned into glucose.

• Bicarbonate is not found in Hartman’s solution. However some of the lactate is oxidised using up hydrogen ions and generating bicarbonate • The constituents of Hartmann’s solution (per litre) include sodium 130 mmol, chloride 109 mmol, potassium 4 mmol, calcium 1.5 mmol and lactate 28 mmol

Glucose 5% It is an isotonic fluid on administration; however, the glucose is quickly metabolised and the resulting solution is hypotonic. It contains 200 kcal/l.

Carbon dioxide produces H+ when dissolved in blood. This is buffered mainly by haemoglobin.

Page | 79

• Carbon monoxide, rather than carbon dioxide, is carried on haemoglobin as carboxyhaemoglobin. • Oxygen, rather than carbon dioxide, combines reversibly with myoglobin. • Carbon dioxide (CO2 ) decreases the oxygen binding power of haemoglobin (Hb) and it is 25 times more soluble in blood than is oxygen. CO2 is dissolved in the blood and via carbonic anhydrase forms hydrogen and bicarbonate ions. The bicarbonate diffuses out of the cell in exchange for chloride ions, the so-called chloride shift.

Mitral regurgitation is likely to be associated with left ventricular failure causing back pressure on the pulmonary circulation. This then results in a transudate pleural effusion.

An effusion with less than 2.5 g of protein per 100 ml is a transudate. Lights criteria describes an exudate if one of the following is present:

• Pleural protein:serum protein >0.5, • Pleural LDH:serum LDH >0.6, • Effusion LDH level greater than 2/3 of the upper range of serum LDH

Typically, the cause of an exudative effusion includes infection, malignancy or inflammatory causes, whereas transudates are by cardiac failure, nephrotic syndrome or atelectasis, although exceptions do occur.

Extracellular fluid depletion leads to activation of the renin–angiotensin–aldosterone system, increasing renal sodium reabsorption and potassium excretion.

• ACE inhibitors can cause hyperkalaemia as a side-effect. • Acute tubular necrosis would reduce renal potassium excretion and sodium reabsorption, and thus elevate plasma potassium. • Hypoaldosteronism would raise the potassium concentration. • In metabolic acidosis the potassium concentration is increased. It falls in metabolic alkalosis. • The loss of potassium ions may feature in both type 1 (distal) and type 2 (proximal) renal tubular acidosis. • A side-effect of an angiotensin-converting enzyme (ACE) inhibitor is hyperkalaemia. • In Conn’s syndrome, there is excessive production of aldosterone which causes sodium retention and potassium loss leading to hypokalaemia.

In haemolysis, free haemoglobin combines with haptoglobin, so serum levels fall as a result.

• Patients with haemolytic states do not become iron deficient. • Lactate dehydrogenase (LDH) is released from red blood cells that are undergoing haemolysis raising the serum LDH level. • Bilirubin levels are increased in haemolysis. • Serum iron levels are unchanged in patients suffering from haemolysis.

Tumours commonly metastasizing to the adrenal glands are lung and breast cancers. Kidney tumours more commonly metastasise to lung and bone. Prostate tumours more commonly metastasise to lung and bone. Pancreas tumours more commonly metastasise to the liver. Colorectal tumours more commonly metastasise to lung and liver

Page | 80

RNA and protein synthesis occur during the first growth phase, G1

• G1 is a gap phase under the influence of the p53 gene. • In normal tissues, cells with significant damage to their DNA are arrested at the G1 phase. • G2 is a gap phase when cells contain twice as much DNA as non-dividing cells. • Duplication (replication) of cellular DNA occurs during S phase, which is followed by a second growth phase (G2). Cytotoxic drugs usually act on cycling cells and can be phase-specific or non-phase-specific. • Mitosis occurs in M phase. • The duration of the cell cycle varies from 20 to 100 h (20–24 h in rapidly growing cells).

Gamma-glutamyl transpeptidase (GGT) can be elevated by testicular tumours, but is not commonly used in clinical practice.

Neuroblastoma is a tumour of the sympathetic nervous system, and is usually highly malignant. As neuroblastic tumours are derived from neural crest cells, they can affect the adrenal medulla (and adjacent retroperitoneal tissue), sympathetic neurones and melanocytes.

Painters, printers, mechanics and others working with petroleum derivatives and organic solvents (benzene exposure) are at an increased risk of developing leukaemia, lymphoma and multiple myeloma.

• Arsenic is used in pesticide manufacture and exposure also occurs in metal smelters. It is associated with skin and lung cancers as well as bladder tumours. erb B-2 is a growth factor receptor oncogene.

• ras is a signal transducer • myc is a transcription factor • PDCD-1 is the gene that codes for programmed cell death 1 receptor protein. • bcl-2 is a programmed cell death regulator • p53 is associated with Li–Fraumeni syndrome. • INK4a is associated with melanoma. • BRCA2 is associated with breast cancer. • WTI is associated with Wilms’ tumour. • VHL is associated with renal malignancy.

Adenomas are benign tumours of ductal or glandular epithelial cells. (Not transitional epithelium)

The distinction between a benign renal adenoma and renal adenocarcinoma is commonly made on the basis of size, tumours less than 2 cm in size rarely being malignant and those greater than 3 cm in size behaving in a malignant fashion.

Familial adenomatous polyposis is a condition which increases the risk of colorectal carcinoma. The APC gene is found on Ch5 and is mutated in a large proportion of individuals with this condition.

• Lynch syndrome, is associated with colorectal cancer (80% lifetime risk), ovarian, endometrial, stomach, small intestine, hepatobiliary, upper urinary, brain and skin cancers. It is due to DNA mismatch repair impairment

Page | 81

• The RB1 gene is a tumour suppressor gene on Ch13. In inherited RB (approximately 40% of cases) there is a germ-line inactivation of the RB1 gene. This condition is usually bilateral. In the more common non-hereditary RB both of the mutations are somatic and this condition tends to affect only one eye. • The risk of developing breast cancer with the BRCA1 mutation is 60–90%, whereas with the BRCA2 mutation, this is 45–85%. • Li-Fraumeni syndrome is an autosomal-dominant condition associated with mutations in the TP53 gene (Ch17), which encodes for the p53 protein. It is associated with osteosarcoma, breast cancer, brain cancer, leukaemia and adrenocortical cancer.

To metastasise, cells must overcome their normal cohesion and be able to invade the tissue, a property facilitated by the secretion of protease enzymes.

• Metastasis requires loss of cellular cohesion (loss of tight junction barrier function). • Adhesion to the basement membrane requires the expression of integrins, but not cadherins, which enhance cell-to-cell adhesion

Hamartomas are not true tumours but represent overgrowth of one or more cell types that have normal constituents. However, they are arranged in irregular fashion (examples are haemangioma, lymphangioma, lipoma and neurofibroma). sis is the best studied growth factor oncogene. It leads to the overproduction of platelet derived growth factor, which stimulates cells to grow.

Sarcoma spreads across fascial planes and metastasises via the bloodstream, the lungs are the most common site of metastatic deposits. They are locally invasive, but spread by ‘pushing’ along tissue planes rather than by direct invasion of major structures. Fascia, major nerve sheaths and adventitia of large vessels are relatively resistant to invasion

BCC is the commonest skin tumour on the face and 90% of all BCCs occur in the head and neck region

The barium enema in carcinoid syndrome can produce mucosal ulcers mimicking Crohn’s disease. Carcinoids are occasionally found in the appendix, therefore all routine appendicectomy cases should have the histology checked in case a right hemicolectomy is required.

• While argyrophil and argentaffin positivity supports the diagnosis, a pathological examination on both cytology and histology is required. • The small bowel and appendix are the most common primary sites for carcinoid tumours.

Oncogenes tend to code for proteins (oncoproteins) involved in key cellular regulatory processes. Oncogenes can cause malignancy not only when their gene product is altered by mutation, but also when they are over- or underexpressed or expressed at an inappropriate stage of the cell cycle

Both leukaemia and solid tumours occur at an increased rate following exposure to ionising radiation.

• Oocytes and spermatogenesis are radiosensitive, and radiation exposure may therefore impair fertility. However, the rate of cancer does not increase Both leukaemia and solid tumours occur at an increased rate following exposure to ionising radiation. Oocytes and spermatogenesis are

Page | 82

radiosensitive, and radiation exposure may therefore impair fertility. However, the rate of cancer does not increase

Both the adenovirus E1B and papillomavirus E6 gene products bind to p53 and interfere with its functioning. Another protein involved in the cell cycle and triggering of apoptosis is the RB protein, the functioning of which is interfered with by a number of viruses including papillomavirus, adenovirus and the SV40 virus.

IPPV creates a positive intra-thoracic pressure (this is normally negative) and a compression tamponade, thereby reducing venous return, cardiac output and therefore blood pressure. Additionally, the alveoli may be subjected to high inflation pressures and result in subsequent p(CO ) barotrauma and pneumothorax.

• IPPV leads to an increase in intra-thoracic pressure and a reduction in cardiac output; there a subsequent is reduction in liver, kidney and intestinal blood flow • Due to the compression tamponade secondary to positive intra-thoracic pressure results in hypotension, not hypertension. • Intermittent positive-pressure ventilation (IPPV) causes an increase in intracranial pressure, as do positive end-expiratory pressure (PEEP) ventilation and obstruction of central venous drainage.

Small-cell cancers of the lung account for less than one-quarter of all lung cancers. It is named after the small round cells that make up these tumours.

• Squamous-cell carcinoma makes up nearly 25%, • adenocarcinoma 40% and • large-cell carcinoma approximately 10% of all lung cancers. o Small-cell carcinoma is a tumour of neuroendocrine origin that may exist in the classical oatcell form or as an intermediate-cell type. o Small-cell lung cancer is almost always caused by smoking while adenocarcinoma is seen in elderly non-smokers. o These tumours are highly malignant and are usually disseminated at presentation. o Small-cell lung cancers are often associated with ectopic hormone production and paraneoplastic syndromes such as adrenocorticotrophic hormone (ACTH) (Cushing’s syndrome), syndrome of inappropriate antidiuretic hormone secretion (causes hyponatraemia) and myasthenic syndrome. Note that hypercalcaemia is much more frequently associated with non-small-cell lung tumours (ie due to parathyroid hormonerelated peptide) than small-cell tumours. o The median survival of patients with small-cell cancer is 3 months if untreated and 12–18 months if treated.

Lung compliance is inversely proportional to the elasticity of the lung. For example, in emphysema, all elastic tissues have been destroyed, compliance increases and the lungs are easier to expand due to less elastic resistance. Lung compliance will increase when the patient is placed in the head-up position.

o A general anaesthetic will result in reduced compliance due to reduction in the function of the respiratory muscles as such patients require mechanical ventilation with positive airways pressures overcome the reduced compliance.

Page | 83

o Compliance increases in hypovolaemia (because there is less blood in lung and it is easier to expand). o Pulmonary fibrosis makes the lung more difficult to expand, as does a general anaesthetic and any feet-up position.

Anaemia and normal lungs, anaemia reduces oxygen delivery, however the extraction by the tissues is the same. This results in a lower mixed venous pO2.

o A patient with anaemia and normal lungs typically has a normal arterial p(O2) o Although the arterial p(O2) is typically normal, the p(O2) of mixed venous blood must fall. This is because mixed venous pO2 is related to oxygen delivery to the tissues.

The causes of ARDS can broadly be divided into direct and indirect insults to the lung. Indirect insults include massive blood transfusion, pancreatitis and fat embolus. Patients diagnosed with ARDS should be cared for in a High Dependency or Intensive Care Unit as it is associated with a high morbidity and mortality.

o Direct insults include lung contusion, aspiration and pneumonia.

CO2 retention causes a respiratory acidosis, which increases serum potassium, in addition to catecholamine release causing an increased risk of cardiac arrhythmias and sweating.

o Increased CO2 causes a rise in CSF pressure. Hyperventilation to reduce CO2 will decrease CSF pressure in head injuries, however, studies have demonstrated that hyperventilation to induce hypocarbia does not impact on survival. Furthermore, hypocarbia results in cerebral vasoconstriction, reducing cerebral blood flow and potentially inducing ischaemia. Current guidance is to aim for a p(CO2) of 4.0 kPa. o The pulse is classically bounding and the patient may vasodilate. o Physiological studies have shown that heart rate, stroke volume, cardiac output, and mean arterial blood pressure (BP) are increased by hypercapnia

Physiological and histological features of ARDS include: increased capillary permeability; interstitial and alveolar oedema; fibrin exudation; hyaline membrane formation, and, later, diffuse late interstitial and alveolar fibrosis.

Cyanosis is a blue coloration of the tissues commonly due to the presence of deoxygenated haemoglobin, rarely it is due to methaemoglobin (reduced haemoglobin), a condition caused by drug reactions.

o Peripheral cyanosis occurs due to excess oxygen utilisation, outweighing supply in the peripheral tissues. o Central cyanosis indicates profound arterial hypoxemia and should be treated with oxygenation while a cause is sought. o In profound anaemia there is too little haemoglobin for enough of it to be deoxygenated to generate visible blue colour. As such, cyanosis does not occur in profound anaemia. o It is the presence of deoxygenated haemoglobin, not the presence of carboxyhaemoglobin, that produces cyanosis.

Page | 84

o Central cyanosis is distinguished from peripheral cyanosis by inspecting the mouth. Blue discolouration of the oral mucosa is indicative of central cyanosis

The treatment of choice for Kaposi’s sarcoma depends on the type, size and the patient’s general health. Not all non-AIDS-related Kaposi’s sarcoma needs treating but if the lesions are large or very visible they can be treated with radiotherapy or chemotherapy. In transplant related Kaposi’s sarcoma reducing or changing the immunosuppressive drugs can improve it, otherwise radiotherapy or chemotherapy may be necessary. In AIDS-related Kaposi’s sarcoma treatment can be with chemotherapy or interferon but the side-effects of interferon treatment can be severe and not all patients will be able to tolerate this. There is little reason to excise Kaposi’s sarcoma surgically since the disease is multicentric and recurs rapidly.

o The course of the disease in non-AIDS patients is slow, and frequently they survive for more than 10 years after the onset of the disease. This pattern is contradictory to that of AIDS related disease, which can be rapidly fatal. With improvements in treatment, however it can now be controlled for many years. o Biopsy confirmation of the skin lesions is essential. However, caution should be exercised when handling such specimens in view of the potential for transmission of AIDS-related disease from blood products and biopsy tissue. o Kaposi’s sarcoma is a multicentric, proliferative disorder of vasoformative tissue that usually follows a malignant neoplastic course. In the early stages, the lesions present as purplish pigmented patches of purpura with haemosiderosis, mostly over the lower legs and feet, similar to a bruise. Unlike a bruise they do not lose their colour when pressed. Later, indurated plaques and firm vascular polypoid excrescences appear. o Kaposi’s sarcoma is of two major types: AIDS related and non-AIDS related (classic, endemic, transplant related). AIDS-related disease is mostly seen in young people while non-AIDS related disease generally affects older or middle-aged individuals.

Solar or actinic keratosis is rough patches of skin caused by damage from ultraviolet light exposure. They have a potential to become squamous-cell carcinoma.

o A Spitz naevus is a benign skin tumour without malignant potential. They are melanocytic lesions and can be mistaken for malignant melanoma o Keratoacanthoma is a rapidly growing benign lesion typically arising around a hair follicle. It has no malignant potential. o Necrobiosis lipoidica is often seen in patients with diabetes mellitus (although not exclusively). It is a disorder of collagen degeneration with a granulomatous response, fat deposition and thickening of blood vessel walls. It appears as a tender yellowish brown patch often on the lower legs. It does not predispose to cutaneous malignancy. o Seborrhoeic keratosis is a benign skin condition with no malignant potential. It develops from a proliferation of epidermal cells and appears as waxy scaly elevated black/brown growths. Typically affecting the face, chest, shoulders and back.

Malignant melanoma, even if stage IA, has a 97% survival rate at 5 years. It is important to understand the basic principles of staging of various common malignancies and to be able to sensibly judge a prognosis based on them. As a general rule higher staged cancers have a worse survival, but the tumour type is important.

Page | 85

o Stage IIIA disease is where there is no evidence of ulceration of the primary lesion. This infers a less aggressive disease process and carries with it a much better prognosis than stage IIIB disease where the primary lesions are ulcerating. o MM Stage IIIB, The 5-year survival rates are around 59% with active treatment and around 43% at 10 years. Stage III disease represents those who have presented with disease. This would be a sensible estimate for stage IV disease, which represents patients with evidence of metastatic disease. o Although metastatic melanoma carries with it a poor prognosis, even stage IV disease has a 40% survival rate at 5 years, dropping to 24% at 10 years o Satellite lesions are areas of tumour located away from but within 2 cm of the primary site. It represents an aggressive tumour. o Amelanotic lesions are associated with a poor prognosis due to difficult recognition but are not a sign of aggressiveness. In amelanotic melanomas the cells do not make melanin, the dark pigment that gives most lesions their brownish colour. They can be pink, purple, red or normal skin colour and are therefore difficult to recognise. Their atypical appearance often leads to delay in diagnosis and advanced lesions are more difficult to treat. The prognosis is poor and the rate of recurrence and metastases is high. o The most aggressive cutaneous lesions are those on the head and neck as they are more commonly associated with vascular and neural invasion. o Flat lesions grow laterally before invading vertically. Nodular melanomas are the most aggressive type of melanoma. ✓ Stage 0 equates only to melanoma in situ. ✓ Stages IA and IB relate to cancers with no lymph node or distant metastatic deposits. Lesions are less than 2mm in thickness without ulceration or thinner than 1 mm with ulceration. The 5-year survival rate is around 92–98%. ✓ Stage II melanomas are categorised by a larger lesion, but with no local lymph node involvement and no evidence of metastatic disease. The 5-year survival rate is any where from 81% to 53%. ✓ Stage III includes any size lesion with any degree of ulceration. Its unique identifier is local nodal involvement. The 5-year survival rate varies from 78% to 40%. ✓ Stage IV melanomas include any cancer with evidence of metastatic spread. The survival at 5 years is around 15–20%. Metastatic disease is stage IV.

Staging of cancers can be either clinical or pathological, but it is important to understand the implications of each on the patient’s prognosis and management. Even locally advanced cancers may be treated with excision, whereas metastatic disease will often require systemic treatment. Malignant melanoma is a common cancer with clear guidelines.

Bowen’s disease is also called squamous-cell carcinoma in situ. The cancer cells are restricted to the epidermis. It predominantly occurs on the legs as a result of sun exposure.

The extensive venous plexus surrounding the prostate makes haematogenous spread likely. Prostate cancer can also spread by direct extension or lymphatics.

o Metastases to regional lymph nodes in the abdomen, Lymphatic drainage routes generally follow the path of arteries. o Basal-cell carcinomas invade locally and metastases are rare occurring in only 0.1% of cases.

Page | 86

o Transluminal metastases are uncommon.

Familial melanoma is recognised in around 1–2% of patients when two first-degree relatives also have melanoma (CDKN2A, CDK4, BRCA1). Although many genetic causes are unrecognised and unstudied, many familial links have been found. BRCA1 carries with it a higher risk of melanoma, along with breast and ovarian cancer.

✓ Treacher Collins syndrome, also known as mandibulofacial dysostosis, is an autosomal dominant syndrome that results in craniofacial defects with normal intelligence. ✓ Both Gorlin syndrome and PTCH2 are associated with basal cell cancer formation. ✓ Rombo syndrome is characterised by milia, atrophoderma and increased rates of basal cell cancer. Bazex–Dupré–Christol syndrome results in hypohidrosis, milia, follicular atrophoderma and basal cell cancers in teens and patients in their early 20s. ✓ Polycystic kidney disease is associated with intracranial aneurysms, renal failure and cyst formation in other organs. Immunology The complement cascade is an enzyme-driven process involving the activation of precursors that promote inflammation, phagocytosis and the direct killing of micro-organisms. There are two pathways – the classical and the alternative – also known as direct and indirect. Direct activation of complement occurs with activated immunoglobulins. The indirect pathway is activated by micro-organisms such as bacteria. Bacterial cell surface molecules interact with the plasma proteins and trigger complement activation.

Anti-neutrophilic cytoplasmic autoantibodies are most commonly seen in patients with granulomatosis with polyangiitis (formerly called Wegener’s granulomatosis), eosinophilic granulomatosis with polyangiitis (formerly called Churg–Strauss syndrome), and microscopic polyangiitis.

• Patients with systemic lupus erythematosus usually demonstrate anti-nuclear antibodies as well as antibodies to dsDNA in their serum. • The most sensitive antibody for detection of rheumatoid arthritis would be anti-CCP antibodies, which are more specific and present earlier in disease than rheumatoid factor antibodies

Typical antibodies associated with systemic sclerosis include; anti-centromere (limited systemic sclerosis) and anti-Scl-70 (diffuse systemic sclerosis)

A heterotopic graft involves transplantation of a donor organ to a different anatomical site than that of the existing organs in a recipient.

• An isograft is a transplant of tissue between genetically identical individuals eg identical twins.

This is the definition of being rhesus positive, these individuals should not donate their blood to rhesus- negative patients – particularly in women of child bearing age as it can precipitate haemolytic disease of the newborn.

• In AB blood group, the patient will have A and B antigen but not anti-A and anti-B antibodies. • group individuals are universal donors (they have anti-A and anti-B antibodies)

Page | 87

A CD4 count less than 200 is typically thought to be the threshold below which individuals are a significantly greater risk of developing an opportunistic infection and an AIDS-defining illness. The CD4 count is a marker of T helper cell numbers and is normally above 800. Although there is often a transient mild drop on seroconversion, the count may be maintained for up to 10 years (and longer if on treatment) before it falls. If untreated, a linear fall in the CD4 count is associated with the development of a number of AIDS-defining diseases. The first is usually Pneumocystis carinii pneumonia (PCP) when the count is about less than 200–300. This is a reliable marker of an at-risk individual and is used as a clinical indication for PCP prophylaxis. Other AIDS-defining diagnoses are less predictable by CD4 count such as lymphoma, Kaposi’s sarcoma and TB.

Ciclosporin works by inhibiting release of interleukin (IL)-2 from T-cells, thereby inhibiting their proliferation.

Anti-HBs implies either recovery from hepatitis B infection or immunity following administration of the hepatitis B vaccine. Anti-HBc implies either previous or current infection

• IgM to HBcAg implies acute or recent infection within the last six months. Surface and core antigens (HBsAg, HBcAg) are detectable during acute infection. HBeAg (envelope) is a good marker of high infectivity, whilst anti-HBeAg suggests a patient who is less infective. Acute infection is also implied by IgM to HBcAg, whilst IgG to HBcAg suggests a previous infection. Virus clearance and recovery correlate with the disappearance of antigens and the appearance of antibodies. Previous vaccination is suggested by the presence of only anti-HBs.

Complement functions can be grouped into: opsonisation, inflammation and membrane perforation.

• Components C5–C9 comprise the membrane attack complex, which perforates cell membranes. • Components of the complement system are heat labile. • Activated fragments of C3 and C5 have proinflammatory actions and will activate mast cells. • Many extracellular organisms, including helminths have surface components that will activate complement by the alternative pathway and fix C3b on their surface, which aids phagocytosis.

C–C chemokine receptor 5 (CCR5) is a protein on the surface of white blood cells that acts as a coreceptor for HIV viral entry into T cells. CCR-5 has been identified as the coreceptor with CD4 for HIV entry into T cells. It was discovered after a group of prostitutes in Thailand failed to contract the disease despite high exposure to the virus. These individuals subsequently were shown to be deficient in the chemokine receptor. In vitro HIV infection could also be inhibited by excess chemokine ligand, which effectively competed for binding at the receptor. Novel strategies for CCR-5 receptor blockage in HIV are being examined.

Rheumatoid factor is an IgM autoantibody directed against IgG. It is found in a wide variety of autoimmune conditions and can be found in 1–2% of members of the population who do not have any autoimmune disease. Rheumatoid factor is often positive in patients with Sjögren syndrome. Sjögren syndrome is the third most common autoimmune disorder, after rheumatoid arthritis and systemic lupus erythematosus. Sjögren syndrome is much more common in women with a reported ratio of 9:1. Sjögren syndrome is characterised by dry eyes and dry mouth. It is a mild illness and care consists primarily of symptom treatment, eg artificial tears. High levels of antinuclear antibody and rheumatoid factor may be present as Sjögren is associated with other autoimmune diseases. Extra-glandular

Page | 88 involvement most commonly affects the kidneys, causing tubulointerstitial nephritis. There is also possible adenopathy with pleomorphic lymph node infiltrates. The secretory glands show a periductal lymphocytic infiltrate.

• Sjögren syndrome appears to be due to a complex interplay between genetic and environmental factors in a susceptible individual. Infiltration of salivary glands by T cells, in particular CD4 T helper cells seems to be key in triggering active Sjögren disease. • Sjögren syndrome can present with multi-system systemic symptoms. Management is mostly directed at controlling symptoms and disease control is achieved with use of immunosuppressive agents such as rituximab. Antibiotics are not typically used in the management of Sjögren syndrome.

Alpha-fetoprotein is the fetal equivalent of plasma albumin and is produced by the fetal liver, yolk sac and intestine. It can be elevated in hepatocellular carcinoma (up to 90% of cases), testicular teratoma, pancreatic, biliary, gastric and bronchial cancers.

Tumour-suppressor genes control abnormal cell proliferation and are inactivated in cancer. p53 is the gene most frequently involved in cancers (more than 50% of all cases). It is seen in the inherited Li– Fraumeni cancer syndrome, lung, breast, colon, and liver cancers as well as leukaemia and lymphomas.

Tumour protein 53 (TP53) is a tumour suppressor gene whose protein products cause cells to arrest in the G1 phase of the cell cycle

• c-Myc is an oncogene involved in lymphoma, breast and lung carcinomas. • erbB2 (Her2) is an oncogene involved in breast and ovarian cancers, gastric, bladder. • K-ras mutations are found in 80% of pancreatic cancers and in other adenocarcinomas (Lung carcinoma, leukemia).

CA-19–9 is elevated in 75–90% of patients with pancreatic carcinoma.

• Serum CEA is elevated in fewer than 5% of patients with Dukes’ A colorectal carcinoma, about 25% with Dukes’ B and 44% of patients with Dukes’ C colorectal carcinoma. • CA-125 is elevated in less than 50% of patients with stage-1 ovarian cancer. Tumour markers are not tumour specific, which is why the results can be difficult to interpret

Major histocompatibility complex class II molecules are found on antigen-presenting cells such as dendritic cells. These present peptides from antigens to CD4 T helper cells.

Leukotrienes consist of a set of peptides that are coupled to a metabolite of arachidonic acid. They are important in the acute inflammatory reaction producing vasoconstriction, increased vascular permeability, bronchoconstriction and chemotaxis. Mast cells are a source of leukotrienes and these play a key role in diseases such as asthma.

Blood transfusion causing ABO incompatibility would be a type 2 hypersensitivity reaction due to antibody-mediated cell destruction. Antibody-dependent hypersensitivity reactions are type-II reactions.

Graves’ is a type 5 hypersensitivity reaction (stimulating antibody) according to the Gell and Coombs classification. It involves IgG antibodies reacting with tissue receptors. Examples are Graves’ disease and

Page | 89 myasthenia gravis. Type- 5 reactions can be considered as a subtype of type 2 reactions, due to binding to receptors rather than to the cell surface

Macrophages possess Fc receptors specific for IgG. Dendritic cells are antigen presenting cells and therefore do not express Fc receptors.

Kinins are a group of polypeptides, proteins and enzymes that – amongst other functions - attract phagocytes. Kinins promote vasodilation and play a key role in helping to regulate systemic blood pressure.

Natural killer (NK) cells are large granular lymphocytes that recognise virus products on cell membranes. They perforate cell membranes and produce rapid nuclear fragmentation (apoptosis).

Tumour markers are most commonly used to monitor therapeutic responses. However, they may also aid in diagnosis, have a role in the detection of relapse and provide prognostic information.

Tumour markers are notorious for not having appropriate levels of specificity to be used for screening purposes in the majority of cases.

Skin-prick tests performed at neat and 1:10 dilutions. This is the recognised investigation to explore cause of an IgE mediated hypersensitivity reaction to anaesthetic agents.

B cells can undergo isotype switching, this is the process of switching through immunoglobulin classes.

B cells have surface MHC class II but plasma cells do not. Plasma cells are fully differentiated from B cells

Most mediators from mast cells are pre-formed, while others include cytokines and chemokines, or are newly formed following degranulation. Thromboxane is one of the newly formed lipid mediators following mast cell degranulation. Thromboxane causes vasoconstriction, platelet aggregation and bronchoconstriction.

Immunoglobulin G is a type of antibody that is the most abundant antibody in humans. It is released by plasma B cells via a variety of mechanisms in response to pathogens such as bacteria, viruses, fungi and so forth. IgG represents about 75% of total antibody levels

• Immunoglobulin M is an antibody produced by B cells and is the largest sized antibody. It is typically the first antibody to appears following initial exposure to an antigen. IgM represents about 9–10% of total antibody levels • Immunoglobulin A is an antibody that plays a key role in mucous membrane immune function and can fix complement via the alternative complement pathway. IgA is the most predominant immunoglobulin in mucous secretions. Immunoglobulin A represents about 15% of total antibody levels. • Immunoglobulin D is produced by immature B lymphocytes and is usually co-expressed by IgM. The function of IgD is still relatively unknown. IgD represents about 0.25% of total antibody levels. • Immunoglobulin E is synthesised by plasma cells and consists of two heavy chains and two light chains. It plays a significant role in type I hypersensitivity reactions. IgE represents about 0.05% of total antibody levels.

Page | 90

Post-streptococcal glomerulonephritis, an immune complex disease. Other infectious causes of immune complex disease include hepatitis B and C virus infection, cytomegalovirus (CMV) infection and malaria.

Intramuscular adrenaline would the treatment of choice in anaphylactic shock but hereditary angio- oedema does not respond to this treatment.

Angioedema condition needs to be treated with the inhibitor, which is present in fresh-frozen plasma or in recombinant preparations. The recombinant preparation is expensive and often is not available immediately, therefore the most practical treatment is fresh-frozen plasma. This condition is due to the deficiency of C1 esterase inhibitor. This causes uncontrolled activation of the classical pathway resulting in angioedema. Patients often present with oro-facial swelling which can cause respiratory obstruction, and abdominal pain. C2 and C4 often fall during an acute attack while C3 levels remain unchanged.

Immune suppression early (3 days) after burn injury is associated with glucocorticoidmediated T cell apoptosis and anti-inflammatory cytokine responses. These hormones are cytokines are driven by a compensatory anti-inflammatory response syndrome (CARS) following the systemic inflammatory response syndrome. Certain tissues appear to do well after transplantation even after modest HLA matching. Liver transplants are thought to express less major histocompatibility complex (MHC) class I and so HLA matching appears not to correlate with outcome. The cornea is relatively avascular so protecting it from rejection.

ABO antibodies are of the IgM subtype while antibodies to Rhesus antigens are of the IgG subtype. IgG antibodies to Rhesus antigens can cross the placenta during the last trimester, whereas ABO antibodies are IgM and hence cannot cross the placenta. The function of serum IgD is unknown. The transplacental passage of immunoglobulin only applies to IgG.

Graft-versus-host disease occurs when immunocompetent stem cells in the transplanted organ, eg bone marrow, attack the recipient. In practice this is prevented by irradiation of the recipient bone marrow and by T-cell depletion of the transplanted bone marrow.

Hypogammaglobulinaemia patients present with recurrent bacterial infections often sinopulmonary infections. Infection with Haemophilus influenzae and Streptococcus pneumoniae occurs with this condition, not Streptococcus pyogenes.

Cytotoxic T lymphocytes are able to recognise antigens presented to them from virally infected cells and trigger immune processes to kill these cells.

• IgA is involved in the protection of mucosal surfaces.

Interleukin-6 is released by macrophages and plays a key role in activation of lymphocytes as well as playing an important role in coagulation. IL-6 together with tumour necrosis factor are the most influential cytokines involved in coagulation activation.

Plasma cells are terminally differentiated B lymphocytes that secrete antibodies. Humoral immunity refers to antibody-mediated responses.

• The complement system is a part of the innate immune system that enhances the ability of antibodies and phagocytic cells to clear pathogens and damaged cells (opsonization), promote inflammation, and activate the cell-killing membrane attack complex.

Page | 91

• Cellular immunity is a protective immune response that does not involve antibodies but instead involves activation of cells and their respective cytokines and chemokines. • Innate immunity is the body’s first line of defence against antigens. It is non-specific and its role is not changed by repeated encounters with antigen. It includes lysozyme, complement, acute phase proteins, pathogens and natural killer (NK) cells. • Passive immunity develops immediately and is due to antibodies produced elsewhere, such as from maternal transmission to the fetus.

Type I hypersensitivity is an immediate hypersensitivity reaction due to immunoglobulin E (IgE)- sensitised mast cells and basophils releasing mediators of inflammation that produce the symptoms of allergy. Complement-mediated (type-2 hypersensitivity) and immune complex (type-3 hypersensitivity) reactions typically produce vascular damage, and therefore bullae and petechiae would predominate in the skin.

Type IV hypersensitivity can result in chronic granulomata and these reactions are based on T cell– antigen interaction, which results in recruitment of other cells to the site. The time course of the reactions is delayed compared with the types I and III, reaching a peak 24–48 h after injection or contact. The classic type IV responses are due to tuberculin and contact allergy (eg nickel, chromate, cosmetics and some drugs).

ABO incompatible blood transfusions are due to a type II hypersensitivity . This occurs due to antibody- mediated towards antigen attached to cells.

Hayfever, asthma and eczema can occur in combination in atopy. Although not entirely understood, this in part appears to be due to a partial type I hypersensitivity reaction.

Skin allergy testing is a method used for medical diagnosis of allergies. A microscopic amount of an allergen is introduced to a patient's skin by: Pricking the skin with a needle or pin containing a small amount of the allergen (‘prick testing’ or ‘scratch testing’). Applying a patch to the skin, where the patch contains the allergen. Release of histamine from mast cells produces the characteristic response in a skin test.

The complement system consists of over 20 component proteins and is involved in bacterial killing. Complement can be activated by the classical pathway, alternative pathway or via the lectin pathway. In the alternate pathway, C3 is activated directly by endotoxins, viruses, bacteria, fungi and other parasites. The complement system plays an important role in the coagulation cascade, particular with regards the contact activation pathway.

Parasitic infestations can be accompanied by a type-I hypersensitivity reaction. Liver flukes can be a cause of parasitic infection.

• Post-streptococcal glomerulonephritis can lead to a type-III (ie immune complex disease) reaction.

Pro-inflammatory cytokines such as interleukin-1, interleukin-6 and tumour necrosis factor-α are produced predominantly by activated immune cells.

• Anti-inflammatory cytokines such as interleukin-4, interleukin-10 and interleukin-13 are involved in the reduction of inflammatory reactions.

Page | 92

• TGF-β is involved inflammatory responses, often as an anti-inflammatory agent. It can act as a pro-inflammatory agent in certain instances, however the predominant effect is thought to be anti-inflammatory.

The infant that is at risk of developing haemolytic disease of the newborn. The mechanism involves passage of Rh-positive erythrocytes from the fetus into the blood circulation of a Rh-negative mother, which usually occurs near the time of delivery. The mother becomes sensitised and can begin producing anti-Rh antibodies. The firstborn child is generally not at great risk, because it is born before an appreciable level of immunoglobulin G (IgG) antibody is formed. However, in a subsequent pregnancy with an Rh-positive fetus, the mother can have a strong secondary immune response and IgG antibodies against Rh antigen can give rise to haemolytic disease of the newborn.

Interleukin-6 (IL-6) is a pro-inflammatory cytokine secreted by T cells and macrophages to stimulate the immune response to trauma especially burns or other tissue damage leading to inflammation. IL-6 is also a ‘myokine’, a cytokine produced by muscle and is elevated in response to muscle contraction

Macrophages live longer than neutrophils, which survive only 3 to 5 days outside the circulation. While some macrophages last only a few weeks, some survive much longer especially if they have ingested inert material such as carbon particles.

CEA can be raised in smokers and in conditions such as ulcerative colitis, pancreatitis and liver cirrhosis.

• The mean half-life is about 10 days. • It is not a useful diagnostic marker for cancers because it is increased in severe benign liver disease, inflammatory conditions (particularly of the gastrointestinal tract), trauma, infection, collagen diseases, renal impairment and smoking. It may however be used in detecting tumour recurrence, especially in the liver following colorectal cancer resection. • CEA is a water-soluble glycoprotein • It is elevated in less than 5% of patients with Dukes’ grade A colorectal cancer, 25% of Dukes’ B, 44% of Dukes’ C and approximately 65% of patients with distant metastases.

While the alternative pathway C3 convertase contains the components C3b and Bb, the classical pathway C3 convertase contains different proteins of the complement system – C4b and C2a

Activation of the classical complement pathway occurs in systemic lupus erythematosus (SLE) due to the large number of dsDNA immune complexes that form and are able to fix complement. It could also be described as a type III hypersensitivity reaction. Complement complexes are deposited in the kidneys and other organs, where they attract other components of the immune system and tissue damage ensues. Complement consumption is common in active disease in these cases.

• Systemic lupus erythematosus (SLE) typically results in activation of the classical complement pathway rather than the alternative pathway

Immunoglobulin A is an antibody that plays a key role in mucous membrane immune function and can fix complement via the alternative complement pathway.

Page | 93

• Immunoglobulin M is an antibody produced by B cells and is the largest antibody. It is typically the first antibody to appears following initial exposures to an antigen. IgM can fix complement via the classical pathway through the Fc portion of the immunoglobulin.

Steroids are useful to reduce the late response of a type 1 hypersensitivity reaction. They should be given intravenously/intramuscularly at a dose of 100–200 mg. Type 1 hypersensitivity is characterised by an early response over minutes to hours and a late response which is characterised by release of chemical mediators with effects which can last several hours to day

• Chemical mediators of early response: (released from mast cells) Histamine, serotonin, proteases, prostaglandin D2 , leukotriene C2 and D2 • Chemical mediators of late response: (products from metabolism of arachidonic acid) Prostaglandins, bradykinin, leukotrienes • Hydrocortisone blocks the generation of leukotrienes and prostaglandins, and hence prevents the late-phase reaction, often seen in asthma. Approximately 30% of deaths related to anaphylaxis occur because of this late-phase reaction.

CD8 T cells are cytotoxic and recognise cell-bound antigens only in association with class I MHC.

• CD4 T cells are T helper cells that recognise antigen only in the context of class II MHC.

Anti-mitochondrial antibodies (AMA) are associated with autoimmune hepatitis. AMA are detectable in serum before abnormalities in liver function and long before the onset of symptoms.

Serum sickness is a type 3 hypersensitivity reaction mediated by immune complex deposition. This can be more common with chimaeric antibodies, which are at least partly derived from animals. Typically it presents with rashes, itching, joint pains and fevers, within a few days of exposure. On discontinuing medication, the symptoms usually settle within about 5 days. Corticosteroids, anti-histamines and analgesia can be used as adjunctive treatment.

Allergic contact dermatitis is caused by a type 4 delayed hypersensitivity reaction to a chemical in contact with the skin. Initial sensitisation can occur 7–10 days after the first contact with a potent allergen. However, it is more usually a consequence of many months or years of exposure to small amounts of the allergen. Once sensitised, contact with the allergen can produce dermatitis within 24–48 h, and all areas of the body are equally susceptible.

Hyperacute rejection is a complement-mediated response in recipients with pre-existing antibodies to the donor (for example, ABO blood group antibodies).

Pyomyositis presents with pain, tenderness and swelling of muscles, and tends to occur in greater frequency in patients who are immunocompromised. Immunodeficiency will delay wound healing and increases the risk of postoperative complications such as wound infection and breakdown. HIV, if left untreated or inadequately treated, can progress to Acquired Immunodeficiency Syndrome (AIDS). Patients with AIDS have a high incidence of anorectal sepsis but undergo the same treatment for anorectal sepsis as non-HIV-infected patients. Pyomyositis presents with pain, tenderness and swelling, with the overlying skin being smooth and shiny, thereby mimicking an abscess.

Page | 94

Perforin is a cytolytic protein found in the granules of CD8 T cells and natural killer (NK) cells. During degranulation, perforin binds to the target cell’s plasma membrane and forms pores in it. These pores then allow granzymes – pro-apoptotic proteases – into the cell. Perforin is structurally similar to complement C9.

Endothelin-1 is a 21-amino acid polypeptide and is a highly potent vasoconstrictor. Levels of endothelin increase in times of stress and it plays a part in modulation of vascular tone, but it is not a vascular cell- adhesion molecule.

Which enzyme is responsible for the production of free radicals in neutrophils?

• NADPH oxidase is found in the wall of the endocytic vacuole. It acts as an electron donor to reduce oxygen to superoxide and hydrogen peroxide • Gluthatione peroxidase and superoxide dismutase, Vit E and catalase are antioxidant that are involved in the neutralisation of free radicals.

The antigen-binding site is situated within the variable region of the immunoglobulin structure, made up of both the heavy and light chains, specifically within the hypervariable regions

There are 11 members of the interleukin-1 family and most of these members are proinflammatory. Interleukin-1 is associated with mucus secretion from the intestinal tract.

• As for IL-4, IL-13 promotes helper T-cell proliferation and B cell IgE synthesis. IL-13 mediates mucus secretion within the respiratory tract in asthma. • IL-5 is involved with the activation and function of eosinophils

Pharma and Micro Glucocorticoids increase bone resorption through increased osteoclastogenesis, as well as by preventing bone formation. Patients taking systemic steroids, or who have Cushing’s disease, are prone to develop osteoporosis and may require monitoring and treatment for this.

• Calcitonin is a hormone released from the thyroid gland that has a small effect of inhibiting bone resorption by inhibiting osteoclast activity. • Oestrogen protects bone from demineralisation and explains the onset of osteoporosis in post- menopausal women. • Bisphosphonates protect bone from demineralisation and are used as a treatment for both osteoporosis and hypercalcaemia. A common example of a bisphosphonate is alendronate.

Tetanus status must be checked and prophylaxis given if required. The normal tetanus primary immunisation schedule is one dose given each at 2, 3 and 4 months as part of routine vaccination, followed by one booster 3 years later (pre-school booster), and a final booster a further ten years later (teenage booster). A tetanus-prone wound includes compound fractures, wounds or burns requiring surgical management that is delayed by more than 6 h or show a significant degree of devitalised tissue, deep penetrating wounds, wounds with foreign bodies, or wounds in patients with systemic sepsis. Below is a summary of the advice from Public Health England – please note, the main determinant of

Page | 95 need for vaccination is now whether the primary course of five doses has been correctly given and NOT whether a dose has been given in the past 10 years.

Rapamycin inhibit the action of interleukin 2

• Ciclosporin and tacrolimus act by inhibiting interleukin-2 production. • Azathioprine and mycophenolate mofetil act by inhibiting purine/pyrimidine biosynthesis.

Suxamethonium is metabolised rapidly by plasma cholinesterase meaning it has a short halflife. Its quick onset and short duration of action makes it popular for use in trauma situations in which rapid intubation may be required. Suxamethonium is a depolarising muscle relaxant. Non-depolarising muscle relaxants include pancuronium, vecuronium and atracurium.

• The action of the non-depolarising muscle relaxants, such as pancuronium, vecuronium and atracurium, are reversed by acetylcholinesterase inhibitors such as neostigmine. Suxamethonium is a depolarising muscle relaxant and is reversed rapidly by plasma cholinesterases

Tamsulosin is an α 1A adrenoceptor antagonist and shows some selectivity for the bladder and causes relaxation of the bladder neck and prostate capsule which may be useful in BPH patients.

• Finasteride is a 5α-reductase inhibitor and inhibits the conversion of testosterone into dihydrotestosterone in cells. It can be used in BPH to both ease symptoms and reduce further growth/enlargement of the prostate.

LMWH such as enoxaparin and dalteparin bind to anti-thrombin and accelerates its inhibition of factor Xa, which converts prothrombin to thrombin. Thrombin therefore is unable convert fibrinogen to fibrin, which is required in clot formation.

Page | 96

• Rivaroxaban is a direct factor Xa inhibitor. • Medications such as dabigatran exhibit their anticoagulant effect by directly inhibiting thrombin.

Adrenaline stimulates both alpha- and beta-receptors. Beta-receptor-mediated effects result in chronotropic and inotropic stimulation to the heart. Alpha-receptor effects stimulate peripheral vasoconstriction.

• At higher doses of adrenaline (adrenaline), alpha-mediated vasoconstriction reduces renal blood flow and can cause oliguria and precipitate acute renal failure. • Noradrenaline is predominantly an alpha-agonist causing potent peripheral vasoconstriction. • Dopexamine is an ionotropic agent, stimulating beta-2 receptors. It also stimulates peripheral dopamine receptors to increase organ blood flow. • Dobutamine acts directly on the heart, stimulating beta-1 receptors to increase contractility and cardiac output

Warfarin is a vitamin K antagonist and inhibits the synthesis of vitamin K-dependent clotting factors II (prothrombin), VII, IX and X.

Angiotensin II stimulates aldosterone synthesis and secretion through the activity of a specific receptor found in the zona glomerulosa. Angiotensin II is one of the most potent endogenous vasoconstrictor agents and inhibits renin release through a negative-feedback loop. Renin stimulates the formation of angiotensin I from angiotensinogen. Angiotensin I is converted to angiotensin II in the lung by angiotensin-converting enzyme (ACE). The renin– angiotensin system is activated by hypovolaemia. Renin is released by the juxtaglomerular cells of the kidney in response to decreased blood flow to the kidney secondary to global hypovolaemia. Renin catalyses the conversion of angiotensinogen to angiotensin I, which is then converted to angiotensin II by ACE, which is found in the lungs. Angiotensin acts to raise blood pressure through vasoconstrictive actions, stimulation of thirst, and through stimulation of aldosterone production and release to retain salt and water. ACE inhibitors can be prescribed as an antihypertensive, but have a common side-effect of dry cough. Angiotensin II receptor inhibitors are a second line for patients unable to tolerate ACE inhibitors

5-Fluorouracil This is a pyrimidine analogue, acting as a thymidylate synthase inhibitor. Antimetabolites interfere with DNA and RNA synthesis by inhibiting the formation of essential nucleic acids. These drugs have in common a structure that either resembles the natural nucleic acid or inhibits a vital enzyme that is needed to produce essential nucleic acids. Examples include methotrexate (dihydrofolate reductase antagonist), 5-fluorouracil, cytosine arabinoside, 6- mercaptopurine and 6-thioguanine. Doxorubicin is an antitumour antibiotic, vincristine is a vinca alkaloid, melphalan is an alkylating agent and paclitaxel is a taxane.

• Doxorubicin inhibits the function of topoisomerase II, this enzyme is important in the breaking down of DNA loops during cell replication. • Melphalan is an alkylating agent, it causes alkylation of guanine, which in turn disrupts DNA and RNA synthesis. • Paclitaxel binds to tubulin and interferes with microtubule function during mitosis, leading to cell apoptosis. • Vincristine binds to tubulin and interferes with microtubule function during mitosis, leading to cell apoptosis.

Page | 97

Cyclosporin can cause gingival hypertrophy in addition to nephrotoxicity, hepatotoxicity and neurotoxicity.

• Azathioprine can cause myelosuppression and acute pancreatitis. • Mycophenolate mofetil can also cause bone marrow suppression and patients require regular full blood counts upon starting it

Rituximab is a monoclonal antibody that can also be used for lymphomas. It can cause cardiac problems such as exacerbation of angina, heart failure and arrhythmias. All immunosuppressive medications carry important side-effects, it is important to be aware of these. Other important/frequently used immunosuppressives and their side-effects include:

Methotrexate – this is an antimetabolite acting as a folic acid antagonist to prevent synthesis of tetrahydrofolate. It is used is rheumatological conditions and numerous cancers. Common side effects include nausea, tiredness, oral ulceration. It can also lead to liver fibrosis and pulmonary fibrosis.

Infliximab – this is a TNFalpha blocker, used in several rheumatological conditions such as rheumatoid arthritis and ankylosing spondylitis. A particular risk of this medication is the increased susceptibility to reactivation of tuberculosis (TB) or new primary infection with TB

Co-amoxiclav is the most likely of these antibiotics to be associated with C. difficile diarrhoea. In addition, it is a commonly-prescribed first line agent for community-acquired pneumonia. Oral agents that are most likely to be associated with C. difficile include clindamycin, broad spectrum cephalosporins, fluoroquinolones (such as ciprofloxacin) and broad spectrum penicillins (such as co- amoxiclav). These can be easily remembered as they are all commonly used broad spectrum antibiotics beginning with ‘C’ with exception to Clarithromycin. Cephalosporins given intravenously are also likely to be associated with C. difficile. C. difficile is a common bacterium and is found as a part of normal bowel flora in 3–5% of the population. When it causes problems with acute infection, two toxins are produced (A is an enterotoxin and B is cytotoxic and results in bloody diarrhoea). Symptoms range from mild diarrhoea to severe colitis, when the bowel mucosa may be covered by a pseudomembrane. Antibiotics treatment of choice is with oral metronidazole or vancomycin, severe infection may require intravenous (IV) metronidazole. Complications can develop, such as toxic megacolon. Clinicians should maintain a high suspicion for C. difficile when treating elderly patients who have received broad-spectrum antibiotics.

• Oral Vancomycin is used for severe Clostridium difficile and is preferred to intravenous.

Metronidazole provides anaerobic cover. Its mechanism of action is through the inhibition of bacterial DNA synthesis.

• Trimethoprim is an inhibitor of folic acid synthesis. • Gentamicin interrupts protein synthesis by binding to the 30S subunit of the bacterial ribosome. • Meropenem is a broad spectrum antibiotic that inhibits bacterial cell wall synthesis. • Vancomycin inhibits bacterial cell wall synthesis.

Ciprofloxacin, belonging to a group called ‘fluoroquinolones’, is a broad-spectrum antibiotic that is active against both Gram-positive and Gram-negative bacteria. Ciprofloxacin is bactericidal. It blocks

Page | 98 bacterial DNA replication by binding itself to the enzyme DNA gyrase (topoisomerase), which is necessary for bacterial DNA replication.

• Sulfonamide Acts as a competitive inhibitor of the enzyme dihydropteroate synthetase • Macrolide (azithromycin and clarithromycin) Inhibition of protein synthesis by binding to the 50S subunit of the bacterial ribosome • Tetracyclin and Tigecyclin Prevent the amino-acyl tRNA from binding to the A site of the ribosome

Ketoconazole inhibits the biosynthesis of ergosterol by blocking the cytochrome P450 enzyme 14-alpha demethylase which prevents the conversion of lanosterol to ergosterol, which is a constituent of fungal- cell membranes.

• Amphotericin B and nystatin impair the permeability of the cell membrane by directly complexing with the membrane ergosterol. • Flucytosine goes through a number of conversion and activation steps before inhibiting RNA and DNA synthesis and so the building of essential proteins. • The target of griseofulvin is the microtubules. By binding to tubulin, griseofulvin interferes with microtubule function and prevents mitosis.

Cyanide poisoning may occur from the production of hydrogen cyanide from burning plastics. It is rapidly absorbed through the lungs and is distributed to all cells where it inactivates cytochrome oxidase and so inhibits cellular respiration and can result in loss of consciousness, neurotoxicity and convulsions. Patients with cyanide poisoning should be treated with oxygen initially. Dicobalt edetate is an antidote, but is associated with anaphylaxis and should only be used if cyanide toxicity is confirmed. Sodium nitrate and sodium thiosulfate are other options. Sodium bicarbonate is used in cases of severe toxicity to neutralise lactic acidosis.

Suxamethonium is a depolarising blocker. It acts by activating the postsynaptic nicotinic acetylcholine receptors at the neuromuscular junction, thereby depolarising the muscle fibre endplate. The resulting sustained depolarisation of the muscle endplate membrane potential initially activates neighbouring muscle fibre voltage-gated sodium channels (and therefore triggers the characteristic initial muscular fasciculations) before then locking the channels in their depolarised inactivated configuration. So further muscle action potentials are prevented and the block is established. It is useful in rapid sequence induction as it provides a flaccid paralysis suitable for intubation within 30–45 s, but also has a short half-life it’s clinical effect usually lasting between 5–10 min.

• Tubocurarine, vecuronium, pancuronium and gallamine are all non-depolarising agents that act as competitive inhibitors of acetylcholine on the postsynaptic nicotinic acetylcholine receptors of the neuromuscular junction. • An understanding of the principles of general anaesthesia is important for the MRCS examination. For emergency surgery (eg bowel obstruction or full stomach), anaesthetists want to minimise the time between loss of consciousness and muscle relaxation, which in most cases is a prerequisite for tracheal intubation. Suxamethonium is the prototypical rapid sequence induction drug, providing intubating conditions within 30–45 s. It is unique among currently used muscle relaxants in that it causes a depolarising block, ie a short period of muscle fasciculation followed by 5–10 min of flaccid paralysis (in most patients).

Page | 99

• Rocuronium is a rapid-acting non-depolarising muscle relaxant of the aminosteroid class. Used at a high dose it provides intubating conditions within 45–60 s and is therefore increasing in popularity for rapid sequence induction of anaesthesia. Atracurium and mivacurium are both non-depolarising muscle relaxants of the benzyl isoquinolinium class. Pancuronium is of the same class as rocuronium but much longerlasting. Its onset is slow (2–3 minutes) and therefore it is unsuitable for rapid sequence induction use. • Additionally it is important to be aware of ‘Sellick’s manoeuvre’. This is the application of cricoid pressure, which is required for rapid sequence induction in all non-fasted patients as further prevention against aspiration • Hexamethonium is a non-depolarising ganglionic blocker. Its action is to block transmission in the postsynaptic autonomic fibres at the ganglions of both the parasympathetic and sympathetic systems. It is not a frequently used drug at present, but was previously used as a treatment for hypertension. • Fentanyl is a synthetic opioid used for both analgesia and anaesthesia. It has a rapid onset of action, and a short half-life

The maximum recommended dose of lidocaine in adults is 200mg. Up until that a dose of 3 mg/kg means can be administered. In 1% lidocaine there is 100mg/10ml, therefore for a maximum of 20ml can be used.

Isoprenaline is a non-selective beta-agonist and as such has both chronotropic and inotropic effects, it is occasionally used in bradycardic states.

• Verapamil is a calcium-channel blocker (class IV antiarrhythmic agent). It slows conduction through the arteriovenous (AV) node and has a negative chronotropic effect. • GTN is a potent vasodilator – it can be used as an infusion in patients with ongoing ischaemic chest pain or in patients with acute heart failure. In the latter category its vasodilatory effect is used to decrease afterload and therefore have an indirect inotropic effect. • Furosemide, as well as being a loop diuretics, also has a vasodilatory action and so decreases afterload and giving an indirect inotropic effect.

Decreased acetylcholine levels in the synaptic cleft weaken the strength of muscle contraction and are used in the treatment of spastic muscle disorders like cerebral palsy. Botulinum toxin A blocks presynaptic acetylcholine release.

• Baclofen acts as a g-aminobutyric acid (GABA) agonist. GABA blocks both the presynaptic and postsynaptic release of acetylcholine • Diazepam potentiates the action of GABA at GABA type A (GABAA) receptors. GABA is the major centrally acting inhibitory neurotransmitter.

Sodium nitroprusside is a vasodilator that can be used to give controlled hypotension during anaesthesia.

Traditional teaching was that use of adrenaline in digits (and other extremities such as the nose or penis) is not recommended due to its vasoconstrictive action that may cause ischaemic necrosis. However, the medical literature does not back this practice, and a Cochrane review found limited evidence to either recommend or avoid the use of adrenaline in digital nerve blocks

Page | 100

• Lidocaine is a class Ib antiarrhythmic drug and can be given to treat cardiac arrhythmias. The British National Formulary recommends its use in the management of ventricular arrhythmias if amiodarone is not available. • Bupivacaine is more lipophilic than lidocaine and so has a longer duration of action. • Local anaesthetics, such as lidocaine, reversibly block activated sodium channels

Lidocaine toxicity begins with paraesthesia around the lips, tinnitus and/or visual disturbance. It progresses to dizziness, convulsions, cardiac arrhythmias and collapse.

The maximum dose for lidocaine with adrenaline is 7 mg/kg. Without adrenaline it is 3 mg/kg. In obese patients, the dose of lidocaine to be given should be based on ideal body weight. The maximum recommended dosages of 500mg or 200mg lidocaine, with and without adrenaline respectively.

There are two subsets of cyclo-oxygenases – 1 and 2, known as COX-1 and COX-2. COX-1 is an enzyme that is integral to the production of prostaglandins that stimulate mucous secretion from the stomach. This mucous protects the stomach from gastric acid. Naproxen inhibits both COX-1 and COX-2, so putting users of it at risk of peptic ulcers. The patient in this scenario has a perforated ulcer.

Diazepam is a long-acting benzodiazepine and acts at the GABA-A receptor to increase the effect of GABA, which is the major inhibitory neurotransmitter in the brain. An anterograde amnesia is produced by diazepam.

• Diazepam is primarily metabolised in the liver into active metabolites and the major active metabolite is desmethyldiazepam. • These effects are reversed by flumazenil, which has a strong affinity for the benzodiazepinebinding site. Naloxone reverses the effects of opiates • There are a number of different pathways/dosing regimens used in acute alcohol withdrawal, however, the general principle is that the benzodiazepine dose starts high and is gradually titrated down as the symptoms of withdrawal reduce.

The toxic effects of nerve gas (ORGANOPHOSPHATE) derive from its ability to inhibit the enzyme cholinesterase. The inhibition of this naturally occurring degradative enzyme engenders a massive accumulation of acetylcholine evoking an overstimulation of the acetylcholine receptors throughout the body. In the heart, specifically, acetylcholine released by the vagal nerve stimulates muscarinic receptors in the cells of the SA node. This results in the opening of potassium channels and hyperpolarisation of the SA node. It therefore takes longer for sodium leakage to cause the membrane potentials of these cells to reach the threshold required for an action potential. The rate of SA node firing is therefore decreased.

• Vagal tone does not impact on myocardial contractility, but affects the heart rate

A child with acute epiglottitis. Acute epiglottitis tends to present with a rapidly progressive sore throat and in the late stages, the patient may have associated inspiratory stridor. The child will be toxic, with a raised temperature and tends to sit forward in the tripod position and drool. The peak incidence is between 2–7 years. The commonest cause in children has classically been Haemophilus influenzae type B, accounting for approximately 90%, however, the incidence has decreased due to vaccination. This accounts for the relative increase seen in adults. Group A b-haemolytic streptococcus has now become a more frequent cause of epiglottitis.

Page | 101

A Browne’s tube is placed among the instruments to assess when sterilisation is complete. It will change colour from red to green. A Bowie–Dick test is used for sterile packs.

• On a standard cycle not all micro-organisms would be destroyed as Prions take a longer time and are difficult to destroy.( prions are misfolded proteins with the ability to transmit their misfolded shape onto normal variants of the same protein. They characterize several fatal and transmissible neurodegenerative diseases in humans and many other animals.) • The autoclave is more effective than dry heat as it has better penetration of materials and causes destruction of the cell wall of micro-organisms.

After splenectomy the patient receives one dose of Hib/MenC and MenB. Then after one month, one dose of MenACWY (4 strains of meningococcal bacteria) and a second dose of MenB.

• After splenectomy patients are at high risk of infection by encapsulated bacteria such as Streptococcus pneumoniae and Haemophilus influenzae. Therefore, following surgery, the patient should receive the pneumococcal vaccine after two weeks and then every five years. Functional antibody responses are better when the first vaccine is delayed by two weeks. • The highest risk of infection after splenectomy is in children up to 16 years of age and adults over 50 years of age and for the first two years post splenectomy. Patients should receive prophylactic antibiotics for the first two years but following that, if they are not high risk, the antibiotics could be discontinued with the appropriate counselling. • Patients should be counselled about the risks of travelling to a malaria endemic country. If they decide to go they should take antimalarials and steps to avoid mosquito bites (insect repellent, nets, long clothing). • Post-splenectomy patients are advised to receive the annual influenza vaccine

The Department of Health guidelines state that all known MRSA carriers should receive eradication therapy when admitted as in-patients – they also advise screening all patients admitted as an emergency or electively to high risk units [such as cardiothoracics, trauma and orthopaedics, vascular units, high dependency unit (HDU)/intensive care unit (ITU)], or if that patient was previously known to be MRSA positive. Decolonisation of asymptomatic MRSA carriers is not routinely required in the community. When eradication therapy is required this normally involves antibacterial skin wash, hair wash, and nasal cream – in patients with wounds it may also include a wound cream to apply. Protocols vary, so local trust policy should be followed.

All confirmed cases, including asymptomatically colonised patients, should be strictly isolated and should be barrier nursed to prevent further spread until MRSA eradication is complete.

MRSA, to reduce the risk of spread of infection the patient should be at the end of the list. Following the operation, the theatre then needs to be appropriately cleaned.

• All patients are routinely screened before surgery for MRSA to enable appropriate planning of the list and reduce the risk of spread of infection. • An urgent cancer operation should not be cancelled due to MRSA from a nasal swab. Many people are carriers of MRSA. It would be important to have a urine culture and administer prophylactic antibiotics guided by the urine culture.

Page | 102

N. asteroids is the commonest Nocardia species to affect humans, often causing pulmonary and disseminated infections. Malignancies, organ transplantation, high-dose corticosteroids or other immunosuppressive therapy, HIV and underlying pulmonary disease are predisposing factors. The infection tends to be subacute in nature, and progressive, with radiographs showing lobar or multilobar consolidation, often with abscesses or cavitation noted. Infection is via inhalation or by direct inoculation of the skin.

• Pneumocystis jiroveci causes pulmonary infection in immunocompromised patients, classically a feature of AIDS. Clinically the patient normally presents with dyspnoea and nonproductive cough, radiologically a fine, initially perihilar, reticular infiltrative pattern is seen. Abscesses are not usually a feature.

Patients with sickle-cell disease are prone to osteomyelitis due to areas of infarction and necrosis within the bone. In patients with sickle-cell disease, Salmonella along with Staphylococcus aureus are the commonest causes.

• Pseudomonas causing osteomyelitis is more common in intravenous drug users.

Hip septic arthritis; with raised inflammatory markers, white-cell count and a pyrexia. They likely would not be weight bearing or would hold the limb in a different position – typically flexed, abducted and externally rotated. The most common causative organism of septic arthritis in all paediatric age groups up to adolescence, including neonates, is Staphylococcus aureus.

• Neisseria gonorrhoeae is the most common cause in young, sexually active adults.

In splenectomy, the pneumococcal vaccine should be given every 5 years. However, in a planned procedure the initial vaccine should occur 4 – 6 weeks pre-operatively then every 5 years. In emergency splenectomy the vaccine would be 2 weeks post-operatively when functional antibody responses are better.

Phagocytosis is the process whereby cells – such as neutrophils and macrophages – ingest solid particles (involving adhesion of the target to the cell surface), facilitated by opsonisation. These cells fuse to form a phagosome, which further fuses with lysosomes, leading to the formation of a phagolysosome. It is within these phagolysosomes that intracellular killing of microorganisms occurs. The neutrophils produce microbicidal agents which act dependently or independently of oxygen. In oxygen-dependent mechanisms neutrophils produce hydrogen peroxide, peroxide anions, hydroxyl radicals and singlet oxygen. The oxygen-independent mechanisms involve lysozyme (muramidase), lactoferrin (which chelates iron and is required for bacterial growth), cationic protein formation and low pH inside phagocytic vacuoles. Release of lysosomal products from the cell damages local tissue by proteolysis via enzymes such as elastase and collagenase, activates coagulation factor XII, and attracts other leukocytes into the area. Some of the compounds released increase vascular permeability, while others are pyrogens, producing systemic fever by acting on the hypothalamus.

• Lysosomal products are produced and stored before phagocytosis, then released at the time of phagocytosis. • Bradykinin is an important mediator of acute inflammation – involved in non-vascular smooth muscle contraction, pain response and increased vascular permeability. It is not, however, part of the phagocytosis process.

Page | 103

• Following phagocytosis many phagocytes do indeed proceed to ‘antigen presentation’, whereby lymphocytes become activated to specific foreign antigens – macrophages and dendritic cells in particular fulfil this role. However, this is not part of the phagocytosis process itself, and occurs after the foreign particle has been engulfed and destroyed.

Chlorhexidine is bactericidal at high concentrations by causing bacterial cell membrane disruption. It is effective against Gram-positive organisms, with moderate activity against Gram-negative organisms.

• Alcohol acts by denaturing proteins and is effective against Gram-positive and Gramnegative bacteria with some antiviral activity

If hair must be removed you should use electric clippers with a single-use head rather than a razor which would increase the risk of infection. Hair removal should not be performed routinely as it can increase the risk of infection. It should be used only when necessary and immediately before surgery to give the lowest risk of infection

Candida albicans is the most common organism associated with balanitis. Balanitis is inflammation of the glans penis. When the foreskin (or prepuce) is also affected, it is called ‘balanoposthitis’. The most common cause is known as ‘non-specific balanitis’ – here lack of aeration and irritation, because of smegma and discharge surrounding the glans penis as a result of poor hygiene, causes inflammation and oedema and can predispose to candidate colonisation. Balanitis has a wide range of other causes including infective (discussed further below), contact dermatitis, psoriasis, seborrhoeic dermatitis, and neoplastic conditions such as squamous cell carcinoma. The most common causative organism is C. albicans, while the most common bacteria is Streptococcus pyogenes. Other infections are due to Gardnerella, Chlamydia, gonorrhoea, syphilis, Staphylococcus, herpes, Trichomonas, and scabies. While any man can develop balanitis, the condition is most likely to occur in men who have a tight foreskin that is difficult to pull back, or who have poor hygiene. Diabetes can make balanitis more likely, especially if blood sugar is poorly controlled. in patients with significant renal impairment, if you were to give this patient cefuroxime he would require a reduced dose

• Cefuroxime does not require any monitoring of levels. In hospital, gentamicin and vancomycin are commonly given antibiotics that require level monitoring according to hospital trust policy.

Staphylococcus aureus is thought to be responsible for 50% of prosthetic joint infections.

• Propionibacterium acnes is more commonly found after shoulder infection. • Neisseria gonorrhoeae is a Gram-negative diplococcus and can cause septic arthritis, more commonly in younger patients that are sexually active.

Quinsy, a peritonsillar , of which Streptococcus pyogenes is the most common causative organism. History may reveal a severe sore throat, possibly unilaterally sore, with fevers. Swallowing may be especially painful with the patient drooling their saliva as a result, with difficulty opening their mouth (trismus), and a ‘hot potato voice’. Additionally neck stiffness and headache are common, sometimes the patient will complain of torticollis. Examination reveals a unilateral bulging, often above the tonsils, resulting in uvula displacement and the patient may also have halitosis. Examination may be difficult due to the trismus, as is likely the case in this patient. Definitive management requires surgical intervention

Page | 104

– either needle aspiration, incision and drainage or tonsillectomy. Additionally, the patient will require IV antibiotics and usually IV fluids (either due to signs of sepsis, or because of dehydration due to difficulty in swallowing).

• Fusobacterium necrophorum, This is the most common organism cultured in the usually polymicrobial condition known as cancrum oris – or oral-facial gangrene – a condition that results in progressive necrosis and destruction of the soft and hard palate. This disease has its highest incidence in sub-Saharan Africa and is very rare in developed countries. Important risk factors include young age (infants and children), poverty, poor living standards and being immunocompromised.

In HIV needlestick injury, The modern 4th generation tests can be used at four weeks and look for both antibody and the p24 antigen.

• Antibody only tests are the most common test for HIV and can detect it from six weeks after the exposure, although for some people it can take up to three months for the test to become positive.

Anti-HBc appears at the onset of hepatitis B and persists for life. Therefore, it can indicate previous or current infection. As IgM anti-HBc is negative this means the patient does not have infection at present or recently. Anti-HBs is present either after vaccination or recovery from infection.

Sterilisation by steam can be used for all instruments that can withstand moist heat and is the most commonly used method. Moisture sensitive instruments such as non-stainless metals, instruments with fine cutting edges for example (that cannot tolerate corrosion) require dryheat sterilisation in hot-air ovens.

• Ethylene oxide sterilisation is restricted to use for heat-sensitive materials eg electrical components, rubber, plastics and lenses. It is used for sutures and single-use items. It has limited availability and is expensive (usually an industrial process). • Hot-air sterilisation is used to process airtight containers where the contained air cannot be sterilised by saturated steam. • Sterilisation by steam is done in an autoclave. Typically holding time is 3 min at 134°C, or 15 min at 121°C. It should be noted also that for autoclaving the surfaces must be adequately cleaned – debris will prevent steam from penetrating to the underlying micro-organisms. • Chemical sterilisation can be achieved with agents such as formaldehyde and ethylene oxide for cystoscopes. • Dry heat requires the instruments to be at 160°C for 2 h to kill all micro-organisms. Items such as cystoscopes will not withstand these high temperatures and would be damaged by the process.

Botulinum toxin reduces ACh availability at the motor endplate and therefore decreases motor activity. Its muscle-relaxing properties have a wide range of therapeutic uses, from glabellar lines, blepharospasm, spasticity, anismus, anal fissure to dystonia.

• it is produced by Clostridium botulinum – a Gram-positive, spore-forming, obligate anaerobe. • Clostridium botulinum classically has seven serotypes, A to G.

Page | 105

• Myasthenia gravis, a condition in which the ACh receptor is blocked by autoantibodies, would be expected to worsen with such treatment. This is because botulinum toxin blocks the release of ACh from the presynaptic axonal terminals of the motor endplate further decreasing ACh activity.

Klebsiella pneumoniae and Escherichia coli are two of the most frequently isolated pathogens in liver abscesses. They are both Gram-negative rods.

Most theatres use high-efficiency particulate air filters, as these filters help minimise the airborne spread of bacteria.

• Theatres should maintain at least 15 air changes per hour. • The corridor air is warmer than theatre air, this helps with the outward flow of air – warm corridor air rises and cooler theatre air replaces it. • Air in the operating theatre should have a positive pressure when compared with corridor air to prevent the flow of air from a non-sterile area to a sterile one. • Theatre air is introduced at ceiling level with exhausts near the floor to assist with the outward flow of air.

Hepatitis C is a single-stranded RNA virus. Hepatitis B is a double-stranded DNA virus.

Acute infectious hepatitis Following a needle stick injury, this should be reported according to local procedure, under the Reporting of Injuries, Disease and Dangerous Occurrences (RIDDOR) regulations to the Health Protection Agency (HPA). Testing of recipient and patient is necessary for hepatitis B, hepatitis C and HIV status. Acute infectious hepatitis, is an example of a ‘notifiable’ disease. A notifiable disease is one which must be reported to Public Health by law. There is an extensive list published by Public Health England (PHE) and when in doubt check with the local public health department. Others include mumps, tuberculosis, meningitis, malaria and measles.

• HIV is not a ‘notifiable’ disease. HIV should, however, be ‘reported’ if sustained as needlestick injury

Diphtheroids, along with coagulase-negative staphylococci, are normal skin flora.

• Viridans streptococci are normal flora in the upper respiratory tract and oral cavity but not the nostrils. • Enterococci are normal flora in the lower gastrointestinal tract. In the urinary system they have the potential to cause infection.

Sterilisation is the process defined as the complete destruction or removal of all viable microorganisms which includes spores and viruses. It is not to be confused with disinfection, the process to reduce the number of viable micro-organisms but not all spores and viruses. 2% glutaraldehyde is used most frequently to disinfect endoscopes but not used to sterilise them. 2% glutaraldehyde, activated with sodium bicarbonate, will disinfect within 10 minutes and sterilise within 10 hours; scopes are not left for the full 10 hours required for sterilization

Page | 106

Data interpretation and imaging ARDS is the most extreme manifestation of acute lung injury. It produces diffuse alveolar shadows on chest X-ray (CXR). It is important to be familiar of the diagnostic criteria of ARDS. The current definition is in the ‘Berlin Definition’ (2013). Key components:

1. Acute (less than 1 week) onset 2. Bilateral opacities consistent with pulmonary oedema on radiograph 3. Such changes are not secondary to cardiac failure of fluid overload. 4. Specific ventilatory requirements are a p (O2)/F O2(PF) ratio less than 300 mmHg with a minimum of 5 cmH2O positive end-expiratory pressure (PEEP). A normal PF ratio is approximately 500 mmHg and a PEEP of 5–15 cmH2O.

The final common pathway of ARDS involves neutrophil activation, and the release of inflammatory mediators and free radicals causing increased alveolar permeability. ARDS should be managed in an Intensive Therapy Unit (ITU)/High Dependency Unit (HDU) environment as rapid deterioration can occur requiring advanced circulatory support.

Ultrasound is the preferred initial modality in the investigation of right upper quadrant pain. It is more sensitive than hepatobiliary iminodiacetic acid (HIDA) scintigraphy and computed tomography (CT) in the diagnosis of acute cholecystitis. The most sensitive ultrasound finding in acute cholecystitis is the presence of cholelithiasis in combination with the sonographic Murphy’s sign.

• A normal gall-bladder is thin walled (3 mm) and pericholecystic fluid are secondary findings in acute cholecystitis. • Some of the less specific findings include gall-bladder distension and sludge in acute cholecystitis. • A dilated common bile duct (CBD) is a non-specific sonographic finding, causes include intramural, intraluminal or extraluminal obstruction: 1. intramural: strictures – benign or malignant, proximal to the area of CBD dilatation 2. intraluminal: stones in CBD, round worms 3. extraluminal: carcinoma of pancreatic head, peri-ampullary carcinoma, enlarged lymph nodes of porta hepatis

Technetium-99m ( 99Tc.m) mercaptoacetyltriglycine (MAG3) is used in dynamic radio-isotope studies of the function of a kidney over a period of time. Progressive uptake of the isotope may occur in a dilated, but not obstructed, system. Intravenous injection of furosemide can then help with rapid clearance to demonstrate if any obstruction is present. As MAG3 has a faster clearance and smaller volume of distribution, it is replacing di-ethylene-triamine-pentaacetic acid (DTPA) in diuretic renography.

• A non-contrasted computed tomography (CT) scan provides very limited information with regards to the urinary tract. CT urography, however, has become the modality of choice over the last decade. Common indications of CT urography include urinary calculus disease, haematuria, suspected renal or urothelial neoplasms. In cases due to the relatively higher dose of radiation exposed to a young girl, CT urography would not be appropriate as first-line of investigation.

Page | 107

• Investigation by intravenous (iv) urography (IVU) involves serial abdominal X-rays after iv injection of contrast, therefore allowing visualisation of the renal parenchyma, calyces, pelvis as well as the collecting system. IVU has been largely replaced by CT urography, as the latter gives both anatomical and functional information, although with a relatively higher dose of radiation. • Micturating cystourethrography is done to determine the presence of vesicoureteric reflux (VUR). Following introduction of contrast into the bladder via a catheter, dynamic X-ray studies allow the assessment of any reflux up the ureter. • Non-contrasted MRI is not used to evaluate the urinary tract. However, MR urography provides superior visualisation (in terms of tissue contrast resolution, sensitivity for contrast enhancement) as compared with CT urography or conventional IVU. Furthermore, as MRI does not utilise ionising radiation, it is favoured in patient populations such as pregnant women, children or patients who require repeated studies of the urinary tract. Currently, it is mostly indicated in children and pregnant women with dilated systems. However it is contraindicated in patients with contrast allergies.

Diethylenetriamine pentaacetate (DTPA) and MAG3 are forms of radioisotope scanning that allow assessment of the relative function of each kidney. It is important to note that MAG3 scanning is preferred over DTPA in neonates, patients with impaired function, and patients with suspected obstruction, due to its more efficient extraction.

All men at the age of 65 years are offered an ultrasound to screen for abdominal aortic aneurysm. At this time if an aneurysm is detected between 3–4.4 cm, then annual screening will take place with ultrasound. As the size of the aneurysm increases, the interval may shorten to 6 monthly surveillance

Duplex Doppler US is used far more readily in practice, however the gold standard test for deep vein thrombosis remains the ascending contrast venography.

Pancreatic pseudocysts can develop the following attacks of acute pancreatitis. They take 3– 6 weeks to mature and usually present with low-grade fever, leucocytosis, chronic abdominal pain and/or persistent rise in serum amylase. If very large they can cause gastric outlet obstruction. The investigation of choice is computed tomography (CT) of the abdomen, which enables the size and location of the collection to be determined. Often they are managed conservatively, but if very symptomatic they can be treated initially with percutaneous (with imaging guidance), internal or external drainage. If recurrence occurs, formal surgical drainage via cystogastrostomy or cystenterostomy are options.

One hip X-ray exposes a patient to 0.3 mSv, which is the equivalent of seven weeks of background radiation.

• Limbs and joints with the exception of the hip joint expose patients to the equivalent of less than 1.5 days of natural background radiation. • A single chest X-ray gives a radiation dose of 0.02 mSv equivalent to three days of background radiation. • 4 months of equivalent background radiation would be from one pelvic X-ray, one abdominal X- ray or one thoracic spine X-ray • A lumbar spine X-ray would be the equivalent of seven months of natural background radiation.

For paediatric patients, lifetime cancer risks are approximately doubled compared with adult patients.

Page | 108

• The UK average background radiation dose is 2.2 mSv per year, regional averages range from 1.5–7.5 mSv. • In patients 70 year and older, the lifetime cancer risks for various imaging investigations are approximately 1/5 of that associated with the 16–69-year-old population. • Chest X-rays expose patients to a small radiation dose of 0.02 mSv but the smallest exposure is with limb and joint (except hip) X-ray with gives less than 0.01 mSv • Abdominal XR is equivalent to 4 months of background radiation, 0.7 mSv radiation dose. • Lumbar spine XR is 1.3 mSv nearly double the abdo XR • A CT head scan gives a radiation dose of 2 mSv compared with 10 mSv given by a CT scan of the abdomen and pelvis • A CT scan of the abdomen and pelvis exposes a patient to 10 mSv of radiation, equivalent to 4.5 years of background radiation and associated with a 1 in 2000 increased lifetime additional risk of fatal cancer.

Page | 109

• A bone scan exposes a patient to the equivalent of two years of background radiation associated with an increased lifetime additional risk of fatal cancer of 1 in 5000. • Cervical spine X-ray is associated with a 1 in 200 000 lifetime additional risk of fatal cancer.

Four facial X-rays are usually taken: Waters’ view (posterior–anterior (PA) with cephalad angulation), Caldwell (PA view), lateral view and occipito-submentovertex view. Waters’ view tends to show all facial structures the best, hence Water’s view is helpful in the diagnosis of facial fractures but less helpful when planning a surgical intervention.

Traumatic aortic disruption, a time-sensitive injury, is a common cause of sudden death after an automobile collision or a fall from great height. A complete tear through the tunica intima, media and adventitia usually leads to rapid exsanguination and death. In aortic rupture survivors, immediate death is prevented due to the vascular continuity maintained by a pseudoaneurysm within an intact adventitial layer or a mediastinal haematoma. A large mediastinal haematoma may shift the trachea to the right. This condition has a variable course ranging from a relatively clinically silent period due to the contained rupture (pseudoaneurysm), to rupture of the pseudoaneurysm, exsanguination and death. Radiographic findings may include a widened mediastinum, obliteration of the aortic knuckle, deviation of the trachea to the right, obliteration of the space between the pulmonary artery and the aorta (obscuration of aorto-pulmonary window), depression of the left main stem bronchus, deviation of the oesophagus (nasogastric tube) and fractures of the first or second rib or scapula. False-positive and false-negative findings occur with each radiographic sign and, rarely (1–2%), no mediastinal or initial chest X-ray abnormality is present in patients with great vessel injury. While conventional digital subtraction angiography has historically been the gold standard investigation, CTA has now replaced it as the first line investigation, not only due to it being non-invasive but also on account of better delineation of the poorly opacifying false lumen, intramural haematoma and end-organ ischaemia.

Non-contrast CTKUB is now the first-line investigation in adult patients that are not pregnant when they present with acute renal colic. It is now recommended over intravenous urogram as it is a more sensitive test and has a similar radiation dose

• In Pregnancy the first line is US KUB

Bone scans are non-specific, and increased uptake of 99Tc -labelled phosphate is found in areas with active bone turnover – eg fractures, infection, local tumour or healing bone after necrosis. Therefore clinical correlation with history, examination and, if necessary, routine Xrays are essential.

• They show destructive lesions from 1.5 cm in diameter. • Bone scans produce images of the whole skeleton. These substances have a half-life of 6.2 h, and the radiation dose compares favourably with skeletal survey. They are a sensitive, cheap and rapid method for screening for skeletal metastases, but are in fact limited in the level of details, or to distinguish between metastases or non-malignant lesions, eg fractures, infection. • They show destructive lesions with bone loss of >50% of bone material. • Bone scans reveal metastases by detecting function changes in osteoblastic–osteoclastic activity and bony vascularity. This function is unaffected by the presence of bony sclerosis (or lysis), which is determined by the relationship between the osteoclastic and osteoblastic remodelling processes.

Page | 110

MRI is more accurate in visualising the anatomy of the diaphragm. It is very sensitive and specific and so is the investigation of choice. Surgical repair is necessary, even for small tears, because the defect will not heal spontaneously. Diaphragmatic injuries result from either blunt or penetrating trauma. A traumatic diaphragmatic rupture is more commonly diagnosed on the left side, perhaps because the liver obliterates the defect or protects it on the right side. In addition, the appearance of bowel, stomach or a nasogastric (nasogastric tube (NG)) tube is more easily detected in the left side of the chest. Right diaphragmatic ruptures are rarely diagnosed in the early post-injury period. The liver often prevents herniation of other abdominal organs into the chest. This, however, may not be representative of the true incidence of laterality and autopsy studies have revealed that leftsided and right-sided ruptures occur almost equally. Blunt trauma produces large radial tears measuring 5–15 cm, most often at the posterolateral aspect of the diaphragm. In contrast, penetrating trauma usually create only small linear incisions or perforations, which are less than 2 cm in size and may often take some time, even years, to develop into diaphragmatic hernias. If a laceration of the left diaphragm is suspected, an nasogastric tube (NG) tube should be inserted. If the tube appears in the thoracic cavity on the chest film, the need for special contrast studies can be eliminated. Minimally invasive endoscopic procedures (thoracoscopy) may be helpful in evaluating the injury to the diaphragm in indeterminate cases.

• Abdominal CT is usually not helpful because of its poor visualisation of the diaphragm.

Extradural haematomas may cross the mid-line, although subdural haematomas may not, due to the falx cerebri.

• Extradural haematomas are usually biconvex in appearance, whereas subdural ones are crescenteric. • Decreased attenuation of a haematoma is usually a feature of a chronic subdural haematoma (over 21 days old). In an acute bleed, the haematoma appears bright (hyperattenuation) on CT scans

An amoebic liver abscess may also cause blunting of the costophrenic angle due to direct irritation of the diaphragm, precipitating an inflammatory pleural effusion

4 signs of osteoarthritis; Joint space narrowing, osteophytes, subchondral cysts, subchondral sclerosis

Renal Cysts radiographical classification

• Bosniak 1 are simple cysts with a rounded wall. They are not malignant and require no further follow up. • Bosniak 2 lesions are minimally complex with a few thin septa or calcifications and are nonenhancing. They do not require further follow up. • Bosniak 2F lesions are minimally complex but have an increased number of septa when compared with Bosniak 2 that are minimally thickened with nodular or thick calcifications. They do require imaging surveillance although there are no guidelines on this, ~5% will be malignant. • Bosniak 3 lesions are indeterminate with ~55% malignant. They appear with thick nodular multiple septa and enhance • Bosniak 4 lesions are clearly malignant solid masses with large cystic or necrotic areas.

Level of Evidence

Page | 111

1a Systematic reviews (with homogeneity) of RCTs 1b Individual RCTs (with narrow confidence interval)

1c All or none RCTs

2a Systematic reviews (with homogeneity) of cohort studies

2b Individual cohort study or low quality RCTs (eg less than 80% follow up)

2c ‘Outcomes’ research; ecological studies

3a Systematic review (with homogeneity) of case–control studies

3b Individual case–control study

4 Case series (and poor quality cohort and case–control studies)

5 Expert opinion without explicit critical appraisal, or based on physiology, bench research or ‘first principles’.

Unpaired t test, As two separate groups are being compared at the same point in time, an unpaired t- test is used. The study is looking at a biological variable in a large sample – the data are assumed to be normally distributed.

• Paired t-tests are used to compare one group at two points in time, eg a patient group before and then after an intervention. • The Friedman test is used to compare three or more groups for statistical significance, if the data are non-parametrical ie do not follow a normal distribution. • Analysis of variance compares three or more groups for statistical significance, if the data are parametrical ie follow a normal distribution. • The Pearson correlation coefficient is a measure of strength of the linear relationship between two variables.

The Gaussian distribution is more usually referred to as the normal distribution and is characterised by being ‘bell shaped’ (unimodal) and symmetrical about its central value. In a Gaussian distribution the mean, median and mode are equal. This situation is characteristic of the bell-shaped distribution.

• The mean is used to calculate the central value in a normal distribution. However, in skewed data (non-parametric data) the median is a more accurate calculation for assessing the central value. The median refers to the middle point of the data. It is the value that halves the data with 50% of values above the middle point and 50% below. • Student’s t-test is an appropriate way of comparing the means of two groups of normally distributed values. When the same group of individuals is assessed on three or more occasions, the assessment times can be compared in pairs using either paired t-tests or Wilcoxon matched pairs rank-sum tests as appropriate. However the number of comparisons needed is large if there are many groups to be compared and the risk of a type I error (false positive result) increases rapidly. If the measure can be assumed to have a normal distribution, the assessment

Page | 112

times can all be compared simultaneously using a repeated measures analysis of variance (ANOVA) • The standard deviation (SD) is a measure of the spread of the data values. Here, 95% of observations lie within 1.96 SDs. Hence, 5% lie outside the interval, 2.5% less than the mean −1.96 SD and 2.5% greater than mean +1.96 SD.

In chronic renal failure, secondary hyperparathyroidism is an appropriate response to low calcium levels that occur due to low calcitriol levels, which is produced in the kidneys. Phosphate is high as it cannot be cleared from the kidneys. ALP is elevated in high turnover renal osteodystrophy due to increased osteoblast activity.

• Paget’s disease of the bone is characterised by excessive breakdown and remodelling of bone. It is characterised by excessive osteoclastic and osteoblastic activity. Blood results in Paget’s disease characteristically reveal an elevated ALP with normal calcium

MCV is calculated from haematocrit and RBC count for example: MCV (fl) = haematocrit × 1000/RBC (10 /µl) = 0.27 × 1000/3 = 90 fl, which is within the normal range.

Arrhythmias, hypotension, vasoconstriction, abnormal haemoglobin or pigments (eg bilirubin), movement, poor tissue perfusion and nail varnish can all affect pulse oximetry readings.

• Anaemia or polycythaemia will not affect the accuracy of the reading values, but these factors should impact the interpretation of these values. For example, an anaemic patient who has saturations of 99% will have fewer haemoglobin molecules, meaning that total oxygen content may still be significantly decreased although the saturations will not reflect this. Carboxyhaemoglobin will also cause a reading to be falsely high – pulse oximetry should be interpreted with caution in these patients.

Clinical governance involves seven pillars: 1. Service user, carer and public involvement. 2. Risk management. 3. Clinical audit. 4. Staffing and staff management. 5. Education and training. 6. Clinical effectiveness. 7. Clinical information.

Guidelines and standards need to be available so that the audit has a benchmark but these can be local, national or international. The audit cycle follows a set process, which begins with identifying set criteria and standards to be reviewed as outlined below. Audit cycle: Identifying an issue or problem Setting criteria and defining standards to be reviewed Collecting data Comparing performance against criteria and standards Implementing change Repeating the audit cycle.

One-way ANOVA and Student’s t-test are both examples of statistical analysis suitable for normally distributed data. Student’s t-test is used to examine the difference between sample means.

• Chi Square is used for non-parametric data. Observations are classified into mutually exclusive classes and the null hypothesis gives the probability that any observation falls into the corresponding classes. • Kruskall–Wallis is also used for non-parametric data. It is used for comparing two or more independent samples of equal or different sizes.

Page | 113

• Mann Whitney test is for non-parametric data to compare two sample means from the same population. It can be used as an alternative (non parametric equivalent) to the independent t- test when data are not normally distributed. • Friedman's two-way ANOVA is used for non-parametric data and aims to understand if there is an interaction between the two independent variables on the dependent variable • Kaplan–Meier curves with associated log-rank analysis provide a comparison of outcomes between different groups over a period of time. The log-rank test assesses the statistical significance of differences between the curve. • Pearson’s correlation coefficient analyses the strength of a relationship between two continuous variables. • Wilcoxson signed-rank test is a non-parametric test used to compared two related samples to assess whether their population mean ranks differ. For example, it can be used to compare repeated measurements on a single sample. • Spearman’s rank correlation coefficient s is a non-parametric measure of rank correlation which examines statistical dependence between the ranking of two variables.

Clearance (ml/min) is calculated using the formula (U × V)/P where U = urine concentration in mg/ml, V = urine production in ml/min, P = plasma concentration in mg/ml.

A type I error (false-positive result) occurs if the null hypothesis is rejected when it is actually true (eg the treatments are interpreted as having different effects when they do not).

• A type II error is a false-negative result and occurs if the null hypothesis is accepted when it is actually false (eg the treatments are interpreted as having equal effects when they are actually different). • Observer bias occurs when for example knowledge of the type of treatment being given or received alters the perception of its effect. For example a patient knowingly given a placebo for pain relief may over-rate their pain. • Selection bias occurs when patients are allocated to the interventions sequentially so it is known in advance which treatment the next patient recruited will receive so a clinician may pre-select a specific patient for a particular intervention

Results of screening programmes for cancer are limited by selection, length and lead-time bias. Lead- time bias occurs when screening advances the date at which diagnosis is made. This, therefore, lengthens the calculated survival time without necessarily altering the date of death. Length bias can also affect screening programmes. This is the tendency for screening to detect a disproportionate number of cancers that are slow-growing and have a better prognosis anyway.

• Interobserver bias occurs when the researcher subconsciously influences the experiment due to cognitive bias, in which judgement may alter how an experiment is carried out. • Population bias or sampling bias arises when some members of a population are more likely to be included in a study than others, leading to systematic differences between the population being studied and the general population • Detection bias occurs when a phenomenon is more likely to be observed for a particular set of study subjects. This may lead to a false inflation of a particular phenomenon being observed in a particular group because the study authors are more likely to look for it within a set group.

Page | 114

• Attrition bias comes about when there is a difference in withdrawals and dropout between study groups, which leads to incomplete outcome data. • Publication bias is a type of bias that occurs in published academic research, and occurs when the outcome of a study influences the decision whether to publish the data.

The t-test can be used to compare the means of two normally distributed samples. The samples do not have to be of the same size as long as they are both normally distributed and independent

The advantage of a cross-over study design is that the same individuals receive a sequence of different treatments therefore reducing the errors associated with individual differences. Most cross-over designs have a ‘balance’ in which all subjects should receive the same number of treatments and participate for the same period of time. each cross-over patient serves as his or her own control.

Anion gap = ([Na ] + [K ]) − ([Cl ] + [HCO ]) (all units mmol/l). There are many causes for decreased, normal or raised anion gap: a decreased anion gap (< 6 mEq/l): hypoalbuminaemia plasma-cell dyscrasia monoclonal protein bromide intoxication normal variant a normal anion gap (6–12 mEq/l): loss of bicarbonate (ie, diarrhoea) recovery from diabetic ketoacidosis ileostomy fluid loss carbonic anhydrase inhibitors (acetazolamide, dorzolamide, topiramate) renal tubular acidosis arginine and lysine in parenteral nutrition normal variant. an elevated anion gap (>12 mEq/l): methanol uraemia diabetic ketoacidosis propylene glycol isoniazid intoxication lactic acidosis ethanol ethylene glycol rhabdomyolysis/renal failure

Wounds only ever regain 80–90% of their pre-injury strength.

• Wounds of the skin, subcutaneous tissues, muscle, fascia and tendon only ever regain 80– 90% of their pre-injury strength.

• Scar maturation starts well before 6 months, and can last from months to years.

• The proliferative phase precedes the maturation phase.

• Type III collagen is produced during the early phase of wound healing, and is weaker than the later-appearing type I collagen (maturation stage). The inflammatory phase is dominated initially by neutrophils that appear in the first few hours and reach a peak in 24–48 h. Their main function is phagocytosis and cleaning the wound of bacteria and foreign material.

• The remodelling phase starts a few days after injury and can last up to 18 months. Depolarisation of the plasma membrane triggers passive release of calcium ions from the sarcoplasmic reticulum through specific channels along a gradient.

• Spontaneous contractions (tetany) of skeletal muscle may occur in response to low (not high) serum calcium levels. • There are two types of muscle fibres: slow, which produce prolonged, maintained contraction, and fast, which produce transient fatigueable contraction.

Page | 115

• Both thick and thin myofilaments are required to move along one another.

• Contraction is sustained until the calcium ions are actively transported back into the sarcoplasmic reticulum and relaxation occurs. By various negative-feedback mechanisms involved in homeostasis, a rise in ambient temperature is typically accompanied by increased respiratory rate.

• Hyperthermia causes cutaneous (peripheral) vasodilation.

• Hyperthermia causes a decrease in appetite.

• Hypothermia causes an increase in catecholamine secretion.

• Hyperthermia causes a reduction in skeletal muscle activity. There is an increase in the release of catecholamines in response to hypothermia.

• Bradycardia occurs due to depression of cardiac pacemaker cells.

• Peripheral vasoconstriction is a normal response to hypothermia and prevents heat loss.

• A fall in core body temperature is associated with a decrease in brain metabolism and activity.

• Therapeutic cooling is one of several methods that can be used to prevent secondary traumatic brain injury

• There is an increase in the release of thyroxine in response to hypothermia Glucose, amino acids and bicarbonate are reabsorbed along sodium in the early portion of the proximal convoluted tubule. Glucose is removed from the urine by secondary active transport.

• The glomerulus is the network of the capillaries at the beginning of the nephron. It produces a selective ultrafiltrate of the blood. The rate of glomerular filtrate is called the glomerular filtration rate (GFR). The glomerulus does not absorb particles, but is a filter to the proximal convoluted tubule.

• The descending loop of Henle is permeable to water due to the presence of acquaporin-1 but the ascending limb is impermeable to water. Na , K and Cl are co-transported out of the ascending limb, which encourages the movement of water out of the descending limb, making the fluid in the descending limb hypertonic. In the ascending limb it becomes more dilute because of the movement of Na and Cl out of the lumen and at the top is now hypotonic to plasma. The loop of Henle is not involved in glucose resorption.

• The distal convoluted tubule is relatively impermeable to water and is involved in the continual removal of the solute, making the tubular fluid further diluted. It is not involved in glucose resorption.

• The collecting ducts are under the influence of antidiuretic hormone (ADH) (vasopressin) to control water absorption. In the absence of ADH, water can enter the urine, promoting diuresis. In the presence of ADH, aquaporins allow the collecting duct to become permeable to water allowing reabsorption and inhibiting diuresis. Active transport is required to polarise the cell membrane and to maintain the resting Na gradient.

Page | 116

• Active transport occurs at the expense of energy (adenosine triphosphate, ATP).

• Carrier proteins are required to transport molecules across the cell membrane.

• Active transport requires either hydrolysis of ATP (primary active transport), or a pre-existent osmotic gradient, such as the resting intracellular/extracellular Na gradient (secondary active transport or coupled transport). An example of this latter type is when glucose is resorbed glucose against its concentration gradient in the kidney. The energy to drive the reabsorption is from the passage of Na along the pre-existing concentration gradient. Immediately after surgery there is a reduction in thyroid hormone secretion. This would be expected to normalise within the first few days.

In Zollinger–Ellison syndrome there is excessive secretion of gastrin.

Gastrin stimulates pepsinogen secretion by chief cells.

In the secretory phase, the endometrium is maintained under the influence of progesterone.

• The secretory phase of the menstrual cycle begins after ovulation occurs. Within the secretory phase, progesterone peaks 7 days after ovulation (day 21).

• Progesterone levels can be used to confirm ovulation.

• There is a basal body temperature increase (0.5°C) in the secretory phase.

• Menstruation follows the secretory phase. Hexose sugars (eg, glucose) are transported by facilitated diffusion. (across the placental barrier)

Mitosis, not meiosis, occurs in the M phase of the eukaryotic cell cycle

• DNA is synthesised in the S phase. The S phase occurs after G1 and before G2.

• The G1 phase (part of interphase) represents synthesis and growth of proteins required for DNA synthesis.

• The G1 phase determines the variability of the cycle length. quick Sequential (sepsisrelated) Organ Failure Assessment (qSOFA) score

• The qSOFA assessment is a quick screening tool with three parameters to identify high-risk patients with sepsis outside of intensive care. A point is given for each of the following; Glasgow Coma Score (GCS) less than 15, respiratory rate ≥22 breaths/min (normal 12–20 breaths/min) and systolic BP ≤100 mmHg

• Blood parameters do not form part of the qSOFA assessment. In the extended SOFA assessment used for intensive care, patients’ platelets, bilirubin and creatinine are all considered. White blood-cell count was previously used as part of the SIRS criteria to identify septic patients. Normal value 6000–15 000/mm (6–15 × 10 /l).

• qSOFA is used as a predictor of mortality and not to identify sepsis and so temperature does not form part of its assessment. A temperature of greater than 38°C formed part of the systemic

Page | 117

inflammatory response syndrome (SIRS) criteria, which are no longer recommended for the diagnosis of sepsis. Normal value 36.1–37.2°C. In malnourished patients, the commencement of feed puts the patient at risk of refeeding syndrome, which can cause cardiac complications including cardiac arrhythmias resulting in sudden cardiac death. Refeeding causes increased glucose availability, increased insulin secretion and decreased glucagon secretion. This results in the synthesis of glycogen, fat and protein that requires phosphate and magnesium. Additionally, insulin stimulates the absorption of potassium. As a result, there are decreased levels of phosphate, potassium and magnesium.

The ova are developed from the primitive germ cells, which are embedded in the substance of the ovaries.

• Human ova are extremely small, measuring about 0.1–0.2 mm in diameter and are enclosed within the egg follicles of the ovaries. By the enlargement and subsequent rupture of a follicle at the surface of the ovary, an ovum is liberated and conveyed by the uterine tube to the cavity of the uterus.

• Each primitive germ cell gives rise, by repeated divisions, to a number of smaller cells called oogonia, from which the ova or primary oocytes are developed. Unless it is fertilised it undergoes no further development and is discharged from the uterus, but if fertilisation take place it is retained within the uterus and develops into an embryo. Proofreading’ refers to the process by which genetic errors are corrected. DNA polymerases are able to detect a base pair mismatch and replace it with an appropriate match, allowing replication to continue.

• Replication refers to the normal process of the DNA sequence being copied, but this term does not imply error.

• Recombination refers to the process by which the combination of two DNA sequences generates a unique, new sequence. This process occurs in meiosis.

• Retrotransposition is transposition via an RNA intermediate (transposition is the movement of a genetic element from one site to another in a DNA molecule).

• Splicing is when the primary transcript of mRNA is processed and introns are removed, with the remaining exons joined together. A mature transcript of mRNA is formed. Stimulation of either β1- or β2-receptors activates adenylate cyclase.

• While adrenaline stimulates β1- and β2-receptors, noradrenaline predominantly stimulates α1- receptors. β2-receptor stimulation causes systemic vasodilation and bronchodilation. This underpins the mechanism of salbutamol

• β-Adrenergic responses typically result from increased production of cAMP. In ileus the gastrointestinal tract is not functional and the patient is unlikely to absorb any enteral feed. When commencing TPN it would be important to closely monitor magnesium, phosphate and potassium levels, as patients that have had a prolonged period of starvation are at risk of refeeding syndrome.

Page | 118

At 1 hour post-injury, the wound healing process will be in the inflammatory phase and neutrophils will be a predominant cell type. At 1 hour post-injury, the wound healing process will be in the inflammatory phase.

• Macrophages increase on days 1–2 and their role in the acute inflammatory phase, includes phagocytosis and growth factor secretion

• Myofibroblasts increase at days 2–4 post-injury.

• Fibroblasts tend to migrate to the wound on days 2–4, and allow for synthesis of extracellular matrix components. Cytoplasmic proteins provide the main contribution to pH buffering of the intracellular compartment.

• The bicarbonate system is an intravascular buffer: CO + H O « H CO « H+ + HCO catalysed by the enzyme carbonic anhydrase.

• In the interstitial (ie extracellular and extravascular) compartment, the bicarbonate system is the main mechanism of pH buffering.

• The phosphate buffer system (HPO + H « H PO ) acts in the intravascular space The most important physiological function of the is To transport fluid and proteins away from the interstitium to the blood. In the absence of a lymphatic system, the interstitial fluid protein concentration would increase greatly, causing widespread extracellular oedema.

In hereditary spherocytosis there is a defect of the structural proteins in the red cells. It is inherited as an autosomal-dominant trait.

• Haemophilia is a sex-linked recessive disorder. Haemophilia A is caused by a deficiency clotting factor VIII. Haemophilia B is caused by a deficiency of clotting factor IX.

• Beta-thalassemia is autosomal recessive.

• Cystic fibrosis shows recessive inheritance. The gene defect is on chromosome 7 and is carried by approximately 1 in 20 Whites.

• Trisomy 21 occurs when an extra chromosome 21 is present and can occur due to a nondysjunction or mosaicism. It is not an inherited disease Apoptosis (programmed cell death) is characterised histologically by nuclear condensation, fragmentation and by defects in the cell membrane. Programmed cell death is a feature of all cells and is associated with a variety of enzymes including endonuclease, granzyme B and ICE (interleukin (IL)-1β- converting enzyme)

• The cell membrane remains intact in the process of breaking down cells into apoptotic bodies along the cascade of programmed cell death.

• Necrosis differs from apoptosis. Necrosis is not programmed cell death, but is rather is pathological cell death due to exogenous injury • Physiological examples of apoptosis include embryological development and cellular turnover

Page | 119

• Necrosis is an inflammatory process whereas apoptosis is not. Diabetic ketoacidosis results in a metabolic acidosis with a high anion gap as a result of acid production. Anion gap is calculated by (Na + K ) − (HCO + Cl )

• Metabolic acidosis can result from excess H+ production, impaired H+ excretion or excess loss of base. With excess base loss, as would happen with a high output stoma, the anion gap would be maintained not increased. The anion gap is normal because the decrease in HCO is matched by increased Cl . In anaphase the paired chromosomes separate and are dragged to the opposite sides of the cell by the microtubules.

• In cytokinesis, an actin fibre forms around the centre of the cell and contracts, pinching it into two daughter cells, each with 23 pairs of chromosomes.

• Interphase comprises phases G1, S and G2 when the cell is in preparation for division.

• Metaphase follows on from prophase and prometaphase. The spindle fibres align with chromosomes along the metaphase plate.

• In telophase, the chromatids arrive at the opposite poles of the cell and disperse after new nuclear membranes are formed. In neurogenic shock there is a loss of sympathetic tone below the lesion that causes bradycardia and loss of vasomotor tone resulting in hypotension. Neurogenic shock does not respond to fluids as the primary problem is loss of sympathetic tone.

Fibroblasts appear in the proliferative phase of wound healing and produce collagen.

• There is an influx of inflammatory cells much earlier than the proliferative phase. This occurs in the inflammatory phase of wound healing.

• Complement activation is seen in the initial inflammatory phase.

• There is remodelling of the extracellular matrix in the maturation (or ‘remodelling’) phase that follows the proliferative phase By definition, capillaries are only one endothelial cell thick and do not contain smooth muscle.

• Hydraulic conductivity (Lp) is the filtration rate per unit of pressure across the membrane. As Lp increases, the net filtration will increase also.

• A larger surface area will allow a larger net filtration. Atrial and ventricular arrhythmias can result from hypothermia; asystole and ventricular fibrillation have been noted to begin spontaneously at core temperatures below 25-28°C.

• Hypothermia causes a shift of the oxygen dissociation curve to the left, so increasing the oxygen content of blood at a given tension.

• Blood viscosity increases as body temperature falls.

Page | 120

• Temperature regulatory mechanisms fail at 30°C and the condition is often life threatening after this point. Hartman composition: Na 131 mmol/l; K 5 mmol/l; Ca 2 mmol/l; Cl 111 mmol/l; HCO3 29 mmol/l; glucose 0 mmol/l; osmolarity 280 mOsm/kg

• HCO3 is actually in the form of lactate and is metabolised to bicarbonate in the liver Fetal wounds have reduced pro-inflammatory signals so the inflammatory infiltrate during the early phase of healing is inhibited. Fetal fibroblasts synthesise more total collagen.

The terminal ileum is responsible for the reabsorption of bile salts and vitamin B12. Bile salt malabsorption can cause chronic diarrhoea.

Each particle of LDL contains one molecule of apolipoprotein B-100, the ligand for the LDL receptor.

• The major fat in low-density lipoproteins is cholesterol.

• Their concentration is determined mainly by the rate of hepatic synthesis and peripheral clearance; dietary saturated fat intake correlates more strongly with LDL-cholesterol concentration than dietary cholesterol intake.

• It is HDL-cholesterol that is involved in reverse cholesterol transport.

• LDLs are formed from VLDL (very-low-density lipoprotein) and IDL (intermediate-density lipoprotein) by successive removal of triglyceride and modification in the circulation The CTZ is outside the blood–brain barrier, in the floor of the fourth ventricle.

• The vomiting centre is present in the reticular formation of the medulla.

• The main receptors are dopamine, histamine and serotonin.

• 5HT3 antagonists (eg ondansetron), not agonists, are effective in controlling vomiting. The a-wave on a JVP waveform corresponds to right atrial contraction and would be absent in patients with atrial fibrillation.

• Large a-waves occur when there is resistance to emptying of the right atrium for example in tricuspid stenosis

• The y-descent represents ventricular filling and this would be slow in tricuspid stenosis. Dobutamine acts on β1 receptors and acts to increase cardiac contractility and cardiac output. Which make it a good inotrope in the ITU.

• The intracranial volume in the adult is approximately 1700 ml; consisting of 1400 ml of brain tissue, 150 ml of cerebrospinal fluid and 150 ml of blood.

• The Monroe–Kellie doctrine states that the skull is a rigid structure with three components (brain tissue, CSF and blood). If the volume of one of these components increases there is a reciprocal decrease in another to maintain a constant ICP.

Page | 121

• Intracranial volume is primarily composed of brain, CSF and blood. There is no lymphatic system within the brain

• Normal intracranial pressure is typically considered to be between 5–15 mmHg.

• The hypothesis only applied when the skull is fused. A positive nitrogen balance is required in growth. Nitrogen balance is a measure of nitrogen input minus nitrogen output. Positive nitrogen balance is associated with periods of growth, hypothyroidism, tissue repair and pregnancy. This means that the intake of nitrogen into the body is greater than the loss of nitrogen from the body, so these is an increase in the total body pool of protein. This is the equivalent of stating that a patient is ‘anabolic’.

• Following a fracture there is a negative nitrogen balance due to increased demands from the reparative process. Trauma induces a hypermetabolic state, which leads to a persistently negative nitrogen balance (as the total protein intake is insufficient to meet the body’s demands).

• In exercise the metabolic rate and nitrogen demand of the body increases. Dietary protein is in a constant state of flux in the body, being broken down into amino-acids, transformed into other compounds, and sometimes reassembled into other proteins. They are also used up as energy, a mechanism that is more prevalent when energy consumption is high (such as in sustained exercise). Muscle protein can thus become a source of energy, thus leading to a negative nitrogen balance.

• Major surgery results in iatrogenic injury and inflammation that increases the metabolic rate, increased urea production and so the nitrogen demands of the body.

• Following sepsis there is a negative nitrogen balance due to the increased requirement from the inflammatory processes Sperm capacitation refers to the physiological changes spermatozoa must undergo in order to have the ability to penetrate and fertilize an egg

• Glycoprotein molecules coating the surface of the sperm cell are solubilised by uterine fluid in the process of capacitation. Capacitation enhances motility and allows the head to penetrate an egg. Capacitation requires increased energy metabolism.

• Capacitation enables the acrosome reaction (The acrosome is a vesicle positioned close to the plasma membrane at the tip of the sperm's head. This vesicle contains soluble proteolytic enzymes and inner membrane proteins. The acrosome reaction is the exocytosis of the acrosome, or the fusion of the acrosomal membrane with the sperm's plasma membrane. This process releases the acrosomal contents to the outside of the sperm and exposes the inner acrosomal membrane proteins on the sperm's outer plasma membrane. The function of the acrosome reaction is to help the sperm get through the egg's protective coat and to allow the plasma membranes of the sperm and egg to fuse. ) • Capacitation does not affect follicle-stimulating hormone (FSH) release or LH The intracellular fluid is about two-thirds of total body water, that is 30 litres out of a total body water content of 45 litres

Page | 122

• The plasma volume measures about 3 litres. The total blood volume is around 5 litres.

• Approximately 400 ml of water is lost daily from the lungs Apoptosis is energy dependent, necrosis is not. In apoptosis cell integrity is preserved. Apoptosis may be either physiological (programmed cell death) or pathological. Necrosis is always pathological.

Wound healing

Endothelial cells appear around days 3–5 and migrate into the wound to form new capillaries.

• Neutrophils appear at day 1-2 and are part of the initial inflammatory response. Macrophages appear around days 1–2 and are involved in the inflammatory phase.

• Myofibroblasts appear around days 2–4 and are important in contraction of the wound.

• Platelets appear immediately and form the platelet plug to achieve primary haemostasis. The flow of fluid can be described by Poiseuille’s law, which demonstrates that flow is related to viscosity, pressure, length and diameter of tubing. Maximum flow would be achieved by a short length, large diameter, low viscosity fluid and high pressure

RNA synthesis occurs in both the G1 and G2 phases. The tumour-suppressor gene p53 is known as ‘the guardian of the genome’ and prevents a cell from entering the S phase of the cell cycle (DNA synthesis)

This arterial blood gas represents a type 2 respiratory failure (‘ventilatory failure’). This has probably been caused by the administration of high flow oxygen in a patient with COPD, resulting in respiratory depression and subsequent hypercapnia. The carotid body chemoreceptors increase ventilation and are activated in response to reduced p (O2 ) and/or elevated p (CO2 ) or H .

• Aortic baroreceptors are stimulated by arterial stretch. An increase in baroreceptor discharge occurs in an attempt to lower blood pressure Mitochondria have the ability to regenerate and replicate spontaneously.

Cerebral blood flow accounts for about 15% of the cardiac output at rest, equivalent to around 700 ml/min. Autoregulatory mechanisms keep this relatively constant despite changes in blood pressure.

• As part of the autoregulatory response of cerebral blood flow both hypercapnia and hypoxia will increase blood flow through cerebral vasodilation.

• Isoflurane increases CBF through cerebral vasodilation. Pulmonary vascular resistance increases at high altitude. Pulmonary arterioles constrict in response to hypoxia (known as the Euler–Liljestrand mechanism), to re-direct blood flow to well ventilated lung regions.

• Pulmonary vascular resistance during space flight would, if anything, decrease as blood flow becomes more uniform.

• With exercise, pulmonary arterial pressure tends to rise causing recruitment and distension of pulmonary vessels leading to a fall in pulmonary vascular resistance.

Page | 123

• Inspiring 100% oxygen would also cause the pulmonary vascular resistance to drop.

• Aminophylline is a bronchodilator and lowers pulmonary vascular resistance. The skin receives about 5% of the cardiac output.

Calcium influx opposes the efflux of potassium so causing the constant membrane potential seen in the plateau phase (phase 2). The plateau phase of the action potential in cardiac muscle (principally due to calcium influx) maintains the membrane at a depolarised potential for as long as 500 ms. The result is that the cell membrane is refractory to further stimulation, allowing the impulse that originates in the sino-atrial node to propagate throughout the entire myocardium just once, thereby preventing re-entry arrhythmias.

Cardiac action potential: phase 0: rapid sodium influx phase

1: small efflux of potassium phase

2: slow influx of calcium phase

3: efflux of potassium phase

4: sodium/calcium efflux, potassium influx.

Cardiac index = cardiac output/body surface area. This formula allows comparison of cardiac function corrected for size of the individual.

The ST segment connects the QRS complex and the T-wave. This segment ordinarily lasts about 0.08 s and is usually level with the PR segment.

In a denervated or transplanted heart the vagus nerves have been severed and therefore do not exert a ‘braking’ effect on the cardiac rate. This leads to an elevated resting heart rate, with a sinus tachycardia

Aortic balloon pumps inflate during diastole, increasing coronary artery perfusion which only occurs in diastole. In addition, it deflates in systole increasing forwards blood flow by reducing afterload through a vacuum effect. The effect is to increase cardiac output and systolic blood pressure. It is commonly used in post cardiothoracic surgery and in weaning patients from cardiopulmonary bypass after continued perioperative injury to myocardial tissue.

Cardiac output = stroke volume × heart rate. The cardiac output increases during pregnancy and reaches its maximal for the pregnancy by approximately 20 weeks’ gestation.

• Cardiac output is decreased on standing from a lying position. The QT interval is measured from the beginning of the QRS complex to the end of the Twave. A normal QT interval is about 0.40 s

• prolonged QT interval. QT prolongation is associated with hypokalaemia, hypocalcaemia, hypothermia, drug treatment, head injury, and the congenital long QT syndrome

• If QT interval is prolonged. It would be useful to review the patient’s prescriptions as causative agents include amiodarone, tricyclic antidepressants, erythromycin, ketoconazole and quinine.

Page | 124

Histamine causes reduced peripheral resistance (via vasodilatation and increased capillary permeability), resulting in increased cardiac output (see formula below). Systemic vascular resistance = ((mean arterial pressure – central venous pressure)/cardiac output) × 80

• Cardiac output is decreased upon standing from a lying position as venous return to the heart is suddenly reduced. According to Hagen–Poiseuille law, the radius of the tube is the most important determinant of flow through a blood vessel. Doubling the radius of the tube will lead to a 16-fold increase in flow at a constant pressure gradient. Hagen–Poiseuille law states that the flow through a vessel is:

• directly proportional to the pressure head of flow

• directly proportional to the fourth power of radius

• inversely proportional to the viscosity

• inversely proportional to the length of the tube. The radius of the tube is therefore the most important determinant of flow through a blood vessel. So, doubling the radius of the tube will lead to a 16-fold increase in flow at a constant pressure gradient. The implications of this are several fold:

1. First, owing to the fourth power effect on resistance and flow, active changes in radius constitute an extremely powerful mechanism for regulating both the local blood flow to a tissue and central arterial pressure. The arterioles are the main resistance vessels of the circulation and their radius can be actively controlled by the tension of smooth muscle within its wall. 2. Second, in terms of intravenous fluid replacement in hospital, flow is greater through a peripheral cannula than through central lines. The reason is that peripheral lines are short and wide (and therefore of lower resistance and higher flow) compared to central lines, which are long and possess a narrow lumen. A peripheral line is therefore preferential to a central line when urgent fluid resuscitation, or blood, is required. Atrial flutter is a supraventricular tachycardia caused by a re-entry circuit within the right atrium. This ECG trace is regular (unlike atrial fibrillation) and the isoelectric line has the sawtooth appearance typical of atrial flutter.

The Valsalva manoeuvre raises CVP as the increased intrathoracic pressure impedes venous return to the heart. The CVP reflects the pressure in the right atrium and so is elevated in right-sided heart failure

The bronchial circulation is part of the systemic circulation and receives about 2% of the cardiac output from the left heart under normal circumstances.

• Bronchial arteries arise from branches of the aorta, intercostal, subclavian or internal mammary arteries. The bronchial arteries supply the tracheobronchial tree with both nutrients and O2. Vascular pressures in the bronchial circulation are similar to those in other systemic vascular beds. The J-wave may be present on the ECG in patients with hypothermia and is an additional upward peak immediately following the QRS complex.

Page | 125

The U-wave may be present on the ECG in hypokalaemia and is an additional upward peak that follows the T-wave

The right coronary artery supplies a third of the blood to the left ventricular muscle – the inferior wall.

Increased venous return results in increased ventricular filling. With an increase in ventricular filling during diastole there is an increase in fibre length, which results in an increase in the force of contraction and stroke volume – cardiac output will therefore also increase. This is an adaptation of stroke volume to venous return.

• Although the energy of cardiac contraction is dependent on the resting length of the cardiac muscle fibre, this relationship is not directly proportional. As you increase the resting length of the muscle fibre the energy gained per unit length increase begins to decrease and eventually further length gain will actually decrease the energy of cardiac contraction. The VO2max only predicts cardiorespiratory complications is incorrect as it aids prediction of mortality and morbidity. Cardiopulmonary exercise testing is a preoperative investigation that has been shown to be a useful predictor of mortality and morbidity. It has also been used to triage patients to the appropriate level of postoperative care with success. In a study of pulmonary resection for cancer, A VO2max of >20 ml kg min suggested no increased risk of complications or death. One reading of 11 ml kg min with evidence of myocardial ischaemia or Ve /VO2 > 35 ml kg min then admission to the HDU is more appropriate.

• Anaerobic threshold is reached when VO2max 50%.

• Very high risk is a value of Vo2max of less than 10 ml kg min Ejection fraction is the stroke volume divided by the end diastolic volume. The volume of blood in the ventricles at the end of diastole is known as the end diastolic volume. As the ventricles contract during systole, the volume of blood ejected per beat is known as the stroke volume. The remaining volume in each ventricle is known as the end systolic volume and acts as a reserve which can be utilised to increase stroke volume in exercise. The fraction of end diastolic volume that is ejected is called the ejection fraction – usually equal to about 60%. The ejection fraction is often used clinically as an indirect index of contractility

Inhaled nitric oxide can be used in pulmonary hypertension or right heart failure, the relaxation of the bronchial smooth muscle results in lowered pulmonary vascular resistance. Nitric oxide is a molecule with broad and diverse effects. It was first described in 1980 as a product of endothelial cells that causes vasorelaxation. This product was called endotheliumderived relaxing factor (EDRF) and was eventually shown to be nitric oxide. Nitric oxide is known to be produced by many cell types and to exert a wide range of biological effects. The physiological actions of nitric oxide include relaxation of gastrointestinal smooth muscle and bronchial smooth muscle, maintenance of vascular integrity, and inhibition of smooth muscle migration and proliferation. Endothelial nitric oxide also plays a critical role in haemostasis, making an important contribution to the normal inhibition of platelet function. Inhibition of platelet adhesion is a property of nitric oxide. Nitric oxide is also an important determinant of blood cell flow and is thought to be involved in immune and inflammatory responses via its production in macrophages, lymphocytes and neutrophils. Nitric oxide is synthesised by a family of enzymes, the nitric oxide synthetases.

Page | 126

Extracellular has higher sodium, chloride and bicarbonate levels when compared with intracellular fluid.

• The intracellular component of body water accounts for about 40% of body weight, and the extracellular component 20%.

• Intracellular fluid has higher concentrations of potassium, phosphate, protein and magnesium. In late pregnancy, maternal blood volume increases by 50%. Haematocrit decreases to 30– 35%. After haemorrhage, pregnant women may lose 30% of their blood volume before exhibiting signs of hypovolaemic shock. Blood will be diverted away from the gravid uterus to the mother’s circulation, therefore the fetus will be the first to suffer from the haemorrhage. Fluid replacement must therefore be aggressive and early. It is recommended when possible that the pregnant woman be positioned on her left side to prevent compression of the inferior vena cava by the uterus.

During sleep vagal tone is increased and thus the heart rate is reduced.

One-third of total body water is extracellular and two-thirds intracellular. Of the extracellular fluid volume, only one-quarter of extracellular fluid is intravascular and therefore only onetwelfth (one-third × one-quarter) of infused 5% dextrose will remain in the intravascular space. In comparison, one-quarter of 0.9% saline will remain in the intravascular space as it contains 154 mmol/l of Na , which is similar to the concentration of Na found in the extracellular compartment.

When the pulmonary Artery is wedged and pressure measured on the venous side of the wedge, the pressure measured is equivalent to that in the pulmonary vein and therefore the left atrium.

Baroreceptors are pressure receptors, they respond to alterations in the stretch of the carotid sinus (rise in mean arterial pressure). They are found in the adventitia of the blood vessels and wall of the heart. The arterial baroreceptors are located in the carotid sinus and aortic arch. When the arterial blood pressure drops, the arterial wall is subjected to less stretch, and the sensory nerves coming from the carotid sinus (sinus nerve) and from the aortic arch (depressor nerve) become less active and send fewer impulses. Upon receiving fewer impulses from the baroreceptors, the cardiovascular centres respond by exciting the sympathetic and inhibiting the parasympathetic nervous systems. This leads to an increase in heart rate and stroke volume, and generalised constriction of arterioles (not brain or heart) and veins. On the venous side, receptors are found in the wall of the left and right atria, at the entrances of the inferior vena cava and superior vena cava and in the pulmonary circulation.

• Carotid sinus baroreceptors respond to stretch, not chemical composition. The carotid body is a chemoreceptor sensitive to changes in arterial p(CO2) and arterial p(O2) as well as temperature and pH. The main feedback of this is to the cardiorespiratory centres of the brain. Salbutamol has β-adrenoceptor activity resulting in tachycardia.

Myocardial blood flow is 250 ml/min at rest, represents 5% of the cardiac output and is dependent on arterial pressure. Flow is only seen in diastole. Pain and vasopressin (antidiuretic hormone) reduce myocardial blood flow

Nitric oxide (NO) is a rapidly diffusible gas with a half-life of only a few seconds. It is formed in endothelial cells via the metabolism of arginine and catalysed by nitric oxide synthase (NOS). NO has immunological and inflammatory functions, and also anti-atherosclerotic, and anti-thrombotic effects. It

Page | 127 also contributes to smooth muscle cell relaxation and the control of blood pressure. In this specific scenario impaired brachial artery relaxation after a period of occlusion is a sign of endothelial dysfunction and associated with increased cardiovascular risk. This, at least in part, is due to endothelial damage and reduced NO production; in turn, reduced NO production brings about an increased susceptibility to further endothelial damage.

• Nitrous oxide (not to be confused with nitric oxide – NO) has a chemical formula of N O. It does have medical uses as an anaesthetic and analgesic, it is also known as ‘laughing gas’ – it is not primarily known as a vasoactive agent.

• Endothelin I is a vasoconstrictive agent produced by endothelial cells.

• Serotonin is a vasoconstrictive agent, it is released by platelets as part of the clot-forming response. Its vasoconstrictive effect improves haemostasis.

Sinus tachycardia is the most common ECG finding associated with pulmonary embolism. However, more specifically, this trace shows low-amplitude deflections and an ‘S1Q3T3’ pattern, with a prominent S wave in lead I, a Q wave and an inverted T wave in lead III. There is also T wave inversion in leads V1– V3. These changes are associated with a large pulmonary embolus.

The normal adult blood volume is approximately 7% of body weight (a 70-kg man has a 5-litre circulating blood volume), and in children it is approximately 8–9% of their body weight. The blood

Page | 128 volume of obese patients is estimated using their ideal body weight, as their true weight gives an overestimation of blood volume

Crystalloids such as NaCl solution, pass more readily from the plasma fluid compartment into the rest of the extracellular fluid compartment. Whereas the sodium content in 0.9% NaCl is approximately isotonic and confines the infused volume to the extracellular fluid compartment, hypertonic NaCl will have an even greater effect than normal NaCl, as hypertonic solutions will draw additional water from the intracellular fluid compartment by osmosis.

Cuthbertson’s original description of the two phases of response to injury describes an initial slowing down (ebb), followed by speeding up (flow), of most of the metabolic pathways. The initial ‘ebb’ phase after either trauma or surgery lasts 2–3 days and consists of a slowing down of the intrinsic metabolic activity. The cardiac output reduces, basal metabolic rate slows and glucose tolerance decreases. The ‘flow’ phase that follows for anything from days to weeks after the initial ‘ebb’ reverses these processes. Muscle catabolism and a negative nitrogen balance will also occur during this second phase.

Cardiac output = heart rate × stroke volume. Unlike adults, children have a limited ability to increase their cardiac output by increasing stroke volume due to a higher proportion of fibrous tissue. Paediatric cardiac output is, therefore, significantly dependent on heart rate

Catecholamine levels are significantly elevated during surgery as part of the stress response and will remain so immediately post-operatively. Over the following approximately six hours these levels will return to normal in a paediatric patient.

• In the paediatric population, stress hormones such as cortisol are elevated for a much shorter duration. Unlike adults, at 24 h these levels would have returned to their pre-operative level. Paediatric formula: weight = (age + 4) × 2

Gastric juice is high in hydrochloric acid and potassium. Severe vomiting in this context leads to loss of H (and therefore alkalosis), loss of chloride ions and hypokalaemia The excess loss of Cl depletes extracellular chloride and, with the luminal loss of hydrogen ions, the net result is metabolic alkalosis. The kidney then tries to maintain blood pH by excreting alkaline urine. HCO3 is excreted with Na and K until the overall volume deficit triggers an expansion of the extracellular volume rather than manintenance of pH. Na is reabsorbed, but K is lost via an aldosterone mediated mechanism and this leads to excretion of H ions, thus resulting in a ‘paradoxical aciduria’ in the alkalotic patient

In children under the age of 2 years, a normal urine output should be >2 ml/kg/h.

In children over 2 years old, urine output should be >1 ml/kg/h

Brown fat is required for thermogenesis in the newborn and is located around the scapulas, mediastinum, kidneys and adrenal glands and comprises around 2-6% pf neonate body weight.

In hypothermia, due to anaerobic metabolism, serum lactate is elevated.

Malignant Hyperthermia MH is passed on in an autosomal dominant manner but with variable penetrance. A family history of anaesthetic problems is an important question to ask when planning surgery. Mortality from this disorder is high, although it has improved with the modern use of IV

Page | 129 dantrolene and cooling methods which are the treatment of choice. Can happen with suxamethoneum and many other anaesthetics. Mortality rate is 30%.

Radiation is the transfer of heat through electromagnetic waves and heat loss through this occurs all the time. Radiation is the major source of heat loss in surgical patients contributing up to 40% of heat loss.

• Vaporation; Energy in the form of heat is used during the vaporisation of water. Water evaporates from the body even when not sweating. At very high environmental temperature evaporation is an important source of heat loss. This is not usually the case in an operating theatre and therefore it contributes approximately 15% to heat loss.

• Conduction is when heat is lost by the transfer of energy to adjacent molecules. It occurs by direct contact with an object of a lower temperature. It is a mode of heat loss during surgery contributing to approximately 5% of heat loss.

• Convection describes the loss of heat to air immediately surrounding the body and contributes up to 30% of heat loss during surgery. Noradrenergic sympathetic stimulation will cause vasoconstriction to the extremities and vasodilation to the trunk to reduce heat loss.

• Shivering can double heat production but does not occur in newborn babies. Instead they have an abundance of brown fat that contains many mitochondria to generate ATP production and heat. Temperatures below 35°C can cause mild platelet dysfunction and below 33°C can also affect the clotting cascade resulting in an increased bleeding risk. Hypothermia causes respiratory depression. Metabolic acidosis is typically found in hypothermia and has been attributed to respiratory depression resulting in inadequate tissue perfusion causing a buildup of anaerobic products of respiration such as lactate.

Measurement of paracetamol levels is sensible although this may not change treatment. Accidental paracetamol overdose is a common problem. Understanding the treatment algorithm, and its timings, is important. Acetylcysteine should be given in all patients whose serum concentration of paracetamol is above the treatment line (graph can be found in the British National Formulary) or when the presentation is delayed (>8 hours). Education for the family is also very important, along with further questioning on possible other medications that have been taken, such as aspirin. In this case there may also be safeguarding issues that need to be explored.

• an overdose of paracetamol (>150 mg/kg) which risks hepatotoxicity. If presentation is delayed (>8 hours) and so should be treated immediately with acetylcysteine. Activated charcoal has no place when the ingestion is more than 1 hour ago. A starting dose for a child of 10 years would be 0.1 mg/kg of intravenous morphine. In a child that weighs 30 kg he should be given an initial dose of 3 mg.

Metoclopramide is a dopamine receptor antagonist. It should not be used in children and young adults due to the high incidence of dystonia and extrapyramidal side-effects.

Page | 130

• Hyoscine is an antimuscarinic used as an anti-emetic typically in motion sickness tablets. It would not be a first line choice for post-operative nausea and vomiting.

• Cyclizine is an anti-emetic with anticholinergic and antihistamine actions. It is not associated with acute dystonic events.

• Ondansetron is a 5-HT3 receptor antagonist licensed for use as an anti-emetic in children and, although there are no guidelines for post-operative nausea and vomiting, is useful in the management of vomiting in children. Ondansetron can however induce acute dystonia. Paracetamol intravenous (IV) dose depends, however, on bodyweight, and adults under 50 kg should have only 15 mg/kg per dose.

In modern NHS hospitals there is often a delay to theatre due to the pressures on CEPOD (Confidential Enquiry into Patient Outcome and Death), so in general, once the decision to operate for appendicitis has been made, antibiotics should be started to control sepsis. In doubtful cases of patients with right iliac fossa pain undergoing observation, antibiotics should be held, because their administration can reduce local inflammation and distort the clinical picture, making a diagnosis more difficult; of course if these patients become septic then they merit antibiotic therapy.

• In some instances of a very unclear diagnosis of appendicitis, this could be considered because the risks of administering antibiotics blindly may expose a patient to risk. Antibiotics on induction, part of the World Health Organization (WHO) checklist, has been shown to improve outcomes. In this case, she is clearly unwell with abdominal sepsis so delay in treatment with antibiotics will worsen her probable outcome. It is sensible to delay an immunisation if the child has a fever. However, it is only a relative contraindication and exceptional circumstances may mean that immunisation is sensible.

• Flu vaccine is egg based and should be avoided if anaphylaxis has occurred, but mild intolerances are not a reason to avoid an immunisation. MMR is not contraindicated in egg allergy.

• Live vaccines depend on the host’s immunity to work. If a child is immunosuppressed, then there is significant concern that the live vaccine could result in active disease. Therefore it is an absolute contraindication. Secretin is secreted by the S-cells in the duodenum in response to the arrival of gastric chyme, which is acidic, reducing the pH of the intestinal lumen. Secretin binds to G-coupled protein secretin receptors in various tissues to exert its effects. Secretin increases the release of bicarbonate from the duodenum and release of bile from the gall-bladder. It also stimulates pepsin secretion from the stomach and insulin release from the pancreas. Secretin inhibits gastrin release and gastric acid secretion.

• Pepsin is the enzyme responsible for the breakdown of proteins and is produced in the inactive form of pepsinogen, by the chief cells of the stomach.

• Amylase is an enzyme involved in carbohydrate metabolism, produced in the pancreas and the salivary glands.

• Lipase is produced in the pancreas and is involved in fat metabolism, breaking down triglycerides into monoglycerides and two fatty acids.

Page | 131

• Elastase is an enzyme produced in the acinar cells of the pancreas involved in the metabolism of proteins The rate of gastric emptying is influenced by two factors: volume of stomach contents and composition of stomach contents. In the absence of stomach distention there is little stimulation for gastric motility and therefore the rate of emptying is slower. Isotonic fluids are emptied at a maximal rate, hypertonic meals have a slower rate of gastric emptying.

• Gastric emptying rate is accelerated when a patient lies supine rather than standing. The proton pump (H /K -ATPase) is found in the apical (luminal) membrane of gastric parietal cells and secretes protons into the lumen of the stomach in exchange for potassium ions. Histamine-stimulated acid production is dependent on the proton pump.

• Histamine binding to receptors on the basolateral membrane of parietal cells activates adenylate cyclase leading to a high intracellular cyclic AMP concentration and activation of protein kinase A. This process has the effect of recruiting more proton pumps (H /K -ATPase) from the cytoplasm to the luminal (apical) membrane of the parietal leading to increase acid production.

• In the proton pump a proton is exchanged with a potassium ion (H /K -ATPase) at the expense of an ATP molecule.

• Acetylcholine-stimulated acid production is again dependent on the proton pump. Acetylcholine binds muscarinic (M ) receptors on the basolateral membrane of the parietal cells, a G protein coupled receptor, and stimulates the secretion of gastric acid via the proton pump. Sodium and water absorption are the main functions of the colon. Sodium is actively absorbed with the aid of a basolateral Na /K ATPase that creates an electrochemical gradient. Water absorption in the colon is via a passive mechanism mostly via enterocyte tight junctions, and is dependent on the absorption of sodium and other solutes, creating electrochemical gradients. Chloride is absorbed in the colon with sodium.

• Bile acids are absorbed in gastrointestinal tract both actively and passively. Active absorption occurs in the ileum. Passive forms of absorption can occur at any point in the gastrointestinal (GI) tract and includes: ionic, non-ionic and micellar diffusion mechanisms.

• Amino acids are absorbed in the small intestine, with the majority of absorption occurring in the jejunum and the remainder in the ileum. They are absorbed by co-transport with Na through sodium dependent amino acid transporters

• In total, 80% of water is absorbed by the small intestine. Water absorption in the colon is via a passive mechanism mostly via enterocyte tight junctions, and is dependent on the absorption of sodium and other solutes, creating electrochemical gradients Parietal cells of the stomach have a dual secretory function. They secrete hydrochloric acid, responsible for the acidic environment of the stomach, but also secrete intrinsic factor. Intrinsic factor is a glycoprotein essential for the absorption of vitamin B by the terminal ileum. In the absence of intrinsic factor, vitamin B deficiency, a condition called pernicious anaemia, occurs that trebles the patient’s risk of developing gastric cancer.

Page | 132

Maltase deficiency secreted by the villi of the small intestine will result in a reduction in the hydrolysis of maltose, therefore there is an increased passage of maltose in the stool.

The cephalic phase of pancreatic secretion involves the integration of sensory inputs, such as sight, smell and taste of food as well as the process of mastication and the presence of certain molecules binding on receptors in the mouth. It is under the control of the vagus nerve. Inputs are integrated in the brainstem and, specifically, the dorsal vagal complex. It is transmitted down to the pancreas via the vagus nerve through cholinergic fibres innervating the acinar cells. Stimulation of the acinar cells leads to pancreatic enzyme secretions. Therefore, a vagotomy would interrupt this circuit of information and reduce the pancreatic secretion from the cephalic phase

• The cephalic phase of the pancreatic meal response accounts for about 20–25% of the total pancreatic exocrine secretion. The vasoactive intestinal peptide, is a 28 amino acid peptide hormone belonging to the glucagon/secretin family. It has various roles in the intestine including: causing vasodilation in the gastrointestinal resistance vessels, relaxation of enteric smooth muscle, inhibition of gastric acid secretion and stimulation of pancreatic juice and bile secretion. Other agents that cause gastrointestinal resistance vessel dilation include: gastrin, glucagon and cholecystokinin.

Vasopressin, also known as antidiuretic hormone, is released from the posterior pituitary and has a vasoconstrictor effect on the gastrointestinal resistance vessels.

Human saliva is composed of water (99.5%) as well as electrolytes, epithelial cells, white blood cells, enzymes and antimicrobial agents. The enzymes include lipase and amylase, which commence the process of digestion of dietary fats and starches respectively in the mouth. Antimicrobial agents found in saliva include IgA antibodies and lysozyme.

• Normal saliva production in adults varies from 1–2 litres per day. The secretion of saliva from the salivary glands is mediated by the autonomic nervous system. The submandibular glands contribute about three-quarters of the saliva, with one-quarter produced by the parotid gland. The other salivary glands only contribute a small amount of saliva to the total amount produced Absorption of calcium occurs in the duodenum and jejunum, and is both active – under the regulation of calcitriol – and passive, through tight junctions.

Somatostatin inhibits the release of gastric acid. Secretion of somatostatin is stimulated when the acid of the stomach drops too low. Somatostatin inhibits the release of gastric acid secretion both by having a direct effect on the parietal cells but also by inhibiting the release of histamine and gastrin, both of which stimulate gastric acid secretion.

Secretin is a hormone secreted from the S-cells in the duodenum in response to the arrival of acidic gastric chyme in the duodenum. Secretin influences the release of pancreatic enzymes by increasing the rate of bicarbonate synthesis.

Electrical activity in the gastrointestinal tract occurs in slow waves, generated and propagated by the interstitial cells of Cajal, relying on calcium current. These slow waves cause depolarisation of the cells in the wall of the bowel and create a peristaltic wave.

Page | 133

• Segmentation, ie contraction of isolated segments found at intervals along the intestine, efficiently mixes the chyme with mucus and hydrolytic enzymes. It is peristalsis that moves the chyme along the intestine.

• It is increased by the action of the parasympathetic system, with acetylcholine acting at the muscarinic receptors. Vasopressin and secretin decrease gastric acid secretion and have been shown to decrease gastric blood flow. Additionally, noradrenaline decreases gastric blood flow by causing vasoconstriction of the gastric arterioles. Similarly, hypovolaemia and shock can decrease gastric blood flow. Maintaining a good gastric blood flow is necessary for protecting the gastric mucosa from damage from the acid gastric contents.

• Vagal stimulation, acetylcholine, histamine and gastrin are all known to increase gastric acid production and have been found to increase gastric blood flow as well. Acetylcholine also causes a direct vasodilator effect on the gastric mucosa arterioles. The salivary glands are supplied both by the sympathetic and the parasympathetic nervous system. The sympathetic nervous system promotes thicker saliva production, necessary for respiration. The parasympathetic nervous system on the other half promotes the secretion of thinner saliva, for facilitation of digestion.

• Around 1.5 litres (1500 ml) of saliva is produced by the salivary glands every day Calcitonin, is a peptide hormone produced in the C-cells or parafollicular cells of the thyroid gland. The C-cells sense an increase in the concentration of calcium in the serum and then secrete calcitonin to bring about a reduction of calcium. Calcitonin inhibits the action of osteoclasts and therefore inhibits bone resorption and also inhibits the reabsorption of calcium and phosphate ions by the kidney, promoting their excretion in the urine.

In long-term hypercalcaemic syndromes the parafollicular cells can hypertrophy.

Glucose will not diffuse through a cell membrane against a concentration gradient. Insulin facilitates the diffusion of glucose across many cell membranes including cardiac muscle cells, skeletal muscle cells, some smooth muscle cells and adipose tissue cells.

Hypoglycemia is defined as a blood sugar below the normal range (3.5–5.5 mmol/l). It most commonly occurs in diabetic patients who have taken their normal insulin dose when the carbohydrate intake was low. Other causes include an insulin secreting tumour of the pancreas (insulinoma), hypothyroidism, severe infections, liver failure. Brain metabolism is highly dependent on glucose as an energy source. Initial steps in brief or mild hypoglycaemia include the release of glucagon to promote glycogenolysis, use of the glycogen stores in the liver and gluconeogenesis. Hypoglycaemia triggers the release of adrenaline from the adrenal glands, as a stress hormone. Increased circulating adrenaline causes the symptoms of sweating, shaking, tachycardia and a feeling of hunger all of which alert the patient to act upon the symptoms and have a meal. Adrenaline also acts synergistically with glucagon in gluconeogenesis. If the hypoglycaemia is not resolved, the brain does not have adequate support and the patient can experience seizures and coma.

Delivery of blood to the macula densa with low Na levels induces the production of renin.

Page | 134

• The renin–angiotensin–aldosterone system is not activated in patients who are induced with isoflurane and hypotensive anaesthesia is maintained. This approach is widely used to prevent haemorrhage during surgery and has been reported as safe. Prolactin secretion is from the lactotroph cells of the anterior pituitary gland under the control of the hypothalamus. The hypothalamic neurosecretory tuberoinfundibulum neurones of the arcuate nucleus in the hypothalamus secrete dopamine, which inhibits prolactin secretion from the anterior pituitary. It is the only pituitary hormone whose control is inhibitory. Oestrogens increase the growth of the lactotroph cell and also inhibit the release of dopamine, both of which lead to an increase in prolactin concentration during pregnancy.

• Secretion of prolactin from the anterior pituitary is inhibited by dopamine secretion from neurones in the hypothalamus Exercise, is one of the physiological stimulators of growth hormone secretion along with sleep, hypoglycaemia and circulating estradiol. A diet rich in protein is also considered a stimulator of growth hormone secretion. Growth hormone is secreted by the somatotroph cells in the anterior pituitary gland.

Thyroglobulin is synthesised within the follicular epithelial cells and secreted into the follicular lumen. It is an important step in production of thyroid hormones and organification of iodine with tyrosine molecules produces T or T . This selection depends upon the number of iodine molecules.

• Thyroid peroxidase first generates I2 by oxidising iodine ions present in the follicular lumen. Thyroid peroxidase then ‘organifies’ the generated I by covalently linking it with the tyrosine residues present in thyroglobulin. This step generates either a single or doubly iodinated species of tyrosine, termed ‘monoiodotyrosine (MIT)’ or ‘di-iodotyrosine (DIT),’ respectively. Progesterone inhibits lactation during pregnancy. The fall in serum progesterone with the passage of the placenta seems to be the important event in establishment of lactogenesis. Furthermore, progesterone administration inhibits casein and α-lactalbumin synthesis in vitro. Once lactogenesis is initiated, prolactin appears to be the key hormone in maintenance of milk synthesis (galactopoiesis) and secretion.

LH hormone is produced by the anterior pituitary once stimulated by gonadotrophin releasing hormone (GnRH). In men LH acts on the interstitial cells of the testes (Leydig cells) to secrete testosterone. LH is produced by gonadotropes, which are basophilic cells in the anterior pituitary gland.

• Follicle-stimulating hormone (FSH) and testosterone are required for spermatogenesis (division of spermatogonia to form spermatids) and spermiogenesis (maturation of spermatids to mature sperm). In the post-partum period, during lactation, there are high levels of circulating prolactin hormone. The high circulating levels of prolactin act as an inhibitor for the release of gonadotrophins [FSH and luteinising hormone (LH)], which are necessary to stimulate follicle maturation and lead to ovulation and menstruation. This occurs in the early post-partum period. After about 40 days post-partum, the prolactin sensitivity drops significantly and gonadotrophins start increasing in concentration stimulating follicle maturation.

Page | 135

As 99% of calcium in the body is bound in the skeleton, only 1% of the total body calcium is available for buffering changes in Ca balance in the body. This 1% known as total serum calcium and is composed of three fractions: the protein bound calcium which is about 40– 50%, the complexed calcium (10%) and the free ionised calcium which is usually about 50%. The calcium concentration in the serum is tightly controlled in the normal range of 2.20–2.60 mmol/l.

The primary effects of aldosterone on the renal system are to increase reabsorption of sodium and increase excretion of potassium and hydrogen ions. Aldosterone acts by increasing the number and activity of the active transport systems in the distal renal tubule to sodium and potassium. Lisinopril by blocking these actions will lead to increased sodium excretion. Drugs with the suffix 'pril' are angiotensin converting enzyme inhibitors – another example is ramipril

A new diagnosis of diabetes mellitus and diabetes insipidus can be mistaken for overactive bladder syndrome in the absence of a careful history. In central diabetes insipidus, failure of antidiuretic hormone (ADH), also known as vasopressin, production prevents resorption of water from the kidneys, whereas in nephrogenic diabetes insipidus, the kidneys are insensitive to the ADH produced. Excessive water is lost via the urinary tract, leading to polyuria and hypovolaemia. Thirst response is appropriately activated, resulting in polydipsia

• Psychogenic polydipsia is excessive intake of fluid in the absence of the physiological stimuli to drink. This is, in approximately 30% of cases, associated with dilutional hyponatraemia. It is not associated with hypernatraemia. Vit D main function in calcium homeostasis includes an increase in the absorption of both calcium and phosphate from the gut.

Oxytocin is a hormone secreted by the posterior pituitary gland. The baby’s suckling stimulates the afferent nerves that connect with the paraventricular and supraoptic nuclei of the hypothalamus, which in turn stimulates pulsatile release of oxytocin from the posterior pituitary gland. Oxytocin binds to receptors on the breast myoepithelial cells surrounding the ducts leading to contraction of the smooth muscle and milk expulsion

The half-life of T3 is only 1 day. T3 is four times more active than the more abundant T4 . In patients with hypothyroidism or post thyroidectomy, Levothyroxine T4 is given as a replacement therapy. T3 is responsible for most of the functions, but due to short half-life daily more frequent dosing would be required. Moreover, in cases of stress, increased dosage may be needed on a daily basis

• The half-life of T4 is 5–7 days. In peripheral tissues it acts as a reservoir for T3 FSH and testosterone synergise to stimulate the synthesis of androgen binding protein (ABP). ABP keeps the levels of testosterone high in the tubular fluid.

Calcium-sensing receptors expressed in the parathyroid glands are responsible for detecting extracellular calcium concentration. PTH secretion by the parathyroid chief cells is stimulated or inhibited in response.

Androgens are steroid hormones, found in both men and women. The main androgen seen in men is testosterone. Other androgens include: dihydrotestosterone and androstenedione.

Page | 136

• Androgens are involved in the development of the testes, act as paracrine hormones on the Sertoli cells leading to spermatogenesis.

• Additionally, androgens stimulate the development of secondary male sexual characteristics. They have anabolic effects by increasing protein synthesis and muscle mass.

• Androgens increase erythropoiesis by stimulating erythropoietin release. The iron in the ferrous state (Fe2) is more efficiently absorbed than in the ferric state (Fe3). In the physiological pH the iron is oxidised to the ferric state (Fe3) which is insoluble. Gastric acid secretions however, lower the pH and assist the absorption of iron in the ferrous state in the duodenum and first part of the jejunum.

The main cause of the change in respiration seen in pregnancy is the high levels of circulating progesterone. This, increases the sensitivity of the respiratory centres of the hypothalamus to small increases in carbon dioxide, A small rise in p(CO2) has the effect of a significant rise in minute ventilation and respiratory rate. This as a result leads to hyperventilation and a lower normal p(CO2).

Zona fasciculata of the adrenal cortex produces cortisol and is commonly affected in Addison’s disease. 90% of the cortex is destroyed before patients become symptomatic The Zona glomerulosa and fasculata are destroyed with sparing of the zona reticularis and medulla.A deficiency of cortisol, as occurs in Addison’s disease, can produce hyperkalaemia and mild acidosis. In primary cases adrenal cortex is defective while in secondary cases anterior pituitary does not produce enough ACTH) Pigmentation differentiates between the two as high ACTH has a partial function of melanin stimulating hormone. hCG is a glycoprotein hormone, produced in pregnancy, which is made by the embryo soon after conception and later by the syncytiotrophoblast (part of the placenta). Its role is to prevent the disintegration of the corpus luteum of the ovary and thereby maintain progesterone production that is critical for a pregnancy in humans. hCG may have additional functions, for instance it is thought that it affects the immune tolerance of the pregnancy. Early pregnancy testing generally is based on the detection or measurement of hCG. Neutralisation of hCG activity will result in disintegration of the corpus luteum, with subsequent death and expulsion of embryo.

Depolarisation of the plasma membrane triggers passive release of calcium ions from the sarcoplasmic reticulum through specific channels along a gradient.

• Contraction is sustained until the calcium ions are actively transported back into the sarcoplasmic reticulum and relaxation occurs There is an increase in the release of catecholamines and thyroxin in response to hypothermia.

Glucose, amino acids and bicarbonate are reabsorbed along sodium in the early portion of the proximal convoluted tubule. Glucose is removed from the urine by secondary active transport.

• The descending loop of Henle is permeable to water due to the presence of acquaporin-1 but the ascending limb is impermeable to water. Na , K and Cl are co-transported out of the ascending limb, which encourages the movement of water out of the descending limb, making the fluid in the descending limb hypertonic. In the ascending limb it becomes more dilute

Page | 137

because of the movement of Na and Cl out of the lumen and at the top is now hypotonic to plasma. The loop of Henle is not involved in glucose resorption.

• The distal convoluted tubule is relatively impermeable to water and is involved in the continual removal of the solute, making the tubular fluid further diluted. It is not involved in glucose resorption.

• The collecting ducts are under the influence of antidiuretic hormone (ADH) (vasopressin) to control water absorption. In the absence of ADH, water can enter the urine, promoting diuresis. In the presence of ADH, aquaporins allow the collecting duct to become permeable to water allowing reabsorption and inhibiting diuresis. By various negative-feedback mechanisms involved in homeostasis, a rise in ambient temperature is typically accompanied by increased respiratory rate.

Carbon monoxide binds to haemoglobin to form carboxyhaemoglobin. The presence of carboxyhaemoglobin leads to inaccuracy in the pulse oximeter, as it will read a falsely high oxygen saturation

The Functional Residual Capacity FRC is the volume of air in the lungs at the end of passive expiration. Emphysematous lungs are more compliant, resulting in a greater FRC.

• Fibrosing alveolitis (idiopathic pulmonary fibrosis) is a restrictive lung disease. FRC is reduced.

• General anaesthesia reduces FRC by approximately 20% due to loss of inspiratory muscle tone.

• FRC is reduced in obesity Dynein is a family of cytoskeletal motor proteins that control the beat of the cilia. It is absent in Kartagener’s syndrome (primary ciliary dyskinesia).

• The mucociliary elevator moves at approximately 1 mm/min in the peripheral airways. The flow rates in the trachea can be much higher, ranging from 5-10 mm/min.

• The respiratory tract secretes about 10-100 ml of mucus a day) in health).

• Unlike bronchi, bronchioles do not have any cartilage in their walls.

• Bronchioles are up to 1 mm in diameter Carbon monoxide has a high affinity than oxygen for haem groups. It binds to the intracellular cytochrome system, causing cellular dysfunction.

• The p (O2) will remain normal. This is because only a small amount of CO is required to bind to haem groups to cause cellular dysfunction and CO poisoning. The amount needed to cause poisoning is low enough that it does not alter the amount of oxygen physically dissolved in the plasma and hence the PaO2. However, ABGs will usually reveal a metabolic acidosis in patients with serious poisoning.

• Carbon monoxide has a 240-fold greater affinity for haemoglobin than oxygen • Headache and confusion are early symptoms of carbon monoxide poisoning. At a 50% carboxyhaemoglobin level the patient will experience hallucinations, ataxia, coma then death.

Page | 138

The Jendrassik manoeuvre involves clenching the teeth and pulling interlocked fingers against each other. It reinforces tendon reflexes.

The Heimlich manoeuvre is a technique to treat asphyxiation. The patient’s upper abdomen is rapidly compressed from behind to cause forced exhalation.

The functional residual capacity is the sum of the expiratory reserve volume and the residual volume

Hyperventilation causes hypocapnia, which causes cerebral vasoconstriction leading to decreased cerebral blood flow. This mechanism can be used therapeutically in hypocapnic ventilation to treat cerebral oedema.

Central chemoreceptors, located in the ventral medulla, are the most important stimulus for respiration. They respond to increased H concentration of cerebral extracellular fluid (which is directly linked to CSF). H /HCO cannot easily cross the blood–brain barrier, but CO does so readily. This frees H ions, causing a low CSF pH and stimulates the central chemoreceptors to fire

The majority of CO2 is transported as bicarbonate. CO2 combines with water to form carbonic acid (H CO3 this is catalysed by carbonic anhydrase), which then dissociates to form a H ion and HCO3

PEFR should be greater than 500 1/min in men and 400 1/min in women.

A Wright’s peak flow meter is used to measure PEFR and is a handy bedside test

A right-left shunt refers to oxygen-poor blood from the right heart that has entered the left heart without undergoing gas exchange in the alveolar capillaries (ie it is shunted away from sites of gas exchange). The physiological shunt is the sum of the anatomical shunt (blood passing from the right ventricle to the systemic circulation via normal anatomical pathways, eg the bronchial vessels, without passing through the pulmonary alveolar capillaries), and the element of pulmonary alveolar capillary blood that has passed through non-aerated or poorly aerated alveoli. Therefore the physiological shunt is always at least as great as or greater than the anatomical shunt

Peripheral chemoreceptors in the aortic arch and carotid bodies are sensitive to arterial oxygen levels. In hypoxia, they will stimulate the respiratory centres to increase ventilation, leading to a decreased p(CO ) and a subsequent respiratory alkalosis.

The pulmonary vessels can accommodate up to 500 ml in an adult man under normal resting conditions. This means that cardiac output can be increased quickly before there is a match in the venous return, as the blood stored in the pulmonary vessels act as a reservoir for the left ventricle.

Hypoxic pulmonary vasoconstriction results in decreased blood flow to poorly ventilated alveoli, improving matching of blood flow to ventilation. inspiratory reserve volume can be inhaled at the end of normal inspiration. The normal adult values range between 1900 and 3300 mls.

• Total lung capacity ranges from 5–6.5 litres in an adult man. The total lung capacity is the sum of the vital capacity and the residual volume.

• Normal peak expiratory flow in an adult man is 520–700 l/min, depending on their age and height.

Page | 139

• Functional residual capacity (the volume at the end of normal expiration) is 2–3 litres in an adult man. The functional residual capacity is the sum between the residual volume and the expiratory reserve volume. It is less than 50% of the vital capacity.

• Normal tidal volume is 300-500 mls. The tidal volume is the amount of air that can be inhaled or exhaled during one respiratory cycle. This depicts the function of the respiratory centres, respiratory muscles, and the mechanics of the lungs and chest walls. The normal adult value is 10% of the vital capacity, thus apporoximately 300 – 500 mls, (6-8 ml/kg); thus number can increase to up to 50% of the vital capacity during exercise. The residual volume (and therefore functional residual capacity and total lung capacity) cannot be measured directly by spirometry. They are measured by either whole body plethysmography, or by using the helium dilution or nitrogen wash-out techniques.

The final seven branches of the bronchoalveolar tree are the respiratory zone and gas exchange can occur here.

• About 1.5–2% of O is dissolved in plasma.

• The conducting zone is made up of the first 16 branches. Its function is to warm and moisten air, filter air, and assist in phonation. the Bohr effect in metabolically active tissue, the oxygen-haemoglobin dissociation shifts to the right, ie for a given partial pressure of O2, oxygen will more readily dissociate. This situation means more oxygen is delivered into the tissues. Active metabolic tissue will have a decreased pH (lactic acid production), increased temperature, increased 2,3-DPG levels and increased CO2 levels. The inverse of any of these factors will shift the curve to the left.

A shift of the oxygen dissociation curve to the left is characteristic of fetal haemoglobin. When compared with adult haemoglobin, it is composed of two alpha and two gamma chains, instead of the usual two alpha and two beta chains of adult haemoglobin. This arrangement assists in the transfer of oxygen across the placenta from the maternal to the fetal circulation. The corollary of this is that fetal tissue oxygen levels have to be low to permit the release of oxygen from the haemoglobin.

Normal mean pulmonary arterial pressure is 15 mmHg. This is the hydrostatic pressure generated by the right ventricle and is 1/6 that of systemic arterial pressure generated by the left ventricle. The pulmonary circulation lacks high resistance arterioles and so the pressure drop between the pulmonary arteries and capillaries is minimised.

• 90 mmHg is the normal systemic mean arterial pressure. In a normal person each 100 ml of blood contains about 15 g of haemoglobin and each gram of haemoglobin can bind to 1.34 ml of oxygen when it is 100% saturated (15 × 1.34 = 20 ml O2/100 ml blood).

Vital capacity would be higher in this young patient. If a patient is a 70 kg man, vital capacity is approximately 70 ml/kg ie 70 × 70 = 4900 ml.

The normal amount of pleural fluid is 2–10 ml. It acts as a lubricant to allow the visceral and parietal layers to move smoothly against each other during respiration.

Page | 140

At the distal convoluted tubule, aldosterone acts to increase the Na /K permeability of the luminal surface of the cells and causes resorption of Na and H2O in exchange for K , which is excreted into the urine. It also acts in the collecting ducts to reabsorb H2O to secrete H ions, thereby regulating plasma HCO3 levels.

Glucose is reabsorbed by using the energy (ATP) provided by co-transport of Na ions down their electrochemical gradient. (Secondary active transport)

An ideal substance for measuring the glomerular filtration rate (GFR) should be freely filtered at the glomerulus and neither secreted nor reabsorbed by the tubules.

• Creatinine is not an ideal substance for measuring GFR as a small amount is secreted by the tubules. This means that slightly more creatinine is excreted in the urine than is filtered. Baseline creatinine is also affected by an individual’s muscle mass.

• Inulin, a polysaccharide molecule, meets these criteria for an ideal substance Glucose is able to cross the glomerular basement membrane into the ultrafiltrate. In normal individuals it is then actively reabsorbed.

Prostaglandins (PGE2 and PGI2) increase the renal blood flow and the glomerular filtration rate

• The renal blood flow and renal perfusion are controlled by several factors including hormones and the sympathetic nervous system. Atrial natriuretic peptide causes dilatation of the afferent arterioles and increases the renal blood flow and the glomerular filtration rate. Prostaglandins (PGE2 and PGI2) increase the renal blood flow and the glomerular filtration rate. Angiotensin II (not angiotensinogen) are produced systemically, and within the kidney, primarily constricts the efferent arterioles to maintain glomerular filtration rate. Adenosine triphosphate selectively vasoconstricts the afferent arteriole and so reduces the renal blood flow. Endothelin is a potent vasoconstrictor secreted by endothelial cells of arterioles, which decreases the renal blood flow. Glucocorticoids dilate the afferent arterioles and increase the renal blood flow and the glomerular filtration rate. Likewise, nitric oxide also causes vasodilatation and increases the renal blood flow In the normal individual the glomerular filtration rate is kept fairly constant at 120 ml/min.

ACE inhibitors block the conversion of angiotensinogen to angiotensin II. Angiotensin II causes constriction of the efferent arteriole to increase the hydrostatic pressure within the glomerulus increasing the filtration rate. Blocking this action causes a decrease in the filtration rate

Proximal convoluted tubule is the site of phosphate reabsorption under control of parathyroid hormone. In the proximal convoluted tubule PTH acts to decrease the resorption of phosphate. PTH reduces reabsorption of phosphate in the PCT, however, because it also enhances the uptake of phosphate from the intestine and bones into the blood, the responses to PTH cancel each other out, and the serum concentration of phosphate remains approximately the same.

In diabetes insipidus, appropriate water intake from thirst will adequately compensate for the potential excess volume loss. Diabetes insipidus is a condition where large amounts of urine are produced due to the inability of the kidney to concentrate urine. The cause can either be cranial which is due to the lack of ADH produced by the posterior pituitary, or nephrogenic which is when there is adequate levels of

Page | 141

ADH but the kidney does not respond to it. Cranial diabetes insipidus can be caused by brain trauma or a pituitary tumour among other things. Nephrogenic diabetes insipidus can be caused by acquired diseases such as amyloidosis, drug therapy such as lithium or hypercalcaemia.

GFR can decrease by as much as 50% before plasma creatinine rises beyond the normal range. Consequently, a normal creatinine does not necessarily imply normal renal function, although a raised creatinine does usually indicate impaired renal function.

Fluid in the distal end of the ascending limb of loop of Henle is hypotonic regardless of the state of hydration because of the active reabsorption of sodium chloride but not water, which is impermeable in this tubular segment. Neurophysiology The knee-jerk reflex is the best-known example of a monosynaptic stretch reflex. Passive stretch of the intrafusal spindle fibres through tapping the tendon with a tendon hammer results in generation of afferent action potentials. These are transmitted via the dorsal roots of the spinal nerves to the ipsilateral ventral grey matter where the only synapse with the efferent neurone occurs. The action potential is transmitted through the ventral nerve roots via an alpha-motor neuron, which synapses with the motor endplate of the muscle fibres of the knee extensors.

The parasympathetic neurons are formed at the upper and lower extremities of the spinal cord, in the mid-brain and hindbrain (fibres are carried by cranial nerves III, VII, IX and X) but also in the sacral spinal cord (S2–S4).

• The ganglia of the parasympathetic nervous are closer or are within the end organ

• The neurons of the sympathetic nervous system are formed throughout the thoracic and upper lumbar spine. The ganglia of the sympathetic nervous system are found in a chain either side of the spine. The ganglia of the parasympathetic nervous system tend to lie near the target organ (eg coeliac ganglion which supplies the foregut).

• The neurotransmitter at the synapses in the ganglia of both the sympathetic and parasympathetic nervous systems is acetylcholine. The synapses of the postganglionic fibres with the end organs in the sympathetic nervous system involve catecholaminergic neurotransmission In normal physiological conditions, the cerebral blood flow (CBF) is usually maintained by both pressure and chemical autoregulation. The pressure autoregulation is facilitated by reflex constriction or dilatation of the precapillary cerebral vasculature in response to mean arterial pressures between 50– 160 mm Hg.

The CSF has a protein content that is approximately 0.5% that of plasma. The low protein content of the CSF prevents some proteins and amino acids acting as ‘false neurotransmitters’.

• Most of the CSF is produced by the choroid plexus, which is situated in the lateral, third and fourth ventricles. CSF is absorbed directly into the cerebral venous sinuses through the arachnoid villi, or granulations, by a process known as mass or bulk flow.

• The composition of CSF is different to plasma. Of importance to mention are the concentrations of K , Ca , bicarbonate, glucose and protein which are lower in CSF than in plasma. This is to

Page | 142

prevent high concentrations of these electrolytes inadvertently exciting neurones present within the brain substance. Resting neuron is positively charged outside. The inside of a neurone is rich in chloride ions and a lesser amount of potassium ions. The outside of the neurone is rich in sodium ions

Aδ fibres are small myelinated fibres with rapid conduction responsible for the rapid sharp localised pain.

• Aα fibres are important for proprioception, not pain transmission.

• Aβ fibres are large myelinated fibres that usually respond to light touch.

• B fibres are not involved in pain transmission and are part of the autonomic nervous system.

• C fibres are small unmyelinated fibres with a slow conduction speed. C fibre stimulation produce a slow dull pain Increasing the membranes conductance to potassium will result in K+ efflux and the membrane potential approaching the value dictated by the potassium equilibrium potential, which is about −90 mV for neurones.(Hyperpolarization)

When acetylcholine is released into the synaptic trough it attaches to acetylcholine-gated ion channels on the postsynaptic membrane, causing them to open. The net effect of opening the channels is to allow large numbers of sodium ions to pour inside the fibre. This creates a local potential inside the fibre that initiates an action potential at the muscle membrane.

Stimulation of the sympathetic nervous system results in the flight or fight response – including an increase in the rate of breathing (tachypnoea).

• Watery salivation is increased by parasympathetic stimulation

• Sweating increases with sympathetic stimulation Merkel’s skin receptors are slowly adapting skin receptors that detect pressure, texture, and low frequency vibration and can be appropriately evaluated by static two-point discrimination.

• Free nerve endings receive sharp acute, and chronic slow pain sensation.

• Meissner corpuscles are rapidly -adaptive receptors to light touch and vibration.

• Pacinian (‘lamellar’) corpuscles are rapidly adapting and especially detect vibration

• Ruffini corpuscles detect stretching of the skin. The thalamus serves as a relay station, receiving pain stimuli from ascending sensory tracts.

• The postcentral gyrus receives pain stimuli

• Proprioception is conveyed in the dorsal columns.

• Spinal reflex activity can be modulated by the corticospinal tract. This explains why in an upper motor neurone lesion there is the development of ‘spastic paresis in which there is increased tone due to uninhibited reflex loops.

Page | 143

• The lateral spinothalamic tract conveys temperature sensation. Most neurones of the spinothalamic tract decussate via the anterior white commissure to the contralateral side of the cord. The resting membrane potential of excitable cells is largely due to the selective permeability of the cell membrane to potassium ions.

The Na /K pump generates the ion gradient across the cell membrane (ie, high intracellular K , high extracellular Na ), but it is the back diffusion of K ions through K channels that are open at rest, which charges the cell membrane.

Rhythmic discharge of neurons in the medulla and pons (the cardiorespiratory centre) produces automatic respiration, and so transection of the brainstem below the medulla stops the respiration

Neurons are supported by glial cells in the central nervous systems. Examples of glial cells include: astrocytes, oligodendrocytes, microglia and ependymal cells

• Dendrites transmit stimuli towards the cell body

• Axons transmit stimuli away from the cell body

• Neurons transmit action potentials more rapidly if they are myelinated.

• Neurones transmit more rapidly if the axons are of larger diameter. When normal, the CSF contains less glucose than is contained in the blood.

• The CSF contains less protein than is contained in the blood.

• The pH of CSF is 7.33, whereas that of plasma is 7.4 due to the higher pCO in CSF than in plasma. Pain impulse received in the dorsal horn can be modulated by other descending spinal inputs. Descending fibres modulate nociceptive inputs via the dorsal horn. It is a site of spinal integration.

Sympathetic innervation to the adrenal medulla is unique in the regard that there is no synapse at ganglia between the spinal cord and organ. The ‘preganglionic’ neurone innervates the Chromaffin cells of the adrenal medulla (through acetylcholine), causing direct release of catecholamines into the bloodstream.

The sympathetic nerves originate in the spinal cord from segments T1– L2. They leave the spinal cord in the anterior roots and pass to the paired sympathetic chains

Page | 144

The primary motor cortex is located in the dorsal part of the precentral gyrus and the anterior bank of the central sulcus. The primary motor cortex (or M1) works in association with premotor areas to plan and execute movements. M1 contains large neurons known as Betz cells, which send long axons down the spinal cord to synapse onto alpha-motor neurones, which connect to the muscles. A stroke affecting this region will result in contralateral weakness.

• The postcentral gyrus is known as the primary somatosensory cortex. It is responsible for the conscious sensation of touch. A stroke affecting this region would cause contralateral paraesthesia.

• The cerebellar vermis is a posterior, midline cerebellar structure. A stroke affecting this region would cause postural instability. Sympathetic stimulation will cause peripheral vasoconstriction by acting on alpha adrenergic receptors.

Acetylcholine and noradrenaline are the two main neurotransmitters secreted in the autonomic nervous system. Neurones secreting acetylcholine are called cholinergic and those secreting noradrenaline are termed adrenergic. Both parasympathetic and sympathetic preganglionic neurones are cholinergic.

• Most postganglionic sympathetic neurones secrete adrenaline or noradrenaline. Exceptions to this is supply to the sweat glands (acetyl choline) and the adrenal medulla which has a direct supply from preganglionic fibres (through acetyl choline).

• Almost all postganglionic parasympathetic neurones are cholinergic. Type II diabetes with poor insulin control may be associated with hepatocellular carcinoma.

• Liver cancer incidence is strongly related to age, with the highest incidence rates being in older males and females. In the UK in 2012-2014, on average each year more than 4 in 10 (44%) cases were diagnosed in people aged 75 and over.

• Men are more commonly affected than women, world-wide this varies between 2:1 and 4:1.

• Common viral risk factors include hepatitis B and C viruses (HBV and HCV). In Europe and North America anti-HCV antibodies are present in around 70% of patients with hepatocellular carcinoma. While in countries with endemic hepatitis B 90% of patients with

• HCC are positive for hepatitis B. Alone, HIV is not a risk factor for HCC although co-infection with HCV and HIV may lead to more rapid progression to HCC.

• Serum a-fetoprotein (AFP) levels are usually raised. Patients with cirrhosis due to HBV, HCV, alcohol, primary biliary cirrhosis or haemochromatosis should be offered a 6-monthly abdominal ultrasound scan and serum AFP test as screening for hepatocellular carcinoma.

Page | 145